1 No. Francais English 1. Parmi les effets secondaires suivants de la

Transcription

1 No. Francais English 1. Parmi les effets secondaires suivants de la
No.
1.
2.
3.
4.
5.
Francais
Parmi les effets secondaires suivants de la chlorpromazine et des
neuroleptiques phénothiaziniques, quel est celui qui s'explique
par leur action antidopaminergique ?
A - Sécheresse de la bouche
B - Eruptions
C - Syndrome extrapyramidal
D - Constipation
Parmi les propositions suivantes concernant la Clonidine, laquelle
est fausse :
A. Est un antihypertenseur d'action centrale
B. Augmente le tonus vagal
C. Peut être proposée dans le traitement des
manifestations de "sevrage" aux opiacés
D. Est contre indiquée dans l'angor
Parmi les propositions suivantes concernant le métoclopramide,
laquelle est inexacte :
A - Est un antiémétique
B - Diminue les contractions antrales
C - Augmente la pression du sphincter inférieur de
l'œsophage
D - Peut être responsable de dyskinésies
Lequel de ces médicaments n’est pas un inducteur enzymatique ?
A - Carbamazépine
B - Rifampicine
C - Phénytoïne
D - Erythromycine
Parmi les effets suivants, lequel n’est pas observé lors du
traitement par un médicament parasympatholytique ?
A - Tachycardie
1
English
Which of the following side effects of chlorpromazine and
phenothiazine neuroleptics is explained by their antidopaminergic
action?
A - Dry mouth
B - Eruptions
C – Extrapyramidal syndrome
D - Constipation
Which of the following statements of clonidine is false?
A – It is a centrally acting antihypertensive
B – It increases vagal tone
C – It can be given in the treatment opioids “withdrawal"
symptoms
D – It is contra- indicated in angina
Which of the following statements of metoclopramide is FALSE
A - Is an antiemetic
B - Reduces antral contractions
C - Increases the pressure of the lower esophageal sphincter
D - May be responsible for dyskinesias
Which of these medications is not an enzyme inducer?
A - Carbamazepine
B - Rifampin
C - Phenytoin
D - Erythromycin
Which of the following effects is not observed during the treatment
with parasympatholytic drugs
A - Tachycardia
B - Mydriase
C - Relaxation du sphincter vésical
D - Relaxation de l'intestin
6. L’ajout de l'adrénaline à une solution de lidocaïne destinée à être
administrée par voie sous-cutanée permet de prolonger la durée
de l'anesthésie locale par :
A - Diminution de la perméabilité de l'endothélium vasculaire
B - Précipitation de la lidocaïne
C - Modification du pH de la solution
D - Vasoconstriction locale
7. Quel est l’effet secondaire qui ne peut être observé lors d’un
traitement par des antidépresseurs tricycliques ?
A - Sécheresse de la bouche
B - Toxicité myocardique
C - Pigmentation cutanée
D - Abaissement du seuil épileptogène
8. Quelle est la propriété que ne possèdent pas les benzodiazépines
?
A - Antidépressive
B - Anticonvulsivante
C - Amnésiante
D - Relaxant musculaire
9. La stimulation des récepteurs bêta-adrénergiques provoque :
A - Un myosis
B - Une bronchoconstriction
C - Un relâchement de l'utérus
D - Une vasoconstriction
10. Le syndrome pseudo-parkinsonien peut être observé après
traitement par le médicament suivant :
A - Acide valproïque
2
B - Mydriasis
C - Relaxation of the bladder sphincter
D - Relaxation of the intestine
Adding adrenaline to a lidocaine solution to be administered by
subcutaneous injection can prolong the duration of local
anesthesia by:
A - Decreased permeability of the vascular endothelium
B - Precipitation of lidocaine
C - Changing the pH of the solution
D - Local Vasoconstriction
What is the side effect that is not observed during the treatment
with tricyclic antidepressants?
A - Dry mouth
B - myocardial toxicity
C - Skin Pigmentation
D - Seizure threshold lowering
What is the property that benzodiazepines do not have?
A - Antidepressant
B - Anticonvulsant
C - Amnesic
D - Muscle Relaxant
The stimulation of beta-adrenergic receptors causes:
A - Miosis
B - Bronchoconstriction
C - Relaxation of the uterus
D - Vasoconstriction
The pseudo-parkinsonian syndrome can be observed after the
treatment with the following medication:
A - Valproic Acid
11.
12.
13.
14.
15.
B - Diazépam
C - Halopéridol
D - Clomipramine
Quel est l’effet secondaire qui ne peut être observé lors d’un
traitement par la Lévodopa?
A - Des dyskinésies
B -Des confusions
C - Des nausées et vomissements
D - Une impuissance
Lequel de ces traitements ne fait pas partie de l’arsenal
thérapeutique de l’acné ?
A - Les rétinoïdes
B - Le péroxyde de benzoyle
C - Les macrolides topiques
D - Le métronidazole
Quelle est la classe thérapeutique qui ne possède pas des
propriétés anticholinergiques ?
A - Antidépresseurs tricycliques
B - Digitaliques
C - Antiparkinsoniens
D - Antispasmodiques digestifs
Quel est le procédé qui ne peut être adopté afin de prévenir la
toxicité rénale des aminoglycosides ?
A- Maintien d'une diurèse importante au moyen du furosémide
B- Surveillance des taux sériques résiduels
C- Adaptation des doses en fonction de la clairance de la
créatinine
D- Correction d'une éventuelle déshydratation
Chez un patient parkinsonien, la trihexyphénidyle :
A- Agit sur les récepteurs de la dopamine
3
B - Diazepam
C - Haloperidol
D - Clomipramine
What is the side effect that can not be observed during the
treatment with Levodopa?
A - Dyskinesias
B - Confusion
C - Nausea and vomiting
D - Impotence
Which of these treatments is not part of the therapeutic arsenal of
acne?
A - Retinoids
B - Benzoyl peroxide
C - Topical macrolide
D - Metronidazole
What is the therapeutic class that does not have anticholinergic
properties?
A - Tricyclic antidepressants
B - Digitalis
C - Antiparkinsonism
D - Digestive antispasmodics
What is the process that may not be adopted to prevent the
nephrotoxicity of aminoglycosides?
A-Maintain diuresis with furosemide
B Monitoring residual serum
C-Adaptation doses according to the creatinine clearance
D-correction of any dehydration
The use of trihexyphenidyl in a Parkinson's patient:
A-Acts on dopamine receptors
16.
17.
18.
19.
B- Agit sur les systèmes cholinergiques
C- Facilite la libération de la dopamine
D- Provoque un myosis
L’acide valproïque possède tous les effets secondaires suivants
sauf :
A- Sédation
B- Thrombocytopénie
C- Hypothyroïdie
D- Hépatotoxicité
Les molécules suivantes peuvent être utilisées comme traitement
thymorégulateur dans les troubles bipolaires sauf:
A- Acide valproique
B- Carbamazépine
C- Lithium
D- Phénytoïne
Quel est le bilan qui ne devrait pas être nécessairement réalisé
avant l’instauration d’un traitement par le Lithium comme
traitement thymorégulateur ?
A- Bilan thyroïdien
B- Bilan hépatique
C- Bilan cardiaque
D- Bilan des électrolytes sanguins (pour vérifier
essentiellement la natrémie)
La majoration de la toxicité des anticoagulants et des sulfamides
hypoglycémiants par voie orale lors de leur association avec un
anti-inflammatoire non stéroïdien est due à :
A- Une inhibition enzymatique
B- Une interaction au niveau des protéines
plasmatiques
C- Une interaction au niveau des transporteurs (P-gp,
4
B-Acts on the cholinergic system
C-Facilitates the release of dopamine
D-Causes miosis
Valproic acid has all the following side effects except:
A-Sedation
B-Thrombocytopenia
C-Hypothyroidism
D-Hepatotoxicity
The following agent can be used as a mood stabilizer in bipolar
disorder treatment except:
A-Valproic acid
B-Carbamazepine
C-Lithium
D-Phenytoin
What is the state that should not necessarily be performed prior to
initiating lithium treatment as mood stabilizers treatment?
A. Thyroid function tests
B. Liver function tests
C. Cardiac function tests
D. Blood electrolytes levels tests (to mainly check serum sodium)
The increase in the toxicity of anticoagulants and oral sulfonylureas
when combined with a non-steroidal anti-inflammatory drug is due
to:
A-Enzyme-inhibition
B- plasma proteins interaction
C-plasma transporters interactions (P-gp, BCRP)
D-Enzyme induction
BCRP)
D- Une induction enzymatique
20. La biodisponibilité d'un médicament administré par voie intra- The bioavailability of a drug administered by direct intravenous
route is:
veineuse directe est de :
A-100%
A- 100%
B-0%
B- 0%
C-Dependent on the half-life of the drug
C- Dépend de la demi-vie du médicament
D-Is unpredictable
D- Est imprévisible
21. Lequel des médicaments anti-infectieux suivants n’est pas
contre-indiqué avec l’alcool ?
A- Métronidazole
B- Kétoconazole
C- Amoxicilline + acide clavulanique
D- Tinidazole
22. Laquelle des molécules suivantes n’est pas recommandée pour
le traitement des infections à Pseudomonas aeruginosa ?
A- Ceftazidime
B- Piperacilline+Tazobactam
C- Cefotaxime
D- Céfépime
23. Laquelle des molécules suivantes n’est pas recommandée pour
le traitement des souches de E. coli porteuses de betalactamases à spectre étendu (BLSE) ?
A- Céfépime
B- Cetriaxone
C- Imipénème
D- Amikacine
24. Laquelle des molécules suivantes n’est pas recommandée pour
le traitement des souches de Staphylococcus Aureus résistantes à
5
Which of the following anti-infective drugs is not contra-indicated
with alcohol?
A-Metronidazole
B-Ketoconazole
C-Amoxicillin + clavulanic acid
D-Tinidazole
Which of the following molecules is not recommended for the
treatment of Pseudomonas aeruginosa infections?
A-Ceftazidime
B-Piperacillin +Tazobactam
C-Cefotaxime
D-Cefepime
Which of the following molecules is not recommended for the
treatment of E. coli beta-lactamase carrying extended spectrum
(ESBL)?
A-Cefepime
B-Cetriaxone
C-Imipenem
D-Amikacin
Which of the following molecules is not recommended in the
treatment of Staphylococcus aureus resistant to methicillin
25.
26.
27.
28.
la méthicilline (SARM) ?
A- Vancomycine
B- Imipénème
C- Teicoplanine
D- Tygécycline
Les antibiotiques suivants sont capables d’induire une
néphrotoxicité même chez les patients possédant une fonction
rénale normale :
A- Les quinolones
B- Les macrolides
C- Les aminoglycosides
D- Les betalactamines
Quel est l’antibiotique qui ne peut être à l’origine
d’allongement du QT avec risque de torsades de Pointe et arrêt
cardiaque ?
A- Azithromycine
B- Amikacine
C- Levofloxacine
D- Clarithromycine
L’héparine est capable d’induire :
A- Une agranulocytose
B- Une insuffisance rénale aigue
C- Une thrombocytopénie
D- Une insuffisance hépatique
Laquelle de ces propositions ne correspond pas à une indication
des antagonistes opiacés (naloxone et naltrexone)?
A- Traitement du sevrage opiacé
B- Traitement du sevrage alcoolique
C- Traitement du surdosage à la morphine
D- Traitement des intoxications à la cocaïne
6
(MRSA)?
A-Vancomycin
B-Imipenem
C-Teicoplanine
D-Tygecycline
The following antibiotics are capable of inducing nephrotoxicity
even in patients with normal renal function:
A-Quinolones
B-Macrolides
C-Aminoglycosides
D-lactams
What is the antibiotic that cannot be the cause of QT prolongation
with risk of torsades de Pointe and cardiac arrest?
A-Azithromycin
B-Amikacin
C-Levofloxacin
D Clarithromycin
Heparin is capable of inducing:
A-Agranulocytosis
B-acute renal failure
C-Thrombocytopenia
D-Liver failure
Which of the following is not an indication of opiate antagonists
(naloxone and naltrexone)?
A-Treatment of opioid withdrawal symptoms
B-Treatment of alcohol withdrawal symptoms
C-Treatment of morphine overdose
D-Treatment of cocaine intoxication
29. Un des médicaments suivants est utilisé sans danger chez les
patients hypertendus présentant une cardiopathie ischémique,
lequel ?
A. Le lisinopril
B. Le minoxidil
C. La phentolamine
D. La diazoxide
30. Un des diurétiques suivants inhibe l'enzyme anhydrase
carbonique dans les cellules épithéliales des tubules proximaux,
lequel ?
A. L’amiloride
B. L’acétazolamide
C. L’hydrochlorothiazide
D. Le furosémide
31. Dans le cas d'une attaque d'angine aiguë, lequel des
médicaments suivants devriez-vous dispenser?
A. L’isosorbide mononitrate
B. L’amlodipine
C. La nitroglycérine
D. Le propranolol
32. En cas d'angor de Prinzmetal, lequel les médicaments suivants ne
peut pas être utilisé?
A. Le propranolol
B. Le diltiazem
C. Le dinitrate d'isosorbide
D. L’amlodipine
7
A drug that is used safely in hypertensive patients with ischemic
heart disease:
A. lisinopril
B. minoxidil
C. phentolamine
D. diazoxide
A diuretic that inhibits carbonic anhydrase enzyme in the proximal
tubular epithelial cells:
A. amiloride
B. acetazolamide
C. hydrochlorothiazide
D. frusemide
In case of an acute anginal attack, you should dispense which of the
following drugs?
A. isosorbide mononitrate
B. amlodipine
C. nitroglycerin
D. propranolol
In case of Prinzmetal's angina, which of the following drugs is NOT
used?:
A. propranolol
B. diltiazem
C. isosorbide dinitrate
D. amlodipine
33. Quel antibiotique produit une crystallurie?
A. Penicilline
B. Sulfamethoxazole
C. Doxycycline
D. Gentamycine
Which antibiotic produces crystalluria?
A. Penicillin
B. Sulfamethoxazole
C. Doxycycline
D. Gentamycin
34. Quelle est la cible d’action des fluoroquinolones :
A. les topoisomerases
B. les ribosomes
C. les PBP membranaires
D. les intégrases
What is the target of action of fluoroquinolones:
A. the topoisomerases
B. ribosomes
C. PBP-membrane
D. integrases
35. Lequel des énoncés suivants n'est
pharmacologique de la digitale?
A. L’effet chronotrope positif
B. L’effet inotrope positif
C. L’augmentation du débit sanguin rénal
D. La diminution de la précharge
36. La zidovudine est un :
A. antiprotease
B. antibacterien
C. inhibiteur de la transcriptase inverse
D. inhibiteur des intégrases
pas
une
action
Which of the following is NOT a pharmacological action of digitalis?
A. positive chronotropic effect
B. positive inotropic effect
C. increase in renal blood flow
D. decrease in preload
Zidovudine is a (n):
A. antiprotease
B. antibacterial
C. reverse transcriptase inhibitor
D. integrase inhibitor
8
.73
Une hypertension chronique chez une femme enceinte peut être
traitée en premier choix avec : A chronic hypertensive pregnant woman may be treated as a first
choice with:
A. Le candésartan
A. candesartan
B. Le fosinopril
B. fosinopril
C. La méthyldopa
C. methyldopa
D. L’hydralazine
D. hydralazine
38. La progestérone :
Progesterone:
A. agit sur des récepteurs membranaires
A. acts upon membrane receptors
B. n’est jamais utilisée seule sans estrogène
B. is never used alone without estrogen
C. augmente le risque de cancer de l’endomètre
C. increases the risk of endometrial cancer
D. domine la phase lutéale
D. dominates the luteal phase
39. Les pilules contraceptives triphasiques :
A. contiennent 3 molécules actives
B. libèrent les principes actifs en 3 phases
C. contiennent des concentrations différentes d’hormones
selon le temps du cycle
D. sont destinées aux femmes qui fument
.04
Un des médicaments suivants n'est pas utilisé comme traitement
de premier choix pour l'hypertriglycéridémie:
A. Le clofibrate
B. La lovastatine
C. Le niaspan
D. La cholestyramine
9
triphasic contraceptive pills:
A. contain 3 active molecules
B. release the active ingredients in 3 phases
C. contain different hormone concentrations according to the
time cycle
D. are intended for women who smoke
A drug that is NOT used as a first line therapy for
hypertriglyceridemia is:
A. clofibrate
B. lovastatin
C. niaspan
D. cholestyramine
41. Lequel de ces médicaments provoque une hypertrophie de la
gencive :
A. la phenytoine
B. l’ibuprofen
C. le tramadol
D. le captopril
Which of these drugs causes enlargement of the gum:
A. phenytoin
B. ibuprofen
C. tramadol
D. captopril
42. L’ethinylestradiol est :
Ethinylestradiol is:
A. une molécule naturelle endogène
A. an endogenous natural molecule
B. est sécrétée par l’embryon
B. is secreted by the embryo
C. est métabolisée par le foie
C. is metabolized by the liver
D. se trouve dans les pilules contraceptives à la
D. is available in contraceptive pills of 35- 50 mg
concentration de 35- 50 mg
concentration
43. Le losartan est un :
A. béta bloquant
B. IEC
C. antagoniste du récepteur de l’angiotensine II
D. diurétique hypokaliémiant
Losartan is a (n):
A. beta-blocker
B. ACEI
C. angiotensin II receptor antagonist
D. hypokalaemic diuretic-
44. L’aténolol est :
A. un IEC
B. un béta bloquant non sélectif
C. un inhibiteur calcique
D. plus hydrophile que le propranolol
Atenolol is:
A. An ACEI
B. A nonselective beta-blocker
C. A calcium channel blocker
D. More hydrophilic than propranolol
10
.04
.04
Le dysfonctionnement érectile peut être dû à l’un des
médicaments suivants:
A. Le sildénafil
B. L’alprostadil
C. La cimétidine
D. Le goséréline
46. Concernant les inhibiteurs calciques :
A. le verapamil n’est pas antiarythmique
B. la nifedipine est antiarythmique
C. l’amlodipine a un tropisme vasculaire
D. le diltiazem est une dihydropyridine
Erectile dysfunction may be caused by:
A. sildenafil
B. alprostadil
C. cimetidine
D. goserelin
Regarding calcium channel blockers:
A. Verapamil is not an antiarrhythmic
B. Nifedipine is an antiarrhythmic
C. amlodipine has a vascular tropism
D. Diltiazem is a dihydropyridine
47. Concernant les antidiabétiques :
A. les biguanides sont moins efficaces que les sulfamides
B. les sulfamides augmentent la sécrétion d’insuline
C. les biguanides augmentent la sécrétion d’insuline
D. les glitazones inhibent les alpha-glucosidases
Regarding anti-diabetic agents:
A. Biguanides are less effective than sulfonamides
B. Sulfonamides increase insulin secretion
C. Biguanides increase insulin secretion
D. Glitazones inhibit alpha-glucosidase
Lequel des médicaments suivants est le traitement de choix pour
l'amibiase?
A. Le métronidazole
B. Le mébendazole
C. L’iodoquinol
D. La chloroquine
49. Quelle molécule peut provoquer une acidose lactique :
A. le glibenclamide
B. la pioglitazone
C. le repaglinide
D. la metformine
Which of the following is the drug of choice for treatment of
amebiasis?
A. metronidazole
B. mebendazole
C. iodoquinol
D. chloroquine
Which molecule can cause lactic acidosis:
A. Glibenclamide
B. Pioglitazone
C. Repaglinide
D. Metformin
11
.44
Parmi les maladies virales suivantes laquelle n’a pas de vaccins
disponibles?
A. La gastro-entérite
B. Le rhume
C. La grippe
D. L’hépatite A
51. Concernant la metformine :
A. elle est fortement hypoglycémiante
B. elle stimule le récepteur SUR1
C. elle provoque un gain de poids
D. elle stimule l’utilisation du glucose par les cellules
.45
Which of the following viral diseases has no available vaccines?
A. gastroenteritis
B. common cold
C. influenza
D. hepatitis A
Regarding metformin:
A. It is strongly hypoglycemic
B. It stimulates SUR1 receptor
C. It causes weight gain
D. It stimulates glucose utilization by the cells
La Candidose chronique disséminée chez les patients stables doit
être principalement traitée par:
A. La griséofulvine
B. La caspofungine
C. Le fluconazole
D. L’amphotéricine B
53. La levodopa :
A. est un anticholinergique
B. un an antidépresseur central
C. est un antiparkinsonien
D. est transformée en acétylcholine
Chronic disseminated candidiasis in stable patients should be
primarily treated by:
A. griseofulvin
B. caspofungin
C. fluconazole
D. amphotericin B
Levodopa:
A. Is an anticholinergic
B. Is a centrally-acting antidepressant
C. Is an antiparkinsonian
D. Is transformed into acetylcholine
12
.40
.45
Lequel des médicaments suivants est un antagoniste des
récepteurs des leucotriènes:
A. La Zileuton
B. Le Cromolyn
C. L’ipratropium
D. Le Montelukast
55. Le phenobarbital :
A. a une demi-vie courte
B. est uniquement adminstré en IV
C. est utilisé dans les crises épileptiques néonatales
D. est une prodrogue
Which of the following is a Leukotriene receptor antagonist:
A. Zileutin
B. Cromolyn
C. Ipratropium
D. Montelukast
Phenobarbital:
A. Has a short half-life
B. Is only administered IV
C. Is used in neonatal seizures
D. Is a prodrug
Indiquer le médicament appartenant aux inhibiteurs de la pompe
à protons:
A. La Pirenzépine
B. La ranitidine
C. L’oméprazole
D. Le bicarbonate de sodium
57. La BNP s’élève significativement :
A. dans la crise hypertensive
B. dans l’endocardite
C. dans l’angor stable
D. dans l’insuffisance cardiaque
13
Indicate the drug belonging to proton pump inhibitors:
A. Pirenzepine
B. Ranitidine
C. Omeprazole
D. Sodium bicarbonate
BNP rises significantly:
A. In hypertensive crisis
B. In endocarditis
C. In stable angina
D. In heart failure
.44
.54
.56
A propos des hypoglycémiants oraux, lequel des énoncés
Which of the following statements is true for oral hypoglycemics?
suivants est vrai ?
A. Meglitinide is ineffective in regulating postprandial glucose
A. la méglitinide est inefficace dans la régulation de la glycémie
B. Metformin is suitable for obese patients
postprandiale
C. Pioglitazone is harmless to the liver
B. La metformine est appropriée pour les patients obèses
D. Acarbose can be used for diabetic patients suffering from
C. La pioglitazone est sans danger pour le foie
gastrointestinal tract disturbances and flatulence
D. L'acarbose peut être utilisé chez des patients diabétiques
souffrant de troubles du tractus gastrointestinal et de flatulence
59. L’enzyme qui s’élève en premier parmi ces propositions, dans Which enzyme level is first elevated in myocardial infarction:
l’infarctus du myocarde est:
A. Troponin-T
A. la troponine T
B. BNP
B. la BNP
C. LDH
C. la LDH
D. CK-BB
D. la CPK-BB
Indiquez l'antidote spécifique pour les benzodiazépines:
A. Le flumazénil
B. La Buspirone
C. Le Zolpidem
D. Le triazolam
Indicate the specific antidote for benzodiazepines:
A. Flumazenil
B. Buspirone
C. Zolpidem
D. Triazolam
Lequel des agents suivants est un inhibiteur sélectif de recapture
de la sérotonine (ISRS)?
A. Le Lithium
B. L’imipramine
C. La fluoxétine
D. La venlafaxine
Which of the following agents is a selective serotonin reuptake
inhibitor (SSRI)?
A. Lithium
B. Imipramine
C. Fluoxetine
D. Venlafaxine
14
.55
.57
Indiquer le médicament appartenant aux antiacides:
A. La Pirenzépine
B. La ranitidine
C. L’oméprazole
D. Le Bicarbonate de sodium
Indicate the drug belonging to antacids:
A. Pirenzepine
B. Ranitidine
C. Omeprazole
D. Sodium bicarbonate
Lequel des médicaments suivants ne fait pas partie de la classe
des mucoprotecteurs?
A. Le Misoprostol
B. La sous-salicylate de bismuth
C. Le Sucralfate
D. La dompéridone
Which of the following drugs does not belong to the class of
mucosal protective agents?
A. Misoprostol
B. Bisthmus subsalicylate
C. Sucralfate
D. Domperidone
64. Le stade NYHA III de l’insuffisance cardiaque se caractérise par:
A. symptômes au moindre effort
B. pas de symptômes
C. douleur thoracique
D. sueurs nocturnes
.54
Indiquer un inhibiteur de la dopa décarboxylase périphérique:
A. L'entacapone
B. La clozapine
C. La Carbidopa
D. La sélégiline
NYHA stage III in heart failure is characterized by:
A. Symptoms on mild effort
B. No symptoms
C. Chest Pain
D. Night-sweats
Indicate a peripheral dopa decarboxylase inhibitor:
A. Entacapone
B. Clozapine
C. Carbidopa
D. Selegiline
15
66. Les pyélonéphrites :
A. sont toujours asymptomatiques
B. sont une infection de la vessie
C. sont accompagnées de fièvre et de frissons
D. sont des infections virales et non bactériennes
Pyelonephritis.:
A. is always asymptomatic
B. is a bladder infection
C. is accompanied by fever and chills
D. is a viral and non-bacterial infection
67. Les dérivés nitrés sont contre-indiqués lorsque :
A. la pression artérielle systolique est < 90 mmHg
B. la pression artérielle diastolique est > à 140 mmHg
C. La personne est diabétique
D. La personne est dylipidémique
Nitro derivatives are contra-indicated when:
A. Systolic blood pressure <90 mmHg
B. Diastolic blood pressure is> 140 mmHg
C. The person is diabetic
D. The person is dyslipidemic
.54
Lequel des antiparkinsoniens suivants est utilisé pour traiter
l'hyperprolactinémie?
A. la Benzatropine
B. La bromocriptine
C. L’amantadine
D. La lévodopa
69.
Parmi ces molecules,
glycoproteines gpIIb/IIIa :
A. le tirofiban
B. le prasugrel
C. le reteplase
D. la streptokinase
laquelle
est
un
inhibiteur
Which of the following antiparkinson drugs is used to treat
hyperprolactinemia?
A. Benzatropine
B. Bromocriptine
C. Amantadine
D. Levodopa
des
16
Among these molecules, which is an inhibitor of glycoprotein
GPIIb/IIIa:
A. Tirofiban
B. Prasugrel
C. Reteplase
D. Streptokinase
.34
Le Tamoxifène est classé comme:
A. un antiprogestatif
B. un antiandrogène
C. un anti-oestrogène
D. un androgène
Tamoxifen is classified as:
A. Antiprogestin
B. Antiandrogen
C. Antiestrogen
D. Androgen
71. Un homme de19 ans se présente en urgence avec une méningite
bactérienne. Laquelle des céphalosporines suivantes la plus
adaptée à ce cas?
A.
B.
C.
D.
La Céfazoline
Le Céfuroxime axétil
La Ceftriaxone
La Céfopérazone
a.
b.
c.
d.
72. Le suivi de l’efficacité d’un traitement à l’héparine non
fractionnée se fait par :
A. l’INR
B. le taux de plaquettes
C. le temps de saignement
D. le PTT
.37
A 19-year-old male presents to the emergency room with bacterial
meningitis.Which of the following Cephalosporin is likely to be
given to this patient?
Une des molécules suivantes est indiquée dans le traitement du
trouble du déficit de l'attention avec hyperactivité (THADA):
A. Le méthylphénidate
B. L’olanzépine
C. La phénytoïne
D. L’Amitryptiline
17
Cefazolin
Cefuroxime axetil
Ceftriaxone
Cefoperazone
Monitoring the efficacy of a treatment with unfractionated
heparin is by:
A. INR
B. Platelet
C. Bleeding time
D. PTT
The following is indicated for Attention-Deficit Hyperactivity
Disorder (ADHD):
A. Methylphenidate
B. Olanzepine
C. Phenytoin
D. Amitryptiline
74. Laquelle de ces molécules est un inhibiteur direct de la
thrombine :
A. hirudine
B. eptifibatide
C. amiodarone
D. furosemide
75. Un homme de 37 ans, toxicomane par voie intraveineuse est
admis pour fièvre et essoufflement. Les prélèvements
d’hémocultures multiples démontrent la présence de S. aureus
résistant à la méthicilline. D'autres résultats sont compatibles
avec l'endocardite. Lequel des antibiotiques suivants est le plus
approprié dans ce cas?
A.
B.
C.
D.
La Ceftriaxone
L’Imipénème
La Céfazoline
La Daptomycine
76. Le suivi d’un traitement par anticoagulant oral anti-vitamine K se
fait par :
A. la vitamine K
B. l’INR
C. le PTT
D. le dosage des plaquettes
18
Which of these molecules is a direct thrombin inhibitor:
A. Hirudin
B. Eptifibatide
C. Amiodarone
D. Furosemide
A 37-year-old intravenous drug abuser isadmitted for fever and
shortness of breath. Multipleblood cultures drawn demonstrate S.
aureus resistant to methicillin. Other findings were consistent with
endocarditis.Which of the following is an appropriateantibiotic in
this case?
a.
b.
c.
d.
Ceftriaxone
Imipenem
Cefazolin
Daptomycin
Monitoring of the oral anticoagulant anti-vitamin K treatment
is by:
A. Vitamin K
B. INR
C. PTT
D. Platelet assay
.33
Concernant le valproate, lequel des énoncés suivants est
incorrect?
A. Il est utilisé pour toute forme d'épilepsie
B. Il cause comme effets secondaires une thrombopénie et une
hépatotoxicité
C. Il peut être utilisé sans danger pendant la grossesse
D. Il peut être utilisé dans la prophylaxie et le traitement
d'épisodes maniaques chez des patients avec un trouble bipolaire
et qui ne répondent pas au traitement par le lithium
78. Quel est le mécanisme d'action de la ciprofloxacine?
a. L'inhibition du ribosome 30s
b. L’inhibition du ribosome 50s
c. L'inhibition de la synthèse de l'ARN
d. L'inhibition de l'ADN gyrase
Concerning valproate, which of the following is incorrect?
A. It is used for all forms of epilepsy
B. It causes thrombocytopenia and hepatotoxicity as side
effects
C. It can be used safely during pregnancy
D. It can be used for the prophylaxis and treatment of manic
episodes in patients with bipolar disorder unresponsive to
lithium
What is the mechanism of action of ciprofloxacin?
a.
b.
c.
d.
Inhibition of the 30s ribosome
Inhibition of the 50s ribosome
Inhibition of RNA synthesis
Inhibition of DNA gyrase
79. En cas de crise d’angor stable, laquelle de ces molécules est In stable angina crisis, which of these is used:
utilisée :
A. Nitroglycerin patch
A. trinitrine patch
B. Subcutaneous isosorbide dinitrate
B. dinitrate d’isosorbide en sous cutanné
C. Calcium channel blocker patch
C. inhibiteur calcique en patch
D. Sublingual nitroglycerin
D. nitroglycérine sublinguale
.44
Lequel est le médicament de choix pour une femme de 28 ans
avec un trouble affectif schizoïde et une difficulté
d’endormissement?
A. L’aripiprazole
B. La chlorpromazine
C. L’halopéridol
D. La rispéridone
Which of the following is the drug of choice for a 28-year-old
woman with schizoid affective disorder and sleeping difficulty?
A. Aripiprazole
B. Chlorpromazine
C. Haloperidol
D. Risperidone
19
81. L'antibiotique bactéricide qui interfère avec la synthèse de la
paroi cellulaire bactérienne et qui peut provoquer le "syndrome
de l'homme rouge" s’il est injecté trop rapidement est :
A.
B.
C.
D.
.45
La Pipéracilline
La Vancomycine
La Gentamicine
La Clindamycine
The bactericidal antibiotic that interferes with bacterial cell-wall
synthesis and that may cause “red mansyndrome” if injected too
rapidly is
a.
b.
c.
d.
La Prégabaline n'est pas indiquée dans :
A. Les saisies de commencement partielles
B. La douleur neuropathique
C. La fibromyalgie
D. La dépression
Piperacillin
Vancomycin
Gentamicin
Clindamycin
Pregabalin is not indicated for:
A. Partial onset seizure
B. Neuropathic pain
C. Fibromyalgia
D. Depression
83. Lequel est un effet secondaire des derivés nitrés ?:
A. Methémoglobinemie
B. Pic hypertensif
C. Hyperglycémie
D. Baisse des taux d’HDL
Which is a side effect of nitrates:
A. Methaemoglobinaemia
B. Hypertensive crisis
C. Hyperglycemia
D. Decrease in HDL
20
84. L'antibiotique qui inhibe de manière irréversible la synthèse de
protéines bactériennes et dont les effets indésirables peuvent
inclure l’ototoxicité et la néphrotoxicité est:
A.
B.
C.
D.
.44
La Doxycycline
La Clarithromycine
La Gentamicine
La Clindamycine
The antibiotic that irreversibly inhibits bacterial protein synthesis
and whose adverse effects may includeototoxicity and
nephrotoxicity is:
a.
b.
c.
d.
Doxycycline
Clarithromycin
Gentamicin
Clindamycin
Lequel des médicaments suivants n'est pas recommandé pour le
mal des transports?
A. La scopolamine transdermique
B. L'aprépitant
C. Le Meclizine
D. La Diphenhydramine
Which of the following is NOT recommended for motion sickness?
A. Scopolamine transdermal patch
B. Aprepitant
C. Meclizine
D. Diphenhydramine
86. L'antibiotique dont les effets indésirables comprennent les
brûlures d'estomac, la photosensibilité, et la dépression possible
de la croissance osseuse chez les jeunes enfants est:
The antibiotic whose adverse effects include heartburn,
photosensitivity, and possible depression of bonegrowth in young
children is:
A.
B.
C.
D.
L’Érythromycine
La Tétracycline
La Gentamicine
La Clindamycine
a.
b.
c.
d.
21
Erythromycin
Tetracycline
Gentamicin
Clindamycin
.43
DP est un homme de 72 ans souffrant d'un ulcère induit par les
AINS et à H. pylori négatif. Il doit continuer son traitement à
l'AINS pour l'arthrose. Quel est le médicament recommandé pour
la guérison de l'ulcère et la prévention des ulcères à venir?
A. Le Sucralfate
B. La Nizatidine
C. L’Hydroxyde d'aluminium
D. Le Pantoprazole
88. Quel médicament antifongique est associé à son mécanisme
exact d'action?
A. Les Échinocandines inhibent la synthèse de polysaccharide
(glucane) dans la paroi de la cellule
B. La Flucytosine se lie à la membrane cellulaire,
C. Le fluconazole est un antimétabolite de l'ARN fongique
D. L'amphotéricine B se lie à une enzyme dépendant du
cytochrome P450-
.48
Lesquels parmi les médicaments suivants affectant la fonction du
tractus GI et ne provoquent pas de constipation?
A. Les anticholinergiques
B. Les Antiacides de magnésium
C. Les opiacés
D. Les antiacides aluminiums
90. Un rapport urée sanguine/créatinine sanguine élevé indique:
A. une insuffisance hépatique
B. une insuffisance prérénale
C. un calcul au niveau des voies urinaires
D. un calcul au niveau des voies biliaires
DP is a 72 year old man with a documented NSAID-induced ulcer
and is H.pylori negative. He must continue the NSAID for
osteoarthritis. Which of the following is the recommended
medication for ulcer healing and prevention of future ulcers?
A. Sucralfate
B. Nizatidine
C. Aluminum hydroxide
D. Pantoprazole
Which antifungal drug is matched with its correct mechanism of
action?
a. Echinocandins inhibit polysaccharide (glucan) synthesis in
the cell wall
b. Flucytosine binds to the cell membrane
c. Fluconazole is an antimetabolite for fungal RNA
d. Amphotericin B binds to a cytochrome P450-dependent
enzyme
Which of the following drugs affecting gastrointestinal function
does not cause constipation?
A. Anticholinergics
B. Magnesium antacids
C. Opiates
D. Aluminum antacids
A high BUN / blood creatinine ratio indicates:
A. Hepatic impairment
B. Prerenal failure
C. A calculus in the urinary tract
A calculus in the biliary tract
22
91. Un homme de 22 ans sexuellement actif se présente à son
médecin de soins primaires avec une miction douloureuse et un
écoulement urétral. La coloration de Gram du liquide de
décharge montre la présence de diplocoques Gram négatif. De la
Ceftriaxone lui a été préscrite pour l'infection gonococcique. Quel
autre médicament doit-il être ajouté?
A.
B.
C.
D.
.85
L'aztréonam
La Doxycycline
L’Imipénème / la cilastatine
La Nitrofurantoïne
A 22-year-old sexually active male presents to his primary care
physician with painful urination and urethral discharge. Gram stain
of discharge fluid shows gram-negative diplococci. He is given
Ceftriaxone for gonococcal infection. What additional medication
should he be given?
a. Aztreonam
b. Doxycycline
c. Imipenem/cilastatin
d. Nitrofurantoin
Les produits tels que le psyllium, la méthylcellulose, et le
polycarbophile sont connus en tant que:
A. Laxatifs stimulants
B. Agents mucilagineux
C. Cathartiques
D. Lubrifiants
Products such as psyllium, methylcellulose, and polycarbophil are
known as:
A. Stimulant laxatives
B. Bulk-forming agents
C. Cathartics
D. Lubricants
93. Les benzodiazépines agissent au niveau d’un site spécifique du
récepteur :
A. GABAergique
B. SérotoninErgique
C. Dopaminergique
D. Muscarinique
23
Benzodiazepines act at a specific receptor site:
A. GABAergic
B. Serotoninergic
C. Dopaminergic
D. Muscarinic
94. Quel est le mécanisme d'action de l'amphotéricine B?
A.
B.
C.
D.
What is the mechanism of action of amphotericin B?
Se lie à l’ergostérol
Inhibe la synthèse de la paroi cellulaire
Inhibe la synthèse de l'ADN
d. Inhibe la squalène époxydase
a. Binds ergosterol
b. Inhibits cell wall synthesis
c. Inhibits DNA synthesis
d. Inhibits squalene epoxidase
95. L'action des benzodiazépines dépend de leur interaction avec des
récepteurs qui suivent?
A.
B.
C.
D.
Le GABA
La Sérotonine
la Glutamate NMDA
La Dopamine
The benzodiazepines’ action depends on their interaction with
which of the following receptors?
a. GABA
b. Serotonin
c. NMDA-glutamate
d. Dopamine
.85
TG est une femme de 43 ans qui se présente avec une histoire de
2 semaines de douleurs épigastriques et un anticorps sérique
positif pour H. pylori. Lequel des énoncés suivants serait-il
considéré comme le traitement initial préféré pour H. pylori?
A. IPP + métronidazole + lévofloxacine
B. IPP + amoxicilline + lévofloxacine
C. IPP + amoxicilline + clarithromycine
D. IPP + métronidazole + bismuth + tétracycline
24
TG is a 43 year old woman who presents with a 2 week history of
epigastric pain and a positive serum antibody for H. pylori. Which
of the following would be considered the preferred initial therapy
for H. pylori?
A. PPI+metronidazole+levofloxacin
B. PPI+amoxicillin+levofloxacin
C. PPI+amoxicillin+clarithromycin
D. PPI+metronidazole+bismuth+tetracycline
97. Le métronidazole agit par ?
A.
B.
C.
D.
Which of the following is involved in metronidazole’s action?
Bloquage de la synthèse de l'acide folique
Perturbation de l'ADN
Inhibition de ribosomes
Inhibition de la topoisomérase
a. Blocking folic acid synthesis
b. Disruption of DNA
c. Inhibition of ribosomes
d.
98. La douleur neuropathique répond bien :
A. aux opiaces
B. aux AINS
C. au paracétamol
D. aux anti-épileptiques
Neuropathic pain responds well to:
A. Opiates
B. NSAIDs
C. Paracetamol
D. Antiepileptics
99. Un agent anti- parkinsonien agissant directement sur le
récepteur de la dopamine est le suivant:
A.
B.
C.
D.
Inhibition of topoisomerase
An anti parkinsonian agent which acts directly on the dopamine
receptor is:
La Lévodopa
L’Amantadine
La Sélégiline
d. La Bromocriptine
a. Levodopa
b. Amantadine
c. Selegiline
d. Bromocriptine
.644
Les effets indésirables locaux de corticostéroïdes inhalés
comprennent:
A. Une Candidose buccale
B. Une hypertension
C. Un retard de croissance
D. Une hyperglycémie
Local adverse effects of inhaled corticosteroids include:
A. Oral candidiasis
B. Hypertension
C. Growth retardation
D. Hyperglycemia
25
101. Les récepteurs opiacés impliqués le plus dans la douleur sont :
A. les récepteurs mu
B. les récepteurs kappa
C. les récepteurs Y
D. les récepteurs oméga-3
Opioid receptors mostly involved in pain are:
A. Mu receptors
B. Kappa receptors
C. Y receptors
D. Omega-3 receptors
102. Une femme de 23 ans ayant subi une appendicectomie se
présente par la suite avec une infection de la plaie. L’examen de
la culture montre un staphylocoque doré résistant à la
méthicilline (SARM). Lequel des antibiotiques suivants serait utile
dans le traitement de cette infection?
A 23-year-old woman who underwent an open appendectomy is
later found to have a wound infection. A culture grows methicillinresistant Staphylococcus aureus (MRSA). Which of the following
antibiotics would be useful in treating this infection?
A.
B.
C.
D.
La Ceftriaxone
La Dicloxacilline
La Vancomycine
La Nafcilline
a. Ceftriaxone
b. Dicloxacillin
c. Vancomycin
d. Nafcillin
103. Lequel parmi les médicaments suivants serait préféré pour le
contrôle de la réponse ventriculaire chez les patients souffrants
de fibrillation auriculaire et d’insuffisance cardiaque?
A.
B.
C.
D.
Le Diltiazem
Le Vérapamil
la Digoxine
L’Ibutelide
Which of the following medications would be preferred for control
of ventricular response in patients with atrial fibrillation and heart
failure?
a. Diltiazem
b. Verapamil
c. Digoxin
d. Ibutelide
26
104. Un homme de 78 ans devait subir une extraction de ses dents. Il
souffre d’une sténose mitrale avec une insuffisance cardiaque
légère. Parmi les médicaments suivants, lequel serait le plus
approprié pour la prophylaxie avant son intervention dentaire?
A.
B.
C.
D.
L’Amoxicilline
Le Co-trimoxazole
L’Imipénème
La Vancomycine
A 78-year-old male is to have his teeth extracted. He also has a
history of mitral valve stenosis with mild cardiac insufficiency.
Which of the following medications would be the most appropriate
for prophylaxis prior to his dental procedure?
a. Amoxicillin
b. Co-trimoxazole
c. Imipenem
d.
105. Le méthylphénidate est utilisé dans la prise en charge de :
A. La schizophrénie
B. La dépression réfractaire
C. Les troubles d’hyperactivité et déficit d’attention
(THADA)
D. L’autisme
.645
Afin de réduire le risque de thrombo-embolie, une patiente sous
contraceptif oral doit prendre:
A. Un produit de combinaison avec une dose plus élevée
d'oestrogène
B. Un produit de combinaison avec une dose plus élevée de
progestatif
C. Un produit de combinaison avec une dose inférieure
d'oestrogène
D. Un produit de combinaison avec une dose inférieure de
progestatif
27
Vancomycin
Methylphenidate is used in the treatment of:
A. Schizophrenia
B. Refractory depression
C. Attention deficit hyperactivity disorder (ADHD)
D. Autism
In order to reduce the risk of thromboembolism, a patient on oral
contraceptive should be prescribed:
A. A combination product with a higher estrogen dose
B. A combination product with a higher progestin dose
C. A combination product with a lower estrogen dose
D. A combination product with a lower progestin dose
107. La bromocriptine a obtenu une nouvelle indication dans :
A. la migraine
B. le diabète type II
C. l’insuffisance cardiaque
D. l’anxiété
.644
Bromocriptine has been recently approved for a new
indication:
A. Migraine
B. Diabetes type II
C. Heart failure
D. Anxiety
Une femme qui prenait un contraceptif oral (œstrogène et
progestatif) pendant plusieurs années est diagnostiquée
d'épilepsie et a commencé la phénytoïne. Quelle est la
conséquence la plus probable de l'ajout de la phénytoïne?
A. Des cas d'agranulocytose ou d’anémie aplasique, exigeant
l'arrêt des deux médicaments immédiatement
B. Des saisies révolutionnaires dues à l’augmentation de la
clairance de la phénytoïne
C Une réduction de l'efficacité du contraceptif
D. Une thromboembolie due à l’oestrogène dans le contraceptif
109. Le mécanisme d'action antibactérienne de la tétracycline
implique
A. La fixation à un composant de la sous-unité ribosomale
50S
B. L'inhibition de l'activité de la translocase
C. Le Blocage de la liaison de l'aminoacyl - ARNt aux
ribosomes bactériens
D. L'inhibition sélective de la peptidyl transférase
ribosomale
28
A woman who has been taking an oral contraceptive (estrogen plus
progestin) for several years is diagnosed with epilepsy and started
phenytoin. What is the most likely consequence of adding
phenytoin?
A. Agranulocytosis or aplastic anemia, requiring stopping both
drugs immediately
B. Breakthrough seizures from increased phenytoin clearance
C. Reduced contraceptive efficacy
D. Thromboembolism from the estrogen component of the
contraceptive
The mechanism of antibacterial action of tetracycline involves
a. Binding to a component of the 50S ribosomalsubunit
b. Inhibition of translocase activity
c. Blockade of binding of aminoacyl – tRNA tobacterial
ribosomes
d. Selective inhibition of ribosomal peptidyltransferases
110. Une thérapeutique préférée dans le cancer du sein est:
A.
B.
C.
D.
A preferred therapeutic regimen in breast cancer is:
Cyclophosphamide + Doxorubicine suivi de paclitaxel
Cisplatine + Etoposide
Melphalan + Prednisone
Cyclophosphamide + Dexaméthasone + Thalidomide
a. Cyclophosphamide + doxorubicin followed by paclitaxel
b. Cisplatin + etoposide
c. Melphalan + prednisone
d. Cyclophosphamide + dexamethasone+ thalidomide
111. Lequel des médicaments suivants, administrés par voie orale, est
le plus susceptible d'être efficace dans le traitement de la colite
due à C. difficile?
A.
B.
C.
D.
.665
L’Ampicilline
La Céfazoline
la Clindamycine
d. Le métronidazole
A propos de la metformine, laquelle des propositions suivantes
est correcte?
A. La metformine peut être utilisée chez les patients atteints de
maladie rénale chronique
B. La metformine peut causer de la diarrhée quand le
médicament est initialement utilisé
C. L’Hypoglycémie induite par la metformine doit être traitée
avec du glucose
D. La metformine est utilisée sans danger dans l'insuffisance
cardiaque décompensée
29
Which of the following drugs, administered orally, is most likely to
be effective in the treatment of colitis due to C difficile?
a.
b.
c.
d.
Ampicillin
Cefazolin
Clindamycin
Metronidazole
Which of the following statements is correct about metformin?
A. Metformin can be used in patients with chronic kidney
disease
B. Metformin may cause diarrhea when it is initially taken
C. Metformin-induced hypoglycemia should be treated with
glucose
D. metformin is safely used in decompensated heart failure
113. Parmi les suivants, quel est l'effet secondaire fréquent qui suit la
cytarabine à haute dose?
A.
B.
C.
D.
L’Otite
La Néphrotoxicité
La Constipation
d. La Conjonctivite
Which of the following is a common side effect of high dose
cytarabine?
a. Otitis
b. Nephrotoxicity
c. Constipation
d. Conjunctivitis
.660
.664
Lequel des hypoglycémiants oraux stimule la sécrétion d'insuline
et provoque une hypoglycémie?
A. Les sulfonylurées
B. Les biguanides
C. Les inhibiteurs DPP-4
D. Les thiazolidinediones
Lequel des médicaments suivants peut être utilisé pour soulager
rapidement les tremblements, les palpitations, l'anxiété et
l’intolérance à la chaleur observée chez les patients souffrant
d'hyperthyroïdie?
A. L'iode radioactif
B. La Propylthiouracil
C. La Methimazole
D. Le Propranolol
30
Which of the following oral glucose lowering agents stimulate
insulin secretion and cause hypoglycemia?
A. Sulfonylureas
B. Biguanides
C. DPP-4 inhibitors
D. Thiazolidinediones
Which of the following drugs may be used to quickly relieve
tremor, palpitations, anxiety, and heat intolerance seen in patients
with hyperthyroidism?
A. Radioactive iodine
B. Propylthiouracil
C. Methimazole
D. Propranolol
116. Un homme de 19 ans asthmatique se présente à votre pharmacie
avec la candidose oropharyngée (OPC). Lequel des médicaments
suivants est responsable pour l’OPC?
A.
B.
C.
D.
L’Albuterol
Le Budésonide
L’Ipratropium
La Théophylline
A 19-year-old male with asthma presents to your pharmacy with
oropharyngeal candidiasis (OPC). Which of the following
medications is responsible for OPC?
a. Albuterol
b. Budesonide
c. Ipratropium
d. Theophylline
117. Une patiente est traitée par la doxycycline dans la tentative de
traiter l'acné Vulgaris ; elle doit suivre tous les conseils suivants
sauf un, lequel ?
A. La consommation excessive de ce médicament peut vous
exposer à un danger inutile et peut rendre votre flore de
la peau résistant au traitement
B. Arrêtez la doxycycline si une grossesse est à prévoir
C. La dose habituelle est 50-100 mg deux fois par jour
D. Espacez la doxycyline de la nourriture pour améliorer son
absorption
.664
Laquelle des préparations suivantes d'insuline a un aspect
trouble?
A. Le detemir
B. Le régulier
C. La glargine
D. Le NPH
A female patient taking doxycyline in attempt to treat acne vulgaris
should be given all of the following advices EXCEPT:
a. Excess intake of this drug can expose you to unnecessary
harm and can render your skin flora resistant to treatment
b. Stop doxycycline in case pregnancy is anticipated
c. The usual dose is 50-100 mg twice per day
d. Space doxycyline from food to enhance its absorption
Which of the following insulin preparations has a cloudy
appearance?
A. Detemir
B. Regular
C. Glargine
D. NPH
31
119. Lequel des antiflatulents suivants est d'habitude ajouté à
l’antiacide?
A.
B.
C.
D.
Which of the following antiflatulents is usually added to antacid?
a. Alginic acid
L'acide alginique
Le Sucralfate
Le Simethicone
La Misoprostol
b. Sucralfate
c. Simethicone
d. Misoprostol
120. Le traitement préféré de l'érythème fessier banal chez un
enfant se fait avec :
A.
B.
C.
D.
.656
The preferred treatment option for an infant with mild diaper
dermatitis is:
La crème d’acyclovir
La pommade de nystatine
La Crème de clotrimazole
La pâte d'oxyde de zinc
a.
b.
c.
d.
L'effet secondaire le plus indésirable de la Methimazole est:
A. L’hépatotoxicité
B. L’agranulocytose
C. L’anaphylaxie
D. La vision floue
Acyclovir cream
Nystatin ointment
Clotrimazole cream
Zinc oxide paste
The most feared adverse effect of Methimazole is:
A. Hepatotoxicity
B. Agranulocytosis
C. Anaphylaxis
D. Blurred vision
32
122. Lequel des médicaments suivants possède une activité
analgésique et antipyrétique sans action anti-inflammatoire?
A.
B.
C.
D.
Which of the following is an analgesic and antipyretic drug that
lacks an anti-inflammatory action?
L'acétaminophène
Le célécoxib
La Colchicine
L’Indométacine
a. Acetaminophen
b. Celecoxib
c. Colchicine
d. Indomethacin
123. Lequel des énoncés suivants correspond à la dose correcte sans
ordonnance de lopéramide?
A.
B.
C.
D.
2mg chaque 6 heures
2 mg d'abord, puis 1 mg prn
2 mg d'abord, puis 1 mg après chaque selles molles.
4 mg d'abord, puis 2 mg après chaque selles molles
Which of the following is the correct non-prescription dose of
loperamide?
a. 2mg q 6 hrs
b. 2 mg initially, then 1 mg prn
c. 2 mg initially, then 1 mg after every loose stool
d. 4 mg initially, then 2 mg after every loose stool
124. Le métronidazole ne doit pas être administré avec de l'alcool
pour éviter l'effet secondaire possible suivant?
A.
B.
C.
D.
Metronidazole should not be used with alcohol because of fear of
which of the following possible side effect?
La réaction disulfirame like
La Crise hypertensive
Le syndrome sérotoninergique
L’Hyponatrémie
a. Disulfiram like reaction
b. Hypertensive crisis
c. Serotonergic syndrome
d. Hyponatremia
33
.654
Quel médicament serait le plus approprié pour le soulagement
de certains signes et symptômes de l'encéphalopathie chez les
alcooliques?
A. L’Esmoprazole
B. Le lactulose
C. Le Propranolol
D. L’Otreotide
126. Parmi les AINS, l’aspirine est unique parce qu'il :
A. Inhibe de manière irréversible l'enzyme cible
B. Empêche les épisodes d'arthrite goutteuse avec l'utilisation à
long terme
C. Réduit la fièvre
D. Inhibe sélectivement la voie de la COX-2
Which drug would be most suitable for the relief of some of the
signs and symptoms of encephalopathy in alcoholic patients?
A. Esmoprazole
B. Lactulose
C. Propranolol
D. Otreotide
Among NSAIDS, aspirin is unique because it
a. Irreversibly inhibits its target enzyme
b. Prevents episodes of gouty arthritis with long term use
c. Reduces fever
d. Selectively inhibits the COX-2 pathway
.653
Un patient atteint de tuberculose et traité avec l'isoniazide
développe des paresthésies, des douleurs musculaires, et une
instabilité. Laquelle des vitamines suivantes doit être donnée afin
d'inverser ces symptômes?
A. La vitamine A
B. La vitamine B1
C. La vitamine B6
D. La vitamine K
34
A patient with tuberculosis treated with isoniazide develops
paresthesias, muscle aches, and unsteadiness. Which of the
following vitamins should be given in order to reverse these
symptoms?
A. Vitamin A
B. Vitamin B1
C. Vitamin B6
D. Vitamin K
128. Pour assurer une administration adéquate de pantoprazole, vous
recommandez de :
A.
B.
C.
D.
Prendre le comprimé en cas de besoin en cas de symptômes
Prendre le comprimé 30 minutes avant le petit déjeuner
Prendre un comprimé après chaque repas
Prendre le comprimé à tout moment pendant la journée,
indépendamment des repas
When counseling on the adequate administration of pantoprazole
what would you recommend?
a. Take the tablet when needed, when you experience the
symptoms
b. Take the tablet 30 minutes before breakfast
c. Take one tablet after each meal
d. Take the tablet any time during the day irrespective of
meals
129. Lequel des énoncés suivants concernant le célécoxib est vrai?
Which of the following statements about celecoxib is true?
A. Il acétyle irréversiblement l'enzyme COX-2
B. Il inhibe à la fois inductible et constitutive de la COX-2
enzyme
C. Il ne produit pas de saignements gastro-intestinaux
D. Il augmente la cicatrisation des ulcères gastro-intestinaux
a. It irreversibly acetylates the COX-2 enzyme
b. It inhibits both the inducible and constitutive COX-2
enzyme
c. It does not produce GI bleeding
d. It increases healing of GI ulcers
35
.674
Un patient prend un inhibiteur de la pompe à protons, du
bismuth, du métronidazole et de la tétracycline. Quel est l'état le
plus probable pour lequel cette association médicamenteuse est
utilisée?
A. La Colite pseudomembraneuse associée aux antibiotiques
B. Le syndrome du côlon irritable
C. Les Ulcères gastriques secondaires à Helicobacter pylori
D. L’ulcère gastrique qui a eu lieu en réponse à fortes doses
d'AINS pour l'arthrite
131. Un homme de 45 ans souhaite prendre un médicament antiinflammatoire non stéroïdien et veut éviter les effets secondaires
gastro-intestinaux. Lequel des médicaments suivants est le plus
approprié?
A.
B.
C.
D.
L’Aspirine
Le célécoxib
L'ibuprofène
Le Piroxicam
36
A patient is taking proton pump inhibitor plus bismuth,
metronidazole and tetracycline. What is the most likely condition
for which this drug combination is being used?
A. Antibiotic-associated pseudomembranous colitis
B. Irritable bowel syndrome
C. Gastric ulcers secondary to Helicobacter pylori
D. Gastric ulcer that occurred in response to high dose NSAID
therapy for arthritis
A 45-year-old male wishes to take a nonsteroidal anti-inflammatory
drug and wants to avoid gastrointestinal side effects. Which one of
the following drugs is most appropriate?
a. Aspirin
b. Celecoxib
c. Ibuprofen
d. Piroxicam
132. Lequel des laxatifs suivants peut être utilisé dans le traitement
de l'encéphalopathie hépatique?
A.
B.
C.
D.
Le Lactulose
L'huile minérale
Le Sorbitol
Le Senné
Which of the following laxatives can be used in the treatment of
hepatic encephalopathy?
a. Lactulose
b. Mineral oil
c. Sorbitol
d. Senna
133. Lequel des médicaments suivants n'est pas utile dans la
dysménorrhée?
A.
B.
C.
D.
.670
Which one of the following drugs is not useful in dysmenorrhea?
a.
b.
c.
d.
L’Aspirine
La Colchicine
L'ibuprofène
Le Naproxen
Un patient admis dans le service de gastroentérologie est traité
avec la sulfasalazine. Quel est l'objectif le plus probable dans
l’administration de ce médicament?
A. La Colite pseudomembraneuse associée aux antibiotiques
B. La diarrhée induite par l’Escherichia coli
C. Les Infections gastriques de H. pylori
D. Les maladies inflammatoires de l'intestin
37
Aspirin
Colchicine
Ibuprofen
Naproxen
A patient admitted to the gastroenterology service is being treated
with sulfasalazine. What is the most likely purpose for this drug to
be given?
A. Antibiotic-associated pseudomembranous colitis
B. Echerichia coli-induced diarrhea
C. Gastric H. pylori infections
D. Inflammatory bowel disease
135. Lequel des laxatifs suivants est utilisé pour traiter la constipation
aiguë, car il produit un mouvement de l'intestin dans environ 30
minutes?
A.
B.
C.
D.
Le Psyllium
le Lactulose
la Glycérine
d. La Docusate
Which of the following laxatives is used to treat acute constipation
since it produces a bowel movement within about 30 minutes?
a. Psyllium
b. Lactulose
c. Glycerin
d. Docusate
.675
Quel est le mécanisme d'action de l'ondansétron?
A. Le Blocage des récepteurs de la sérotonine centrale (5-HT3)
B. Le Blocage des récepteurs de la dopamine
C. Le Blocage des récepteurs de l'histamine H1 dans le tronc
cérébral et l'oreille interne
D. La Suppression de la motilité gastrique et la sécrétion acide
par le blocage muscarinique
137. Lequel des médicaments suivants peut être utilisé en toute
sécurité chez les femmes enceintes au cours du troisième
trimestre de la grossesse?
A.
B.
C.
D.
L'acétaminophène
Les médicaments anti-inflammatoires non stéroïdiens
La Warfarine
L’Aspirine
What is the mechanism of action of ondansetron?
A. Blocking central serotonin (5-HT3) receptors
B. Blocking dopamine receptors
C. Blocking histamine H1 receptors in the brainstem and inner
ear
D. Suppressing gastric motility and acid secretion via
muscarinic blockade
Which of the following medications can be safely used in pregnant
patients
during the third trimester?
a.
b.
c.
d.
38
Acetaminophen
Nonsteroidal anti-inflammatory drugs
Warfarin
Aspirin
138. Un pharmacien a reçu une ordonnance pour l'alendronate 35 mg
une fois par semaine pour la prévention de l'ostéoporose avec
des instructions pour le patient à prendre avec de la nourriture et
de rester debout pendant au moins 30 minutes après l'ingestion.
Lequel des éléments suivants peuvent être utilisé pour corriger la
prescription ci-dessus?
A. L'alendronate doit être pris au moins 30 minutes après un
repas, de préférence le petit déjeuner
B. L'alendronate doit être pris avant le coucher
C. Les patients doivent se coucher pendant 1 heure après
l'administration d'alendronate
D. L'alendronate doit être pris en dehors des repas en début de
matinée
.678
Le Célécoxib est préféré aux AINS classiques:
A. chez les patients ayant une insuffisance cardiaque
B. chez les patients qui sont à risque élevé de développer un
ulcère gastro-duodénal
C lorsque le début du soulagement de la douleur est nécessaire
immédiatement
D. chez les patients présentant un infarctus du myocarde
140. La morphine est contre-indiquée en cas de blessure à la tête car
A.
B.
C.
D.
Elle ne soulage pas la douleur de la blessure à la tête
Elle peut faire monter la tension intracrânienne
Elle peut augmenter la pression artérielle systolique
Elle est susceptible de causer une dépendance
A pharmacist received a prescription for Alendronate 35 mg once
weekly for prevention of osteoporosis with instructions to the
patient to take with food and remain upright for at least 30
minutes following ingestion. Which of the following can be used to
correct the above prescription?
a. Alendronate should be taken at least 30 minutes after a
meal, preferably breakfast
b. Alendronate should be taken before bedtime
c. Patients should lie down for 1 hour following administration
of Alendronate
d. Alendronate should be taken without food in the early
morning
Celecoxib is preferred to standard NSAIDs:
A. in patients who have cardiac failure
B. in patients who are at high risk of developing
gastroduodenal ulcer
C. when onset of pain relief is required immediately
D. in patients with myocardial infarction
Morphine is contraindicated in head injury because
a.
b.
c.
d.
39
It does not relieve the pain of head injury
It can raise intracranial tension
It can raise systolic blood pressure
It is liable to cause addiction
141. Laquelle des recommandations suivantes est la plus adéquate
chez un patient traité avec de la warfarine ?
a. Évitez tous les aliments contenant de la vitamine K
b. Évitez la consommation excessive d'alcool, car il diminue
l'efficacité de la warfarine
c. Maintenez un apport alimentaire en vitamine K cohérent
d. Maintenez un régime actuel, car les vitamines et les
suppléments alimentaires interagissent très peu avec la
warfarine
142. LR est un homme de 60 ans ayant des antécédents de fibrillation
auriculaire, de diabète sucré et d’hypertension. Le cardiologue
voudrait commencer la prévention primaire de l'accident
vasculaire cérébral chez le patient. Sa clairance de la créatinine
est d'environ 15ml/min et les tests de la fonction hépatique sont
normaux. Lequel des médicaments suivants est le plus
approprié?
a. La warfarine
b. Le dabigatran
c. Le rivaroxaban
d. ASA + clopidogrel
143. Un homme de 59 ans ayant des antécédents de diabète sucré et
d'hypertension est admis à l'hôpital pour un infarctus du
myocardel (MI) avec élévation du segment ST. Lequel des
médicaments suivants devrait faire partie de son traitement
médical lors de la décharge?
A. Le lorazepam
B. Le bisoprolol
C. L'ibuprofène
D. d. Le furosémide
40
Which of the following is proper nutrition counseling for patients
who are being treated with warfarin?
a. Avoid all foods containing vitamin K
b. Avoid excessive alcohol intake as it decreases the
effectiveness of warfarin
c. Maintain a consistent dietary vitamin K intake
d. Maintain current diet because vitamins and dietary
supplements minimally interact with warfarin
L.R is a 60 year old man with a history of Atrial Fibrillation, Diabetes
Mellitus and Hypertension. The cardiologist would like to start the
patient on primary stroke prevention. His creatinine clearance is
around 15ml/min and normal Liver function tests. Which of the
following is the most appropriate recommendation?
a. Warfarin
b. Dabigatran
c. Rivaroxaban
d. ASA + Clopidogrel
A 59 year old male with a previous history of diabetes mellitus and
hypertension is admitted to the hospital with an ST elevation
mycocardial infarction (MI). Which of the following medications
should be part of his medical regimen upon discharge?
a. Lorazepam
b. Bisoprolol
c. Ibuprofen
d. Furosemide
144. Une femme de 42 ans qui a récemment perdu son mari vous dit
qu'elle ne dort pas la nuit. Après son interrogatoire, vous
constatez qu'elle n'a pas la dépression ou la toxicomanie. Que
recommanderiez-vous?
a. Recommander une benzodiazépine à action prolongée
b. Recommander une benzodiazépine à action brève
c. Recommander la fluoxétine
d. Recommander la trazadone
145. Un homme de 50 ans atteint de cirrhose alcoolique et d’ascite
abdominale est admis en urgence. Il présente depuis trois jours
de la fièvre, des frissons, des nausées / vomissements et des
douleurs abdominales. Une paracentèse a été effectuée et a
révélé un liquide d'ascite nuageux. Le liquide d'ascite a été
envoyé pour la culture et la cytologie. La meilleure
antibiothérapie empirique pour ce patient est:
A. L’ azithromycine
B. La Cefepime
C. L’Amoxicilline
D. La Ceftriaxone
A 42-year-old female who recently lost her husband tells you that
she is not sleeping at night. After questioning her further, you
determined that she does not have depression or substance abuse.
What would you recommend?
a. A long acting Benzodiazepine
b. A short-acting Benzodiazepine
c. Fluoxetine
d. Trazodone
A 57-year-old male with alcoholic cirrhosis and abdominal ascites
presents to the Emergency room with a 3 day history of fever,
chills, nausea/vomiting and abdominal pain. Paracentesis was
performed and revealed a cloudy ascitic fluid. The ascitic fluid was
sent for culture and cytology. The best empirical antibiotic regimen
for this patient is:
a. Azithromycin
b. Cefepime
c. Amoxicillin
d. Ceftriaxone
146. Une femme de 19 ans, enceinte se présente à votre pharmacie A 19 year-old pregnant female presents to your pharmacy with
avec des symptômes de dysurie, pollakiurie et de l'urgence symptoms of dysuria, urinary frequency and urgency. Which of the
urinaire. Lequel des médicaments suivants est le meilleur agent following is the best empirical antibiotic agent for her cystitis?
a. Ciprofloxacin
antibiotique empirique pour sa cystite?
b. Sulfamethoxazole/trimethoprim
A. Ciprofloxacine
c. Nitrofurantoin
B. Sulfaméthoxazole / triméthoprime
d. Vancomycin
C. Nitrofurantoïne
D. Vancomycine
41
147. Un homme de 23 ans a été mordu par le chien de son ami il y a 2
jours. Il présente aujourd'hui une plaie avec un écoulement
purulent qui ne guérit pas. Lequel des suivants est le traitement
antimicrobien le plus approprié?
A. L’amoxicilline / acide clavulanique
B. La doxycycline
C. Le sulfaméthoxazole / triméthoprime
D. L’érythromycine
148. Une femme se présente à la clinique à 19 semaines de gestation.
Elle se plaint de la fièvre, une sensation de brûlure à la miction.
Elle est allergique à la pénicilline. Lequel des antibiotiques
suivants recommanderiez-vous?
A. L’amoxicilline / acide clavulanique
B. TMP / SMX
C. La Nitrofurantoïne
D. . L’ évofloxacine
149. Une femme de 29 ans, enceinte a des antécédents d'asthme et
devient à court de souffle quand elle exerce. Quel agent
recommanderiez-vous pour empêcher l'asthme induit par
l'exercice de ce patient?
A. Le Zileuton
B. L’Albuterol
C. Le Cromolyn
D. d. Le Formotérol
42
A 23 year-old male patient who got bitten by his friend’s dog 2 days
ago. He presents today with a non-healing wound draining purulent
material. Which of the following is the most appropriate
antimicrobial treatment?
a. Amoxicillin/clavulanic acid
b. Doxycycline
c. Sulfamethoxazole/trimethoprim
d. Erythromycin
A female presents to the clinic at 19 weeks of gestation
complaining of fever, burning sensation upon urination. She is
allergic to penicillin. Which of the following antibiotics would you
recommend?
a. Amoxicillin/Clavulanic acid
b. TMP/SMX
c. Nitrofurantoin
d. Levofloxacin
A 29-year old pregnant female has a history of asthma and
becomes short of breath when she exercises. What agent would
you recommend to prevent this patient’s exercise-induced asthma?
a. Zileuton
b. Albuterol
c. Cromolyn
d. Formeterol
150. Lequel des effets secondaires suivants est le plus probable Which of the following side effect is MOST likely to result from the
use of Metformin?
résultant de l'utilisation de la metformine?
a. Edema
A. L’ Œdème
b. Cardiac toxicity
B. La toxicité cardiaque
c. Gastrointestinal disturbances
C. les Troubles gastro-intestinaux
d. Hyperkalemia
D. L’Hyperkaliémie
151. Quel est l'effet de l'exposition prolongée aux œstrogènes sans What is the effect of prolonged estrogen exposure unopposed by
progesterone on the endometrium?
opposition par la progestérone sur l'endomètre?
a. Metabolic syndrome
A. Le syndrome métabolique
b. Dysmenorrhea
B. La dysménorrhée
c. Endometrial hyperplasia
C. L’Hyperplasie de l'endomètre
d. Endometriosis
D. L’Endométriose
152. Laquelle des classes suivantes de médicaments est plus probable Which of the following classes of medications is MOST likely to
cause drug-induced hyperprolactinemia?
de causer une hyperprolactinémie induite ?
a. ACE-Inhibitors
A. Les Inhibiteurs de l'ECA
b. B-blockers
B. Les B-bloquants
c. Oral hypoglycemics
C. Les Hypoglycémiant oraux
d. Antidepressants
D. Les Antidépresseurs
153. Lequel des dérivés suivants n'est PAS un analogue de l'insuline à Which of the following is NOT a rapid acting insulin analog?
a. Insulin LisPro
action rapide ?
b. Insulin Glulisine
A. L'insuline lispro
c. Insulin Glargine
B. L'insuline glulisine
d. Insulin Aspart
C. L'insuline glargine
D. L'insuline asparte
43
154. Quel énoncé à propos de fluconazole est exacte?
A. Ne pénètre pas la barrière hémato encéphalique
B. A le moins d'effet de toutes les azoles sur le
métabolisme hépatiquedes médicaments
C. est un inducteur des enzymes hépatiques qui
métabolisent les médicaments
D. est très efficace dans le traitement de l'aspergillose
155. S'agissant de l'utilisation clinique de formulations de liposomes
de l'amphotéricine B, quelle déclaration est exacte?
A. Ils n'affectent pas la toxicité liée à la perfusion
B. Moins cher à utiliser que l'amphotéricine B
C. Plus efficace dans les infections fongiques, car ils augmentent
l'absorption tissulaire d'amphotéricine
D.Ils diminuent la néphrotoxicité de l'amphotéricine B
156. Lequel des énoncés suivants est correct ?
A. La Digoxine – inhibe le canal Ca2+
B. La Dobutamine – active les recepteurs bêta 1 adrénergiques
C. Le Valsartan - bloque les récepteurs bêta-2 adrénergiques
D. La Spironolactone - réduit l'angiotensine II
Which statement about fluconazole is accurate?
A. Does not penetrate the blood-brain barrier
B. Has the least effect of all azoles on hepatic drug
metabolism
C. Is an inducer of hepatic drug-metabolizing enzymes
D. Is highly effective in the treatment of aspergillosis
157. La Lidocaïne:
A. Est un anti-arythmique de classe III
B. est administrée par IV
C est éliminée par voie rénale
D. bloque les courants de Ca2+
Lidocaine:
A. Is a class III anti-arrhythmic
B. Is administered IV
C. Is renally eliminated
D. Blocks Ca2+ currents
44
Regarding the clinical use of liposomal formulations of
amphotericin B, which statement is accurate?
A. They do not affect the infusion-related toxicity
B. They are less expensive to use than conventional
amphotericin B
C. They are more effective in fungal infections because they
increase tissue uptake of amphotericin B
D. They decrease the nephrotoxicity of amphotericin B
Which of the following drug-mechanism of action pairs is properly
matched?
A. Digoxin – inhibits Ca2+ channel
B. Dobutamine – activates beta 1 adrenergic receptors
C. Valsartan – blocks beta 2 adrenergic receptors
D. Spironolactone – reduces angiotensin II
158. Cet anti-arythmique est associé à la fibrose pulmonaire This antiarrhythmic is associated with potentially fatal pulmonary
fibrosis:
potentiellement fatale:
A. Propranolol
A. Le Propranolol
B. Mexiletine
B. Le Mexiletine
C. Flecainide
C. La Flécaïnide
D. Amiodarone
D. L’amiodarone
159. Cette bêta-lactamine est efficace contre le Pseudomonas This beta-lactam is effective against pseudomonas Aeruginosa:
A. Carbenicillin
aeruginosa, laquelle ?
B. Ampicillin
A. Le Carbinecillin
C. Nafcillin
B. L’ ampicilline
D. Ceftaroline
C. Le Nafcillin
D. La Ceftaroline
160. Lequel des mécaments suivants n’est pas un analgésique Which of the followings is not an opioid analgesic?
a. Morphine sulfate
opioïde?
b. Oxycodone
A- Le sulfate de morphine
c. Diclofenac
B- L’Oxycodone
d. Fentanyl
C- Le Diclofenac
D- Le Fentanyl
161. La Méthadone est souvent utilisée dans les programmes de Methadone is often used in drug treatment programs for narcotic
traitement de la dépendance aux narcotiques. Quelle est la addiction. What is the classification of this medication?
classification de ce médicament?
a. Non-steroidal anti-inflammatory
A. Anti-inflammatoires non stéroïdiens
b. Opioid analgesic
B. Analgésique opioïde
c. Antimigrainous agent
C. Agent antimigraineux
d. Agent for gout
Agent antigoutteux
45
162. Quelle est la classification de l'Olanzapine?
A. Un Antidépresseur
B. Un Antipsychotique
C. Un Antiépileptique
Un Hypnotique
What is the classification of olanzapine?
a. Antidepressant
b. Antipsychotic
c. Antiepileptic
d. Hypnotic
163. Quel est le mécanisme d'action de Ramelteon?
A. Un inhibiteur de la Catéchol-O-méthyl transférase (COMT)
B. Un agoniste des récepteurs de la mélatonine
C. Un Antipsychotique
D. Un Inhibiteur Sélectif de la recapture de la sérotonine (ISRS)
What is the mechanism of action of ramelteon?
a. Catechol-O-methyl transferase (COMT) Inhibitor
b. Melatonin receptor agonist
c. Antipsychotic drug
d. Selective serotonin reuptake inhibitor (SSRI)
164. Lequel des médicaments suivants ne se lie pas aux protéines Which one of the following drugs does NOT bind to plasma protein
to a significant extent?
plasmatiques dans une mesure significative?
a. ketoprofen
a. La kétoprofène
b. allopurinol
b. L’allopurinol
c. propranolol
c. Le propranolol
d. warfarin
d. La warfarine
165. Une femme de 53 ans ingère la moitié d'une bouteille de
comprimés d'aspirine et se présente plus tard au service
d'urgence avec des douleurs gastro-intestinales et des
vomissements. Le traitement de cette patiente pourrait inclure
l'administration de:
A. Bicarbonate de sodium, car il augmente l'ionisation des
salicylés dans les urines
B. Bicarbonate de sodium, car il diminue l’ionisation des
salicylés dans les urines
C. L'acide ascorbique, car il augmente l'ionisation des salicylés
dans les urines
D. L'acide ascorbique, car il diminue l'ionisation des salicylés
dans les urines
46
A 53-year-old woman ingests half of a bottle of aspirin tablets and
later presents to the emergency department with gastrointestinal
pain and vomiting. The treatment of this patient would include
administration of:
a. Sodium bicarbonate because it increases ionization of
salicylate in the urine
b. Sodium bicarbonate because it decreases ionization of
salicylate in the urine
c. Ascorbic acid because it increases ionization of salicylate in
the urine
d. Ascorbic acid because it decreases ionization of salicylate in
the urine
166. Une femme de 33 ans est amenée au service d'urgence après
l'ingestion d'une grande quantité inconnue d’Acétaminophène.
L’utilisation de la N-acétylcystéine dans le traitement de cette
patiente est basée sur le but de:
A. L'inhibition de l'absorption du médicament
B. L'augmentation de l'élimination du médicament
C. L’inactivation du métabolite toxique
D. La diminution de la liaison du récepteur cible
167. L’administration oculaire topique de phényléphrine ou
tropicamide peut causer :
A. Cycloplegia
B. Myosis
C. Mydriase
D. Cycloplegia et Myosis
A 33-year-old woman is brought to the emergency department
after ingesting a large but unknown quantity of acetaminophen.
Use of N-acetylcysteine in the treatment of this patient is based on
the goal of:
a. Inhibiting drug absorption
b. Increasing drug elimination
c. Inactivating a toxic metabolite
d. Decreasing target receptor binding
Topical ocular administration of either phenylephrine or
tropicamide may cause
a. Cycloplegia
b. Miosis
c. Mydriasis
d. Cycloplegia and Miosis
168. L’efficacité de la décontamination gastro-intestinale varie en Gastrointestinal decontamination efficacy varies based on:
fonction:
a. Time of chemical ingestion
A. Du moment de l’ingestion du produit chimique
b. Response to the ingested chemical
B. De la réponse au produit chimique ingéré
c. Toxicant serum levels
C. Des taux sériques de produit toxique
d. Metabolism of toxicants
D. Du métabolisme des substances toxiques
169. L’Ipecac est contre-indiqué chez les patients:
a. Avec un réflexe nauséeux
b. Prévus à ne plus répondre dans les quatre prochaines heures
c. Affaiblis, les femmes enceintes, et les personnes âgées
d. Les patients cardiovasculaires
47
Ipecac is contraindicated in patients:
a. With a gag reflex
b. Expected to become unresponsive within next four hours
c. Debilitated, pregnant, and elderly
d. Cardiovascular patients
170. La diminution de l'absorption de la toxine comprend:
A. La Diurèse
B. La modification de pH urinaire
C. Le Charbon activé
D. L’Hémodialyse
Decrease absorption of toxin include:
a. Diuresis
b. Manipulation of urine pH
c. Activated Charcoal
d. Hemodialysis
171. L'antidote qui est utilisé dans la gestion de l'intoxication par The antidote that is used in the management of the intoxication by
Opiates is :
opiacés est:
a. Oxygen
a. L’Oxygène
b. Naloxone
b. La Naloxone
c. Physostigmine
c. Le Physostigmine
d. Atropine
d. L'atropine
172. La toxidrome narcotique comprend:
A.
B.
C.
D.
La Salivation
Le Larmoiement
L’Hypertension
La dépression respiratoire
173. Lequel des médicaments suivants,
probablement une acidose métabolique?
A.
B.
C.
D.
Narcotic toxidrome include:
a. Salivation
b. Lacrimation
c. Hypertension
d. Respiratory depression
La Théophylline
La Nicotine
Le Lévothyroxine
L’Acétazolamide
provoque
le
plus Which one of the following drugs is most likely to cause metabolic
acidosis?
a. Theophylline
b. Nicotine
c. Levothyroxine
d. Acetazolamide
174. Lequel des suivants n'est pas un effet indésirable connu de la
cyclosporine?
A. a. L’Hirsutisme
B. La Néphrotoxicité
C. L’Hyperplasie gingivale
D. L’Ulcération buccale
48
Which of the followings is NOT a known adverse effect of
cyclosporine?
a. Hirsutism
b. Nephrotoxicity
c. Gingival hyperplasia
d. Oral ulceration
175. Lequel des médicaments suivants ne nécessitera pas un
ajustement rénale chez les patients avec des fonctions rénales
diminuées?
a. L’Allopurinol
b. L'atorvastatine
c. Le Mepiridine
d. Le Linézolide
176. Lequel des médicaments suivants entraîne une nécrose tubulaire
aiguë?
a. La Cisplatine
b. Le méthotrexate
c. L’Héroïne
d. L’Or
177. Lequel des énoncés suivants serait le plus bénéfique à retarder la
progression de la maladie rénale chronique chez un patient
atteint de diabète de type 1 et de microalbuminurie?
A.
B.
C.
D.
L’inhibiteur de l'ECA et le diurétique de l'anse
L’inhibiteur de l'ECA et / ou ARA
L’ARA et le diurétique de l'anse
Les inhibiteurs de l'ECA, l’ARA et le diurétique de l'anse
Which of the following drugs will not require renal adjustment in
patients with declined kidney functions?
a. Allopurinol
b. Atorvastatin
c. Mepiridine
d. Linezolid
Which of the following drugs will cause acute tubular necrosis?
a. Cisplatin
b. Methotrexate
c. Heroin
d. Gold
Which of the following would be most beneficial in a patient with
type 1 diabetes and microalbuminuria to delay progression of
Chronic Kidney Disease?
a. ACE inhibitor and loop diuretic
b. ACE inhibitor &/or ARB
c. ARB and loop diuretic
d. ACE Inhibitor, ARB and loop diuretic
178. Quel (s) médicament (s) est / sont classé (s) comme inhibiteur de Which medication (s) is/are classified as calcineurin inhibitor?
la calcineurine ?
a. Sirolimus and cyclosporin
A. Le Sirolimus et la Cyclosporine
b. Tacrolimus and cyclosporin
B. Le Tacrolimus et la Cyclosporine
c. Sirolimus and tacrolimus
C. Le Sirolimus et le Tacrolimus
d. Sirolimus, tacrolimus and cyclosporin
D. Le Sirolimus, le Tacrolimus et la Ciclosporine
49
179. Lequel des énoncés suivants n'est pas un analogue de la vitamine
D activée?
A. Le Calcitriol
B. L’Ergocalciferol
C. Le Paricalcitol
D. L’ Alfacalcidol
Which of the following is NOT an activated Vitamin D analogue?
a. Calcitriol
b. Ergocalciferol
c. Paricalcitol
d. Alfacalcidol
180. Quelle est l’effet indésirable le plus fréquemment rapporté avec What is the most common reported adverse reactions with epoetin
l'époétine alfa et la darbépoétine alfa?
alfa and darbepoetin alfa?
A. Nausée
a. Nausea
B. Constipation
b. Constipation
C. Hypertension
c. Hypertension
D. Anaphylaxie
d. Anaphylaxis
181. Lequel des éléments suivants doit être surveillé si un malade Which one of the following should be monitored if a patient with
atteint de virus d’immunodéficience humaine (VIH) reçoit comme human immunodeficiency virus (HIV) is receiving fosamprenavir
traitement à base de fosamprénavir / ritonavir?
/ritonavir based regimen?
A. La neuropathie périphérique
a. Peripheral neuropathy
B. Les Interactions médicamenteuses avec les médicaments
b. Drug interactions with drugs metabolized by CYP1A2
métabolisés par le CYP1A2
c. Hyperglycemia, fat redistribution, and lipid abnormalities
C. L’Hyperglycémie, la redistribution des graisses, et les
d. Nephrolithiasis
anomalies lipidiques
D. La Nefrolithiase
182. BK est un malade de 29 ans séropositif avec nombre de CD4 BK is a 29-year-old HIV-positive patient with CD4 count of 75/mm3.
75/mm3. Il a reçu le diagnostic de cryptococcose méningée et est He received the diagnosis of cryptococcal meningitis and is
traité avec succès. Lequel des énoncés suivants est le meilleur successfully treated. Which of the following is the best follow-up
traitement à suivre pour ce patient?
therapy for this patient?
A. Le Désoxycholate d'amphotéricine B
a. Amphotericin B desoxycholate
B. La Caspofungine
b. Caspofungin
C. La terbinafine
c. Terbinafine
D. Le Fluconazole
d. Fluconazole
50
183. Lequel des énoncés suivants est le principal effet secondaire de
l'éthambutol?
a. L’Urine décolorée
b. La Névrite optique
c. L’Ostéoporose
d. La Neutropénie
.640
Un homme de 45 ans demande à son médecin une prescription
de sildénafil pour améliorer ses performances sexuelles. En
raison des risques d'une interaction médicamenteuse grave, ce
médicament ne doit pas être prescrit avec :
A. Les Inhibiteurs de l'enzyme de conversion de l'angiotensine
B. Les Bloquants Β-adrénergiques
C. Les vasodilatateurs nitrés
D. Les Diurétiques thiazidiques
.644
Quel est le mécanisme qui conduit aux effets bénéfiques de
l'Altéplase?
.645
A. Blocage des récepteurs ADP des plaquettes
B. Inhibition de la production de thromboxane par les
plaquettes
C. Inhibition de la synthèse ou de l'activation des facteurs de
coagulation dépendant de la vitamine K
D. Promouvoir la conversion du plasminogène en plasmine
Lequel des énoncés suivants n'est pas un effet négatif potentiel
de la clozapine?
A. Le gain de poids
B. Le pseudoparkinsonisme
C. La sédation
D. La rétention urinaire
51
Which of the following is the main side effect of ethambutol?
a. Discolored urine
b. Optic neuritis
c. Osteoporosis
d. Neutropenia
A 45 year old man asks his physician for a prescription for sildenafil
to improve his sexual performance. Because of risks from a serious
drug interaction, this drug should not be prescribed with:
A. Angiotensin-converting enzyme inhibitors
B. Β-adrenergic blockers
C. Nitrovasodilators
D. Thiazide diuretics
What is the mechanism that leads to the beneficial effects of
alteplase?
A. Blocking platelet ADP receptors
B. Inhibiting platelet thromboxane production
C. Inhibiting synthesis or activation of vitamin K-dependent
coagulation factors
D. Promoting conversion of plasminogen to plasmin
Which of the following is NOT a potential adverse effect of
clozapine?
A. weight gain
B. pseudoparkinsonism
C. sedation
D. urinary retention
.643
.644
.648
Lequel des antipsychotiques atypiques suivants serait le moins
susceptible de causer des symptômes extrapyramidaux?
A. Le rispéridone
B. L’olanzapine
C. La quétiapine
D. L’aripiprazole
Une femme de 36 ans se présente avec une histoire de 2 mois
d’humeur dépressive. C'est le premier épisode de dépression
majeure de la patiente. Lequel des antidépresseurs suivants,
serait le plus approprié pour le traitement de cette patiente?
A. La nortriptyline
B. L’escitalopram
C. La phénelzine
D. La mirtazapine
Un patient qui a reçu la dose efficace optimale de sertraline
pendant 2 semaines pour le traitement de son premier épisode
de dépression majeure se plaint que le médicament ne
fonctionne pas et qu’il souhaite changer de médicament. Quelle
est la recommandation appropriée?
A. A.Appelez le médecin pour recommander le passage du
patient aux IMAOs
B. Encourager le patient à continuer avec le régime actuel, et lui
informer qu'il peut prendre 4-6 semaines avant la réponse
complète est évidente.
C. Recommander ajoutant lithium pour augmenter le régime
actuel.
D. Recommander combinant avec la mirtazapine.
52
Which of the following atypical antipsychotics would be the least
likely to cause extrapyramidal symptoms?
A. risperidone
B. olanzapine
C. quetiapine
D. Aripiprazole
A 36-year –old woman presents with a 2-month history of
depressed mood. This is the patient's first episode of major
depression. Which of the following antidepressants would be most
appropriate in the treatment of this patient?
A. nortriptyline
B. escitalopram
C. phenelzine
D. mirtazapine
A patient who has received sertraline optimal effective dose for 2
weeks for the treatment of her first episode of major depression
complains that the medication is not working and would like to be
switched to another agent . What is the appropriate
recommendation?
A. Call the physician to recommend switching the patient to
MAOIs
B. Encourage the patient to continue with the current
regimen, and inform him or her that it may take 4-6 weeks
before the full response is evident .
C. Recommend adding lithium to augment the current
regimen.
D. Recommend combining with mirtazapine.
.684
.686
.685
.687
Lequel des stabilisateurs de l'humeur suivants serait le plus
approprié dans les maladies du foie?
A. Le lithium
B. L’acide valproïque
C. La carbamazépine
D. La lamotrigine
Un patient avec son premier épisode majeur de dépression doit
recevoir un traitement antidépresseur pendant au moins ---------- après la rémission
A. 2 semaines.
B. 6 semaines.
C. 2 mois.
D. 6 - 12 mois
Lequel des médicaments suivants a un risque de nécrose de la
mâchoire?
A. Le carbonate de calcium
B. Le bisphosphonate
C. Le Raloxifen
D. L’oméprazole
Laquelle des catégories suivantes de médicaments est plus
probable de causer une hyperprolactinémie?
A. Les bêta-bloquants
B. Les antidépresseurs
C. Les antihistaminiques
D. Les contraceptifs oraux
53
Which of the following mood stabilizers would be most appropriate
in liver disease?
A. lithium
B. valproic acid
C. carbamazepine
D. lamotrigine
A patient with his first major depression episode should receive
antidepressant therapy for at least----------- after remission
A. 2 weeks.
B. 6 weeks.
C. 2 months.
D. 6– 12 months
Which of the following drugs has a risk of jaw necrosis?
A. Calcium carbonate
B. bisphosphonate
C. Raloxifen
D. Omeprazole
Which of the following classes of medications is most likely to
cause hyperprolactinemia?
A. Beta-blockers
B. Antidepressants
C. Antihistamines
D. Oral contraceptives
.680
.684
.685
Pour les patients qui étaient sous une thérapie à glucocorticoïde
de long terme et qui souhaitent arrêter le médicament, un arrêt
progressif de glucocorticoïdes est nécessaire pour permettre la
récupération de:
A. La hématopoïèse dans la moelle osseuse
B. La fonction des ostéoblastes normale
C. Le contrôle par la vasopressine de l'excrétion de l'eau
D. Le système hypothalamo-pituito-surrénalien
Lequel des glucocorticoïdes suivantes produit le moins une
rétention de sodium?
A. L’Hydrocorticosone
B. La Prednisolone
C. La dexaméthasone
D. La Fludrocortisone
Lequel des énoncés suivants décrit le mieux le mécanisme par
lequel la bromocriptine produit ses effets souhaités?
A. Le blocage des récepteurs d'oestrogène
B. L’Augmentation de la synthèse de l'hormone folliculostimulante (FSH)
C. L'inhibition de la libération de prolactine
D. La Stimulation de la synthèse des œstrogènes ovariens et de
progestatif
54
For patients who have been on long-term therapy with a
glucocorticoid and who wish to discontinue the drug, gradual
tapering of glucocorticoid is needed to allow recovery of:
A. Hematopoiesis in the bone marrow
B. Normal osteoblast function
C. The control by vasopressin of water excretion
D. The hypothalamic-pituitary-adrenal system
Which of the following glucocorticoids produces the least sodium
retention?
A. Hydrocorticosone
B. Prednisolone
C. Dexamethasone
D. Fludrocortisone
Which of the following statements best describes the mechanism
by which bromocriptine produces its desired effects?
A. Blockade of estrogen receptors
B. Increasingsynthesis of follicle-stimulating hormone (FSH)
C. Inhibition of prolactin release
D. Stimulation of ovarian estrogen and progestin synthesis
.683
.684
.688
.544
Une femme de 23 ans désire un contraceptif oral combiné pour
la protection de la grossesse. Lequel des facteurs du malade
suivants, conduirait un professionnel de la santé de
recommander une autre forme de contraception?
A. La Preuve de l'hirsutisme
B. L’Histoire de la maladie de reflux gastro-intestinal et est
actuellement en oméprazole
C. L’Histoire de la maladie inflammatoire pelvienne
D. L’Histoire de la migraine avec aura
Le niveau de glucose peut être augmenté dans le sang par
l'utilisation de:
A. Les glucocorticoïdes
B. La consommation de protéines
C. Les Biguanides
D. L'enzyme de conversion de l'angiotensine (IEC)
L’Amiodarone provoque des anomalies de la thyroïde
principalement due à:
A. sa grande teneur en iode
B. L’Inhibition de la thyroïde peroxydase
C. Le Blocage des récepteurs périphériques de la thyroxine
D. L'induction du catabolisme de la T3
Lequel des énoncés suivants, est NON utilisé en
pharmacothérapie de l'hyperplasie bénigne de la prostate?
A. L’oxybutynine
B. La prazosine
C. La goséréline
D. Le dutastéride
55
A 23 year old woman desires a combined oral contraceptive for
pregnancy protection. Which of the following patient factors would
lead a health professional to recommend an alternative form of
contraception?
A. Evidence of hirsutism
B. History of gastrointestinal reflux disease and is currently
taking omeprazole
C. History of pelvic inflammatory disease
D. History of migraine with aura
Blood glucose levels may be increased by the use of:
A. Glucocorticoids
B. Protein consumption
C. Biguanides
D. Angiotensin-converting enzyme (ACE) inhibitors
Amiodarone causes thyroid abnormalities mainly due to:
A. its large iodine content
B. Inhibition of thyroid peroxidase
C. Blockage of peripheral thyroxine receptors
D. Induction of the catabolism of T3
Which of the following is NOT used in pharmacotherapy of benign
prostatic hyperplasia?
A. oxybutynin
B. prazosin
C. goserelin
D. dutasteride
.546
.545
.547
.540
L'anesthésie générale par voie intraveineuse qui produit une
anesthésie dissociative est:
A. Le Midazolam
B. La Kétamine
C. Le Fentanyl
D. Le Thiopental
Indiquer un l'inhibiteur sélectif de la catéchol-Ométhyltransférase (COMT), ce qui augmente l'absorption
centrale de la lévodopa :
A. La Carbidopa
B. Le Benzhexol
C. L'entacapone
D. La sélégiline
Lequel des énoncés suivants est incorrect pour l'amitriptyline?
A. Elle inhibe la recapture de sérotonine
B. Elle inhibe la recapture de la noradrénaline
C. Elle est associée à la bouche sèche et la rétention d’urine
D. Elle est utilisée pour le traitement de l'anxiété
Les symptômes vasomoteurs continus chez une femme de 52 ans
ménopausée et recevant 0,3 mg d'estrogènes équins conjugués
par voie orale peuvent être gérés par:
A. La Modification à un régime équivalent en œstrogène
transdermique
B. La Modification à un modulateur sélectif des récepteurs aux
œstrogènes
C. L'augmentation de la dose quotidienne d'oestrogène
D. La diminution de la dose quotidienne d'oestrogène
56
The intravenous general anesthetic which produces dissociative
anesthesia is:
A. Midazolam
B. Ketamine
C. Fentanyl
D. Thiopental
Indicate a selective catechol-O-methyltransferase
inhibitor, which increases central uptake of levodopa:
A. Carbidopa
B. Benzhexol
C. Entacapone
D. Selegiline
(COMT)
Which of the following statements is incorrect for amitriptyline?
A. It inhibits serotonin reuptake
B. It inhibits noradrenaline reuptake
C. It is associated with dry mouth and urine retention
D. It is used for the treatment of anxiety
Continued vasomotor symptoms in a 52-year-old postmenopausal
woman receiving 0.3 mg of oral conjugated equine estrogens can
be managed by:
A. Changing to an equivalent transdermal estrogen regimen
B. Changing to a selective estrogen-receptor modulator
C. Increasing the daily estrogen dose
D. Decreasing the daily estrogen dose
.544
.545
.543
Une personne avec diabète sucré a une microalbuminurie.
L'intervention correcte en ce moment est de:
A. Démarrer l’Hydrochlorothiazide, sauf si hypertendu
B. Démarrer le Ramipril, sauf si hypertendu
C. Démarrer le Ramipril, même s'il n'est pas hypertendu
D.
Démarrer le Valsartan, sauf si hypertendu
Pourquoi le methimazole est il la thérapie antithyroïdienne de
choix dans la plupart des patients atteints de la maladie de
Graves?
A. Il a une demi-vie plus courte que le propylthiouracil.
B. Il est excrété par les reins, donc aucun réglage n'est nécessaire
pour une maladie du foie.
C. Il provoque moins d'effets indésirables tel que l'hépatotoxicité
D. Il bloque la conversion de T4 en T3.
Lequel parmi les médicaments antipsychotiques suivants, est le
meilleur traitement initial pour un patient souffrant de diabète
sucré et de schizophrénie?
A. L’Aripiprazole
B. L’Halopéridol
C. La Thioridazine
D.
La Clozapine
57
A person with diabetes mellitus has microalbuminuria. The correct
intervention at this time is to:
A. Start hydrochlorothiazide, only if hypertensive
B. Start ramipril, only if hypertensive
C. Start ramipril, even if not hypertensive
D. Start Valsartan, only if hypertensive
Why is methimazole the antithyroid therapy of choice in most
patients with Graves’ disease?
A. It has a shorter half-life than propylthiouracil.
B. It is renally excreted, so no adjustment is needed for liver
disease.
C. It causes fewer adverse effects such as hepatotoxicity
D. It blocks the conversion of T4 to T3.
Which of the following antipsychotic medications is the best initial
treatment for a patient suffering from diabetes mellitus and
schizophrenia?
A. aripiprazole
B. haloperidol
C. thioridazine
D. clozapine
208. Un patient prend une dose potentiellement mortelle de A patient took a potentially lethal overdose of aspirin. What drug
l'aspirine. Quel médicament adjoint serait-il utile, si risque vital, would be a helpful, if not lifesaving, adjunct to manage this severe
pour gérer cette intoxication grave à l'aspirine?
aspirin poisoning?
a. L'acétaminophène
a. Acetaminophen
b. La N-acétylcystéine
b. N-acetylcysteine
c. Phenobarbital
c. Le phénobarbital
d. Sodium bicarbonate
d. Le bicarbonate de sodium
209. Dans l'évaluation totale, quel anti-inflammatoire non stéroïdien a In overall assessment, which non-steroidalanti-inflammatory drug
été considéré comme le plus sécurisant ?
has been consideredto be the safest
a.
b.
c.
d.
a. L’Aspirine
b. Indomathacin
c. L'ibuprofène
d. Piroxicam
58
Aspirin
Indomathacin
Ibuprofen
Piroxicam
210. Un patient se présente aux urgences pour surdosage d'un
médicament. Le médecin sait ce qu’était le médicament, et
ordonne un «traitement symptomatique», approprié ainsi que
des doses multiples de N-acétylcystéine. Sur quel médicament le
patient a fait un surdosage ?
A patient presents in the emergency department with an overdose
of a drug. The physician knows what the drug is, and so orders
appropriate “symptomatic and supportive care,” plus multiple
doses of N-acetylcysteine. Upon which drug did the patient
overdose?
a.
b.
c.
d.
a. L'acétaminophène
b. L’Aspirine
c. La Colchicine
Acetaminophen
Aspirin
Colchicine
Diphenhydramine
d. La Diphenhydramine
211. Lequel des énoncés suivants peut mieux soulager la
dysménorrhée occasionnelle?
Which of the following may best relief occasional dysmenorrhea?
a. Over the counter codeine
une.
b. Naproxen
a. la codéine devant le comptoir
c. Acetaminophen
b. Le Naproxen
d. Vitamin E
c. L'acétaminophène
d. La vitamine E
59
212. Une diminution de la formation de l'angiotensine II et une
augmentation de la concentration de bradykinine expliquent
l'action vasodilatatrice des:
A decrease in the formation of angiotensin II and an increase in the
concentration of bradykinin explain the vasodilator action of:
a.
b.
c.
d.
a. Alpha-bloquants
b. bêta-bloquants
Alpha-blockers
Beta-blockers
Angiotensin-converting enzyme inhibitors
Angiotensin receptor blockers
c. Inhibiteurs de l'enzyme de conversion de l'angiotensine
d. Antagonistes des récepteurs de l'angiotensine
213. Lequel des énoncés suivants est un vasodilatateur artériel avec
un effet direct de diminuer la fréquence et la contractilité
cardiaques?
a. Le Diltiazem
b. Le Propranolol
c. La Prazosine
d. Le Captopril
60
Which of the followingis an arterial vasodilator with a direct effect
to decrease heart rate and cardiac contractility?
a.
b.
c.
d.
Diltiazem
Propranolol
Prazosin
Captopril
214. Quel est l'effet négatif potentiel d'une monothérapie avec des
diurétiques thiazidiques, après plusieurs mois de traitement?
What is the potential adverse effect of monotherapy with thiazide
diuretics after several months of treatment?
a. L’Hyperkaliémie
a. Hyperkalemia
b. La Triglycéride et l'hypoglycémie
b. Elevated triglyceride and hypoglycemia
c. l'épuisement de volume et l’insuffisance rénale
c. Volume depletion and renal failure
d. la Rétention de volume compensatoire et l’hypokaliémie
d. Compensatory volume retention and hypokalemia
215. Lequel des diurétiques suivants serait le plus utile dans le
traitement aigu des patients comateux avec une lésion cérébrale
et un œdème cérébral?
Which of the following diuretic would be most useful in the acute
treatment of comatose patient with brain injury and cerebral
edema?
a. L’Acétazolamide
a. Acetazolamide
b. L’Acide Ethacrynic
b. Ethacrynic acid
c. Le Furosémide
c. Furosemide
d. Le Mannitol
d. Mannitol
61
216. Quel traitement antihypertenseur doit être évité en type 1
diabète sucré?
Which antihypertensive therapy should be avoided in type-1
diabetes mellitus?
a. Les Inhibiteurs de l'ECA
a. ACE inhibitors
b. Les Doses élevées de diurétiques
b. High dose diuretics
c. les agents à action centrale
c. centrally-acting agents
d. les bloqueurs des canaux calciques
d. calcium channel blockers
217. Comment les inhibiteurs de l'ECA améliorent-ils
l'hémodynamique cardio-vasculaires?
How do ACE inhibitors improve cardiovascular hemodynamics?
A. Les Inhibiteurs de l'ECA prévient la cardiomyopathie associée
à la grossesse
B. Les Inhibiteurs de l'ECA augmentent les niveaux de la
bradykinine pour inhiber la tachyarythmie
C. Les Inhibiteurs de l'ECA antagonisent les effets
vasoconstricteurs de l'angiotensine à ses récepteurs
D. Les IEC provoquent une dilatation artérielle et veineuse et
limitent le développement de la cardiomyopathie en post
infarctus du myocarde
62
a. ACE inhibitors prevent pregnancy associated
cardiomyopathy
b. ACE inhibitors increase bradykinin levels to inhibit
tachyarrythmia
c. ACE inhibitors antagonize the vasoconstrictive effects of
angiotensin at its receptors
d. ACE inhibitors cause arterial and venous dilation and limit
the development of post myocardial infarction
cardiomyopathy
218. Les effets des diurétiques de l’anse sur l'excrétion ionique Loop diuretics effects on ionic excretion include all of the following
comprennent toutes les caractéristiques suivantes, SAUF:
EXCEPT:
A.
B.
C.
D.
Augmentation de l'excrétion de sodium
Diminution de la perte de magnésium
Augmentation de la perte de calcium
Augmentation de la perte de potassium
a. Increased sodium excretion
b. Decreased magnesium loss
c. Increased calcium loss
d. Increased potassium loss
219. Le mécanisme d'action des diurétiques thiazidiques est:
A.
B.
C.
D.
Mechanism of action of Thiazide diuretics is:
a.
b.
c.
d.
Inhibition du transporteur NaK2Cl
Inhibition de l'anhydrase carbonique
Inhibition du cotransporteur NaCl dans le tubule distal
Un antagoniste de l'aldostérone
220. Le Mécanisme d'action de l'acétazolamide est:
Inhibition of the NaK2Cl transporter
Inhibition of carbonic anhydrase
Inhibition of the NaCl cotransporter in the distal tubule
An antagonist of aldosterone
Mechanism of action of Acetazolamide is:
a.
b.
c.
d.
a. Inhibe le transporteur NaK2Cl
b. Inhibe l'anhydrase carbonique
c. Inhibe le cotransporteur NaCl dans le tubule distal
d. Un antagoniste de l'aldostérone
63
Inhibits the NaK2Cl transporter
Inhibits carbonic anhydrase
Inhibits the NaCl cotransporter in the distal tubule
An antagonist of aldosterone
221. Le Mécanisme d'action du furosémide est:
Mechanism of action of Furosemide is:
a.
b.
c.
d.
a. Inhibe le transporteur NaK2Cl
b. Inhibe l'anhydrase carbonique
c. Inhibe le cotransporteur NaCl dans le tubule distal
Inhibits the NaK2Cl transporter
Inhibits carbonic anhydrase
Inhibits the NaCl cotransporter in the distal tubule
An antagonist of aldosterone
d. Un antagoniste de l'aldostérone
222. Principalement, les décongestionnants nasaux appartiennent
auquel des groupes de médicaments suivants?
Principally, nasal decongestants belong in which of the following
drug groups?
a. Alpha1 agonists
b. Antihistamines
c. Narcotics
d. Antimuscarinic drugs
a. Les agonistes de Alpha1
b. Les antihistaminiques
c. Les Stupéfiants
d. Les Antimuscariniques
223. L'utilisation de décongestionnants nasaux est contre indiquée The use of nasal decongestants is contraindicated when a person is
lorsqu’une personne prenne un des médicaments suivants. taking which of the following drugs?
Lequel ?
a. Antihypertensive drugs
b. MAO inhibitors
a. Les Antihypertenseurs
c. Insulin
b. Les Inhibiteurs de la MAO
d. Nonsteroidal anti-inflammatory analgesic drugs
c. L’Insuline
d. Les Analgésiques anti-inflammatoires non stéroïdiens
64
224. L’Ototoxicité est un effet secondaire unique de quel groupe de Ototoxicity is a unique side effect of which group of diuretics?
diurétiques?
a. Loop diuretics
A. Les diurétiques de l'anse
b. Thiazide diuretics
B. Les diurétiques thiazidiques
c. Potassium sparing diuretics
C. Les Diurétiques épargneurs de Potassium
d. Osmotic diuretics
Les Diurétiques osmotiques
225. Quelle classe des diurétiques est utile dans la réduction de la
pression intraoculaire pendant les crises aiguës de glaucome?
Which class of diuretics is useful in reducing intraocular pressure
during acute attacks of Glaucoma?
a.
b.
c.
d.
a. Les diurétiques de l'anse
b. Les diurétiques thiazidiques
c. Les Diurétiques epargneurs de potassium
Loop diuretics
Thiazide diuretics
Potassium sparing diuretics
Osmotic diuretics
d. Les Diurétiques osmotiques
226. Lequel des laxatifs suivants est contre-indiqué chez les patients
présentant une obstruction intestinale ou de perforation?
Which of the following laxatives is contraindicated in patients with
intestinal obstruction or perforation?
a. Le lavement fleet
a. Fleet enema
b. Les Suppositoires à la glycérine
b. Glycerin suppositories
c. le Lactulose
c. Lactulose
d. Le Psyllium
d. Psyllium
65
227. Parmi la liste suivante, le laxatif avec le plus grand risque d'abus
est:
a. Le Macrogol
Among the following, the laxative with highest risk of abuse is:
a. Macrogol
b. Senna
b. Le Senne
c. Carboxymethylcellulose
c. le Carboxyméthylcellulose
d. Magnesium salts
d. Les sels de magnésium
228. Lequel des énoncés suivants est classé comme un fibrinolytique?
Which of the following is classified as a fibrinolytic?
a.
b.
c.
d.
a. L’Énoxaparine
b. La Warfarine
c. La Ticlopidine
Enoxaparin
Warfarin
Ticlopidine
Alteplase
d. L’Alteplase
229. Comment le sildénafil et les nitrates organiques interagissent How do sildenafil and organic nitrates interact to precipitate severe
pour précipiter l’hypotension sévère?
hypotension?
a. Both agents promote vasodilation.
A. Les deux agents favorisent la vasodilatation.
B. Les deux agents réduisent la contractilité du myocarde.
C. Les deux agents détournent le flux sanguin systémique au
corps caverneux.
D. Les deux agents diminuent le tonus sympathique normal
du système vasculaire systémique.
66
b. Both agents reduce myocardial contractility.
c. Both agents divert systemic blood flow to the corpus
cavernosum.
d. Both agents decrease the normal sympathetic tone of the
systemic vasculature.
230. Comment expliquer l'efficacité de l'eptifibatide dans l'inhibition What accounts for the efficacy of Eptifibatide in inhibiting platelet
de l'agrégation plaquettaire?
aggregation?
A. L'eptifibatide améliore la production de cellules endothéliales
de la prostacycline, qui inhibe l'agrégation plaquettaire.
B. L’Eptifibatide est un antagoniste au récepteur plaquettaire
GPIIb / IIIa, empêchant la liaison du fibrinogène aux
plaquettes.
C. L'eptifibatide est un anticorps monoclonal souris-humain
dirigé contre la thromboxane dérivée des plaquettes,
empêchant son activation de l'agrégation plaquettaire.
L’Eptifibatide catalyse la dégradation du fibrinogène, empêchant
sa liaison aux plaquettes activées.
231. Une femme de 32 ans hypertensive souhaite devenir enceinte.
Son médecin l'informe qu'elle devrait passer à un autre
médicament antihypertenseur. Lequel des médicaments suivants
est contre-indiqué pendant la grossesse absolument?
a. Eptifibatide enhances endothelial cell production of
prostacyclin, which inhibits platelet aggregation.
b. Eptifibatide is an antagonist at the platelet GPIIb/IIIa
receptor, preventing fibrinogen binding to platelets.
c. Eptifibatide is a mouse–human monoclonal anti-body
directed against platelet-derived thromboxane, preventing
its activation of platelet aggregation.
d. Eptifibatide catalyzes the degradation of fibrinogen,
preventing its binding to activated platelets.
A 32-year-old woman with hypertension wishes to become
pregnant. Her physician informs her that she will have to switch to
another antihypertensive drug. Which of the following drugs is
absolutely contraindicated in pregnancy?
a. L’Aténolol
a. Atenolol
b. Le Captopril
b. Captopril
c. La Prazosine
c. Prazosin
d. Le Propranolol
d. Propranolol
67
232. Un homme de 73 ans avec une histoire de chute à la maison se
trouve à avoir de l'hypertension. Lequel des médicaments
suivants est le plus susceptible de provoquer une hypotension
orthostatique et donc un risque accru de chutes?
a. Les Inhibiteurs de l'ECA
A 73 year old man with a history of falling at home is found to have
hypertension. Which of the following drugs is most likely to cause
postural hypotension and thus an increased risk of falls?
a. ACE Inhibitors
b. Alpha blockers
b. Les Alpha-bloquants
c. Beta 1 blockers
c. Les Beta 1 bloquants
d. Angiotensin receptor blockers
d. Les Antagonistes des récepteurs de l'angiotensine
233. Le Vérapamil est associé à qui l'un des suivants?
Verapamil is associated with which one of the following?
a. L’Hypoglycémie
a. Hypoglycemia
b. L’Intervalle PR diminué
b. Decreased PR interval
c. La Constipation
c. Constipation
d. La Tachycardie
d. Tachycardia
68
234. Quel type de traitement serait le plus approprié pour une femme
de 58 ans avec le diabète de type 2 et de cholestérol total de 210
mg / dL, taux de cholestérol HDL de 52 mg / dL, de triglycérides
958 mg / dL, et le cholestérol LDL est de 131 mg / dL?
Which type of therapy would be most appropriate for a 58-year-old
female with type 2 diabetes and total cholesterol 210 mg/dL, HDL
cholesterol 52 mg/dL, triglycerides 958 mg/dL, and LDL cholesterol
131 mg/dL?
a. La Fénofibrate 160 mg par jour
a. Fenofibrate 160 mg daily
b. L’Atorvastatine 40 mg par jour
b. Atorvastatin 40 mg daily
c. La Pravastatine 20 mg par jour
c. Pravastatin 20 mg daily
d. La cholestyramine deux fois par jour
d. Cholestyramine twice daily
235. Une femme de 38 ans avec l’hyperlipidémie se présente à la
clinique de soins primaires à une aggravation des douleurs
musculaires. Son taux de cholestérol total était de 268 mg / dL,
son LDL était de 167 mg / dL, son HDL était de 42 mg / dL, et son
triglycérides était de 254 mg / dL lors de sa dernière visite. La
décision a été prise d'ajouter un autre médicament
hypolipémiant à son simvastatine. Elle n'a jamais eu de douleurs
musculaires avant et se demande si le nouveau médicament est à
l'origine de sa nouvelle douleur. Quel médicament a été
probablement ajouté à la simvastatine?
A 38-year-old woman with hyperlipidemia presents to the primary
care clinic with worsening muscle aches. Her total cholesterol was
268 mg/dL, her LDL was 167 mg/dL, her HDL was 42 mg/dL, and her
triglyceride was 254 mg/dL during her last visit. The decision was
made to add another lipid-lowering medication to her simvastatin.
She never had muscle aches before and is wondering if the new
medication is causing her new pain. What medication was most
likely added to her simvastatin?
a. Cholestyramine
a. La Cholestyramine
b. Colestipol
b. Le Colestipol
c. Ezetimibe
c. L’Ezetimibe
d. Gemfibrozil
d. Le Gemfibrozil
69
236. Le propranolol ne devrait pas être prescrit pour un patient
souffrant d’angine et traité par :
a.
b.
c.
d.
Propranolol should not be prescribed for a patient of angina
pectoris who is already
Nifedipine
Felodipine
Verapamil
Isosorbide mononitrate
receiving
a.
b.
c.
d.
237. Dans le traitement du choc cardiogénique, le médicament de
choix est :
a.
b.
c.
d.
In the treatment of cardiac shock, the drug of choice is
a.
b.
c.
d.
Dopamine
Propranolol
Phenoxybenzamine
Metaraminol
238. Quel médicament diminue la tension artérielle et inverse
l’hypertrophie cardiaque ?
abcd-
Nifedipine
Felodipine
Verapamil
Isosorbide mononitrate
Dopamine
Propranolol
Phenoxybenzamine
Metaraminol
Which drug besides lowering blood pressure arrests and even
reverses cardiac hypertrophy
Les IEC
Les beta bloquants
Les inhibiteurs calciques
Les diurétiques
a.
b.
c.
d.
70
ACE inhibitors
Betablockers
Calcium channel blockers
Diuretics
239. Un patient sous chimiothérapie vomit fréquemment ; quel
médicament serait utile dans son cas ?
a.
b.
c.
d.
A patient undergoing cancer chemotherapy is vomiting frequently.
A drug that might help in this situation is:
Bromocriptine
Cimetidine
Loratadine
Ondansetron
a.
b.
c.
d.
240. En urgence, le traitement de premier choix pour l’exacerbation
de l’asthme est :
a.
b.
c.
d.
La Theophylline
β-agoniste
Gluccorticoide
Antihistaminique
abcd-
In the emergency department, the preferred first-line therapy for
asthma exacerbation is:
a.
b.
c.
d.
241. L’effet indésirable commun lié à la dose du salbutamol est :
Bromocriptine
Cimetidine
Loratadine
Ondansetron
Theophyllin
β-agonist
Corticosteroid
Antihistamine
The common and dose related side effect of salbutamol is
La diminution de la tension artérielle
Les tremblements
L’hyperglycémie
La stimulation du système nerveux central
a.
b.
c.
d.
Decrease in blood pressure
Muscle tremor
Hyperglycaemia
Central nervous system stimulation
242. Lequel des médicaments anti diarrhéiques suivants délivrés sans Which of the following anti diarrheal OTC drug is related chemically
prescription est lié chimiquement semblable à la mépéridine, un to the strong opioid-analgesic meperidine?
opioïde analgésique puissant ?
a. Aluminum hydroxide
a. Hydroxide d’Aluminum
b. Diphenoxylate
b. Diphenoxylate
c. Loperamide
c. Loperamide
d. Magnesium hydroxide
d. Hydroxide de Magnesium
71
243. Quelle information concernant les suppléments calcique est
VRAIE ?
Which of the following counseling information about calcium
supplements is TRUE?
a- Peuvent être pris avec les autres médicaments sans
problème puisqu’il n’y a pas d’interaction
b- Les suppléments calciques devraient être pris en doses
multiples pour améliorer l’absorption
c- Vous pouvez prendre les suppléments avec fibres de
façon concomitante puisqu’il n’y a pas d’interaction
d- Un effet indésirable commun est la diarrhée
e-
a. You can take the supplements with other drugs since there
is no interaction
b. Calcium supplements should be taken in divided doses not
as one dose to increase absorption
c. You can take the supplements with fibers concurrently since
there is no interaction
d. Common adverse effects include: diarrhea
244. Lequel est un conseil Durant la prise d’insuline intermédiaire ?
abcd-
Bien agiter le flacon avant utilisation
Injecter l’insuline à n’importe quel moment de la journée
Injecter l’insuline en IV seulement
Garder les flacons dans la poche
Which of the followings is counseling information about the
intermediate acting insulin?
a. Shake well the vial before use
b. Inject insulin at any time throughout the day
c. Inject insulin as intravenous injection only
d. Keep the vials in their pockets
72
245. Lequel parmi les énoncés suivants concernant les suppléments Which of the following statement regarding Vitamin D supplements
de Vitamine D est VRAI?
is TRUE?
abcd-
Il n’y a qu’une seule forme disponible de la vitamine D
La vitamine D ne peut pas être prise avec le calcium
Donnée seulement pour les personnes âgées
Donnée pour la prévention et le traitement de
l’ostéoporose
b. There is only one form of vitamin D available
c. Vitamin D cannot be taken with Calcium
d. Given only to elderly patients
e. Given for osteoporosis prevention and treatment
246. Lequel parmi les énoncés suivants concernant les antidiabétiques Which of the following statements regarding sulfonylurea is
sulfamides est CORRECT ?
CORRECT?
a- Disponibles en formulation parentérale
b- Peuvent augmenter le risque d’hypoglycémie et de gain
de poids
c- Peuvent être pris au coucher
d- Ne peuvent être associés à d’autres thérapies
a. Available in parenteral formulation
b. Might induce the risk of hypoglycemia and weight gain
c. Must be taken at bed time
d. Cannot be combined with other therapies
73
247. La carrière musicale d’un homme violoniste talentueux de 21 ans,
est en difficulté car il devient anxieux, souffrant de palpitations et
de tremblements lorsqu’on lui demande de jouer devant une
audience. Quel médicament peut réduire ses symptômes ?
A talented 21-year-old violinist’s musical career is in trouble
because he becomes acutely anxious, suffering from tremors and
palpitations, whenever he is asked to play in front of an audience.
What medication can help him in reducing his symptoms?
a. Desipramine
a. Desipramine
b. Citalopram
b. Citalopram
c. Risperidone
c. Risperidone
d. Propranolol
d. Propranolol
248. Une femme de 56 ans, diagnostiquée depuis l’âge de 20 ans
comme ayant une schizophrénie paranoïaque, a reçu des
antipsychotiques typiques pour plusieurs années. Depuis deux
ans, elle a commencé à souffrir de dyskinésie tardive. Que faut-il
faire à ce sujet?
A 56-year-old woman who was diagnosed with paranoid
schizophrenia in her early twenties has received daily doses of
various typical antipsychotics for many years. For the past 2 years,
she has had symptoms of tardive dyskinesia. Which of the following
actions should be taken next?
a. Start the patient on benzatropine
abcd-
Commencer la benzatropine chez la patiente
Commencer le propranolol chez la patiente
Commencer la diphenhydramine chez la patiente
Passer à la clozapine chez la patiente
b. Start the patient on propranolol
c.
Start the patient on diphenhydramine
d. Switch the patient to clozapine
74
249. Un homme de 72 ans commence à ressentir une rétention
urinaire aiguë et une vision floue après la prise d’antidépresseur
pour 5 jours. Quel médicament en est probablement la cause ?
A 72-year-old man develops acute urinary retention and blurred
vision after taking an antidepressant for 5 days. Which of the
following medications is most likely to cause such side effects?
a. Venlafaxine
a. Venlafaxine
b. Bupropion
b. Bupropion
c. Escitalopram
c. Escitalopram
d. Amitriptyline
d. Amitriptyline
250. Lequel parmi les suivants peut causer nausée et vomissement si
ingéré avec de l’alcool ?
Which of the following drugs can cause nausea & vomiting if
ingested with alcohol?
a. Disulfiram
a. Disulfiram
b. Methadone
b. Methadone
c. Lorazepam
c. Lorazepam
d. Naltrexone
d. Naltrexone
75
251. TM est un homme de 78 ans, récemment diagnostiqué de
maladie d’Alzheimer. Son médecin explique à la famille qu’il
aurait besoin d’un médicament qui augmenterait l’acétylcholine
dans le cerveau pour améliorer son état cognitif. Lequel parmi
ces médicaments serait adéquat ?
TM is a 78 year old, newly diagnosed Alzheimer’s disease patient.
His clinician explained to his family that they need to give the
patient a medication that will increase acetylcholine in the brain to
improve his cognition. Which of the following medications might
have been prescribed for TM?
a. Donepezil
a. Donepezil
b. Clozapine
b. Clozapine
c. Glatiramer Acetate
c. Glatiramer Acetate
d. Desipramine
d. Desipramine
252. Une femme de 42 ans est traitée pour épilepsie focale depuis 8
ans. Elle se présente à l’officine à cause de l’augmentation de la
taille de ses gencives et de hirsutisme sur son visage. Quel
médicament cette femme prend-elle probablement ?
a. Acide Valproique
A 42-year-old woman, with a history of epilepsy is being treated
with an antiepileptic drug for focal seizures since 8 years. She is
presenting to your pharmacy today because she is experiencing an
increase in the size of her jaw over the past years and production
of hair over her face. Which of the following might be the drug that
this lady has been taking?
b. Phenytoine
a. Valproic Acid
c. Gabapentine
b. Phenytoin
d. Lamotrigine
c. Gabapentin
d. Lamotrigine
76
253. Une femme déprimée de 52 ans, se présente pour une
exacerbation de sa dépression parce qu’elle a été diagnostiquée
de diabète et elle souffre de nouvelle neuropathie douloureuse
au niveau des jambes. De plus, elle est en ménopause et souffre
de bouffées de chaleur. Quel médicament peut traiter ses
problèmes psychiatriques et médicaux concomitants ?
a.
b.
c.
d.
Escitalopram
Duloxetine
Trazodone
Vilazodone
A depressed 52-year-old woman presents with a worsening of
depression after she was informed she has diabetes, and she has a
new onset of painful neuropathy in her legs. Furthermore, she is
postmenopausal and suffering immensely from vasomotor hot
flashes. What treatment may best target her psychiatric and
medical comorbidities?
a.
b.
c.
d.
254. TE, une femme de 43 ans, qui travaille en tant qu’enseignante
dans une école, se présente à son médecin pour difficultés à
l'initiation du sommeil. Elle a maintenant beaucoup de pensées la
nuit et est «incapable de stopper son cerveau." Elle a été
diagnostiquée pour insomnie. Lequel des médicaments suivants à
prescrire, peut être indiqué pour son état?
Escitalopram
Duloxetine
Trazodone
Vilazodone
TE, A 43-year-old woman, who works as a teacher at a school,
presents to her doctor complaining of difficulties with sleep
initiation. She now has many thoughts at night and is "unable to
shut her mind off." She has been diagnosed with insomnia. Which
of the following prescription medication is indicated for her
condition?
a. Zolpidem
a. Zolpidem
b. Butalbital
b. Butalbital
c. Valerian
c. Valerian
d. Diphenhydramine
d. Diphenhydramine
77
255. Lequel des médicaments suivants est le plus approprié pour le Which of the following medication is the most appropriate
traitement d’un garçon de 10 ans diagnostique ayant des treatment for a 10 years old by diagnosed with absence seizure?
convulsions d’absence?
a. Phenytoin
a. Phenytoin
b. Carbamazepine
b. Carbamazepine
c. Ethosuximide
c. Ethosuximide
d. Clobazam
d. Clobazam
256. Lequel parmi les suivants n’est pas un antitussif ?
Which of the following is NOT an antitussive?
a. Codeine
a. Codeine
b. Butamirate
b. Butamirate
c. Dextromethorphan
c. Dextromethorphan
d. Guaifenesine
d. Guaifenesin
257. Lequel parmi les suivants est classifié comme pediculicide ?
Which of the following is classified as a pediculicide?
a. Amoxicillin
a. Amoxicillin
b. Lindane
b. Lindane
c. Fluconazole
c. Fluconazole
d. Benzyl alcohol
d. Benzyl alcohol
78
258. Lequl parmi les suivant n’est PAS une composante d’une solution
de rehydratation orale ?
Which of the following is NOT a component of an oral rehydration
solution?
a. Sodium
a. Sodium
b. Chlorure
b. Chloride
c. Calcium
c. Calcium
d. Potassium
d. Potassium
259. Quel médicament n’est PAS considéré comme décongestionnant
oculaire ?
A. Naphazoline
B. Tetrahydrozoline
C. Oxymetazoline
D. Antazoline
Which of the following drugs is not considered an ophthalmic
decongestant?
a. Naphazoline
b. Tetrahydrozoline
c. Oxymetazoline
d. Antazoline
260. Lequel parmi les suivants traite efficacement et sans effet
indésirable un bouchon de cerumen ?
Which of the following is used safely and effectively for excessive
impacted cerumen in the middle ear?
a. Neomycine
a. Neomycin
b. Peroxyde de Carbamide
b. Carbamide peroxide
c. Docusate de calcium
c. Docusate calcium
d. Huile de ricin
d. Castor oil
79
261. Quel antiplaquettaire recommandez-vous à un patient souffrant
d’angine ?
a. Warfarin
a. Warfarin
b. Acenocoumarol
b. Acenocoumarol
c. Heparin
c. Heparin
d. Aspirin
d. Aspirin
262. Que recommandez-vous pour le traitement de l’hypertension
d’un homme de 55 ans ayant une clairance de créatinine de
50ml/min ?
abcd-
Which of the following antiplatelet would you consider
recommending for patients with angina?
Which of the following would you recommend for the treatment of
hypertension in a 55 year old male with chronic kidney disease
having a CrCl of 50 ml/min?
IEC
Diurétique thiazidique
Beta bloquant
Antyagoniste calcique
a. ACE inhibitor
b. Thiazide diuretic
c. Beta Blocker
d. Calcium channel blocker
263. Lequel parmi les suivants recommandez-vous pour réduire le LDL
de 55% ?
Which of the following would you consider recommending for a
patient when you anticipate a decrease of 55% in LDL cholesterol
a. Ezetimibe
a. Ezetimibe
b. Cholestyramine
b. Cholestyramine
c. Rosuvastatin
c. Rosuvastatin
d. Niacin
d. Niacin
80
264. En conseillant un patient concernant le chewing-gum à la
nicotine, quel est le plus convenable ?
In counseling patients on the proper use of the nicotine gum, what
would be the most appropriate advice?
a- Utiliser la gomme avec les aliments et les boissons
b- Continuez de mâcher la gomme jusqu’à la disparition de
l’envie de fumer
c- Mâcher la gomme jusqu’à l’apparition d’un goût de
poivre ou de menthe
d- Avaler la gomme de nicotine pour que la nicotine soit
bien absorbée.
a. Use the gum with food and beverages
b. Continue chewing a piece of gum until the craving for
cigarettes stops
c. Chew the gum slowly until a peppery or minty taste
emerges
d. Swallow the gum for nicotine to be absorbed
265. Quel médicament recommandez-vous pour un enfant
asthmatique de 7 ans, sachant que la mère préfère la voie orale ?
a. Budesonide
Which of the following medications can you recommend for a 7
years old child with asthma knowing that the mother prefers a
medication that can be administered orally?
a. Budesonide
b. Fluticasone
b. Fluticasone
c. Montelukast
c. Montelukast
d. Cromolyn
d. Cromolyn
81
266. Lequel parmi les suivants recommandez-vous pour un patient
souffrant de reflux gastro-eosophagien sévère ?
abcd-
Which of the following would you recommend in a patient with
severe GERD?
Carbonate de calcium
Esomeprazole
Cimetidine
Sucralfate
a. Calcium carbonate
b. Esomeprazole
c. Cimetidine
d. Sucralfate
267. Lequel parmi les suivants recommandez-vous en cas de diarrhée
non infectieuse ?
Which of the following would you recommend in non-infective
acute diarrhea?
a. Cholestyramine
a. Cholestyramine
b. Loperamide
b. Loperamide
c. Psyllium
c. Psyllium
d. Diphenoxylate
d. Diphenoxylate
268. Les IEC et ARA traitent l’hypertension de façon similaire et ont
presque les mêmes effets indésirables. Lequl parmi les suivants
est un effet indésirable des IEC seulement?
A.
B.
C.
D.
Vertige
Toux sèche
Dysfonctionnement érectile
Hypotension
82
ACE inhibitors and ARBs treat hyper-tension in a similar fashion and
have similar side effects. Which of the following is a side effect of
ACE inhibitors only?
A.
Dizziness
B.
Dry cough
C.
Erectile dysfunction
D.
Hypotension
269. Lequel parmi les suivants est un inhibiteur calcique agissant sur
les muscles lisses de la paroi vasculaire essentiellement ?
A.
Amlodipine
B.
Diltiazem
C.
Losartan
D.
Nitroprusside
Which of the following is a calcium channel blocker that works
primarily on vascular smooth muscle?
A.
Amlodipine
B.
Diltiazem
C.
Losartan
D.
Nitroprusside
270. Quel est le mécanisme d’action de l’isosorbide dinitrate ?
What is the mechanism of action of isosorbide dinitrate?
a- Bloque l’activité de l’angiotensine II au niveau de ses
récepteurs
b- Cause une élimination des fluides en excès
c- Augmente l’inotropie cardiaque
d- Réduit la précharge
a. Blocks the activity of angiotensin II at its receptor
b. Causes excess fluid elimination
c. Increases cardiac inotropy
d. Reduces preload
271. Un homme de 57 ans avec insuffisance cardiaque et
hypertension, est maintenu sous digoxine et diurétique. Quels
indicateurs sanguins sont importants pour faire le suivi du patient
et éviter les effets indésirables sévères ?
A 57-year-old man with congestive heart failure and hypertension
is maintained on digoxin and diuretic. Which of the following blood
indicators is most important to follow in this patient to avoid
severe side effects?
a. Bicarbonate
a. Bicarbonate
b. Chloride
b.
c. Glucose
c. Glucose
d. Potassium
d. Potassium
83
Chloride
272. Un homme hypertendu de 61 ans développe une fibrillation
auriculaire. Son traitement inclut simvastatine et metoprolol. Son
médecin lui prescrit un anticoagulant agissant directement sur la
thrombine. Quel est ce medicament?
A 61-year-old man with hypertension develops atrial fibrillation. His
medications include simvastatin and metoprolol. His physician
prescribes an anticoagulant for clot prophylaxis, which directly
inhibits thrombin. Which drug is this?
a. Aspirin
b. Dabigatran
c. Heparin
d. Warfarin
273. Un homme de 58 ans se présente aux urgences après une perte
soudaine du contrôle moteur côté gauche avec aphasie. Le temps
d’arriver aux urgences, les symptômes étaient déjà révolus. On
lui prescrit comme antiagrégant un inhibiteur des récepteurs
plaquettaires à l’ADP. De quel médicament s’agit-il ?
a. Aspirin
b. Dabigatran
c. Heparin
d. Warfarin
A 58-year-old man presents to the emergency department (ED)
after experiencing a sudden loss of motor control on his left side
accompanied by aphasia. By the time he reached the ED, his
symptoms had already resolved. He is given a prescription for a
drug that binds platelet ADP receptors to prevent their
aggregation. Which drug is this?
a. Clopidogrel
b. Enoxaparin
c. Eptifibatide
d. Tirofiban
a. Clopidogrel
b. Enoxaparin
c. Eptifibatide
d. Tirofiban
The mechanism of action of furosemide is in which of the following
locations?
274. Où agit le furosémide ?
abcd-
La partie ascendante de l’anse de Henle
Les tubules collecteurs
Les tubules distaux
Les tubules proximaux
a.
b.
c.
d.
84
Ascending limb of the loop of Henle
Collecting duct
Distal tubule
Proximal tubule
275. Quel est le mécanism d’action de l’Alteplase ?
abcd-
What is the mechanism of action of alteplase?
Se lie au récepteur glycoprotéique IIb/IIIa
Bloque les récepteurs à l’ADP
Convertit la plasminogène en plasmine
Inhibe la COX1 et COX2
a.
b.
c.
d.
276. Un homme de 56 ans se présente chez le médecin avec une
difficulté d’uriner ; au toucher rectal, une prostate hypertrophiée
est retrouvée. Il est traité par terazosine, ce qui améliore ses
symptômes de façon dramatique. Quel effet indésirable est-il
possible qu’il ressente ?
abcd-
Diarrhée
Vertige
Ballonnement
Céphalées
What is the mechanism of action of Spironolactone?
Antagoniste des récepteurs de l’aldostérone
Inhibiteur de l’anhydrase carbonique
Inhibiteur de la réabsorption de NaCl
Inhibe le cotransporteur au Na+/K+/2Cl-
a. Aldosterone receptor antagonist
b. Carbonic anhydrase inhibitor
c. Inhibits NaCl reabsorption
d. Inhibits Na+/K+/2Cl- cotransport
278. Comment agit la digoxine dans l’insuffisance cardiaque ?
abcd-
A 56-year-old man presents to his primary care physician
complaining of difficulty urinating. Digital rectal exam reveals an
enlarged prostate. The patient is started on terazosin, after which
his symptoms improve dramatically. Which of the following side
effects he is most likely to experience?
a. Diarrhea
b. Dizziness
c. Flatulence
d. Headache
277. Quel est le mécanisme d’action de la spironolactone ?
abcd-
Binds to glycoprotein receptor IIb/IIIa
Blocks ADP receptors
Converts plasminogen to plasmin
Inhibits COX-1 and COX-2
How does digoxin help in heart failure?
Bloque l’activité de l’angiotensine II et son récepteur
Cause l’élimination de l’excès des fluides
Augmente l’inotropie cardiaque
Réduit la précharge
a. Blocks the activity of angiotensin II at its receptor
b. Causes excess fluid elimination
c. Increases cardiac inotropy
d. Reduces preload
85
279. Quel est le mécanisme d’action de la prazosine ?
What is the mechanism of action of prazosin?
a- Antagonise les récepteurs alpha 2 adrénergiques
b- Inhibe la 5 alpha réductase
c- Inhibe la synthèse de la testostérone
a.
b.
c.
d.
Antagonizes alpha2-adrenergic receptors
Inhibits 5-alpha-reductase
Inhibits testosterone synthesis
Relaxes prostatic smooth muscle
d- Relaxe les muscles lisses prostatiques
280. Lequel parmi les suivants bloque la synthèse de novo du Which of the following drugs blocks de novo cholesterol synthesis
cholestérol au niveau des hépatocytes ?
in hepatocytes?
a. Cholestyramine
b. Colesevelam
c. Ezetimibe
d. Rosuvastatin
281. Une femme de 37 ans a une hyperlipidémie est traitée pour
diminuer les taux de triglycérides et de cholestérol sanguins. Elle
pense arrêter le traitement à cause d’un rash cutané au niveau
du cou et visage, avec rougeur et démangeaisons. Quel
médicament prend-elle probablement ?
a. Atorvastatine
b. Fenofibrate
c. Gemfibrozil
d. Acide Nicotinique
e.
a. Cholestyramine
b. Colesevelam
c. Ezetimibe
d. Rosuvastatin
A 37-year-old woman with hyperlipidemia is taking a drug to lower
her triglyceride and blood cholesterol levels. She is considering
stopping her therapy, however, because of a red, itchy rash on her
face and neck that occurs following some doses. Which drug is she
taking?
a. Atorvastatin
b. Fenofibrate
c. Gemfibrozil
d. Nicotinic acid
86
282. Un homme dyslipidémique de 43 ans, sous Atorvastatine, a un
profile lipidique non contrôlé. Le médecin lui prescrit du
Colesevelam comme traitement adjoint. Quel changement est
alors attendu avec ce nouveau traitement ?
A 43-year-old man with dyslipidemia comes to the clinic for a
routine checkup. He currently takes Atorvastatin but is not
achieving his target lipid profile. The physician prescribes
Colesevelam to help. Which of the following changes is expected as
a result of this new therapy?
a.
b.
c.
d.
a- Diminution du HDL
b- Diminution des triglycérides
c- Hyperglycémie
Decreased HDL
Decreased triglycerides
Hyperglycemia
Increased triglycerides
d- Augmentation des triglycérides
283. Quel est le mécanisme d’action de glimepiride ?
What is the mechanism of action of glimepiride?
a.
b.
c.
d.
a- Réduit la libération du glucagon
b- Augmente la libération d’insuline
c- Augment la sensibilité à l’insuline au niveau des tissus
périphériques
Decreased glucagon release
Increased insulin release
Increased insulin sensitivity in peripheral tissues
Inhibits intestinal brush border enzymes
d- Inhibe les enzymes des cellules intestinales à bordure en
brosse
284. Lequel des médicaments suivants, bloque les enzymes des Which of the following drugs blocks brush border enzymes to
bordures en brosse pour inhiber l’absorption intestinale du inhibit intestinal absorption of cholesterol?
cholestérol ?
a. Cholestyramine
b. Colesevelam
a. Cholestyramine
c. Rosuvastatin
b. Colesevelam
d. Ezetimibe
c. Rosuvastatin
d. Ezetimibe
87
285. Une femme de 43 ans, avec diabète type 2, a pris de l’insuline
avec les repas ainsi que la metformine. Sa glycémie est toujours
mal contrôlée. Son médecin lui prescrit un médicament
supplémentaire, qui est un analogue d’un peptide endogène qui
augmente la sécrétion d’insuline. Quel est ce médicament ?
a. Exenatide
b. Glipizide
c. Miglitol
d. Rosiglitazone
e.
Un
homme de 57 ans, hyperlipidémique, se présente aux
286.
urgences avec douleur sévère au niveau du gros orteil droit. La
douleur est à 10/10 et s’aggrave avec le contact. L’aspiration au
niveau de l’articulation confirme la présence de goutte. Il avait un
épisode de goute il y a dix ans. Quel hypolipémiant pris par le
patient pourrait mener à cet effet ?
a. Cholestyramine
b. Ezetimibe
c. Gemfibrozil
d. Niacin
287. Lequel des suivants prévient la sécrétion acide en antagonisant
un récepteur ?
a- Hydroxyde d’aluminium
b- Fexofenadine
c- Omeprazole
d- Ranitidine
A 43-year-old woman with Type-2 diabetes has been taking insulin
with meals as well as metformin. Her blood glucose remains poorly
controlled. Her doctor prescribes an additional drug, which is an
analog of an endogenous peptide that enhances insulin secretion.
What drug is this?
a. Exenatide
b. Glipizide
c. Miglitol
d. Rosiglitazone
A 57-year-old man with hyperlipidemia presents to the emergency
department with severe pain in his first right toe. The pain is a
10/10 and worse with any contact. Joint aspiration confirms gout.
He had an episode of gout about 10 years ago. What medication
used to treat his hyperlipidemia may have led to his gout flare?
a. Cholestyramine
b. Ezetimibe
c. Gemfibrozil
d. Niacin
Which of the following drugs prevents acid secretion by
antagonizing a receptor?
a. Aluminum hydroxide
b. Fexofenadine
c. Omeprazole
d. Ranitidine
88
288. Une femme obèse de 33 ans se plaint de douleurs à la poitrine en
post-prandial. Son médecin lui prescrit du pantoprazole; ce
dernier peut retarder l’absorption d’un des nutriments suivants.
Lequel ? :
A 33-year-old woman who is obese complains of post-prandial,
burning chest pain. Her physician prescribes pantoprazole.
Pantoprazole may impair absorption of which of the following
nutrients?
a. Vitamin B12
b. Vitamin E
c. Vitamin A
d. Vitamin D
289. Une femme enceinte de 26 ans, 33 semaines de gestation, se
présente aux urgences avec douleur et œdème de son mollet
droit. Un scan duplex confirme la présence de thrombose
veineuse profonde. Quel est le traitement le plus approprié à
prescrire pour le reste de la grossesse ?
a. Vitamin B12
b. Vitamin E
c. Vitamin A
d. Vitamin D
A 26-year-old woman at 33 weeks gestation presents to the
emergency department with pain and swelling in her right calf. A
duplex of the right calf confirms the presence of a deep vein
thrombosis (DVT). What is the most appropriate treatment for the
rest of her pregnancy?
a. Abciximab
a. Abciximab
b. Aspirine
b. Aspirin
c. Heparine
c. Heparin
d. Warfarine
d. Warfarin
290. Lequel des médicaments suivants diminue le nombre d’attaques Which of the following drugs would decrease the amount of
asthmatiques en prévenant un dérivé de l’acide arachidonique de asthma attacks by preventing an arachidonic acid derivative from
se lier sur son récepteur ?
binding to its receptor?
a.
b.
c.
d.
Aspirin
Celecoxib
Ipratropium
Montelukast
a.
b.
c.
d.
89
Aspirin
Celecoxib
Ipratropium
Montelukast
291. Une fille de 15 ans se présente à son médecin de soins primaire
en se plaignant de rhinorrhée et d’irritation au niveau des yeux.
Elle avoue que les premiers symptômes apparaissaient au
printemps il a quelques années, mais se répètent chaque
printemps et deviennent de plus en plus gênants. Quel
médicament parmi les suivants empêcherait la libération du
médiateur chimique majeur dans son cas ?
A. Cromoglycate de sodium
B. Diphenhydramine
C. Ranitidine
D. Theophylline
292. Un homme de 33 ans avec une histoire d’asthme se présente aux
urgences après une piqure d’abeille. Le médecin traitant prescrit
de l’adrenaline en sous-cutané. Quelle est parmi les suivant la
contrindication à noter pour ce médicament ?
A. Diabète
B. Glaucome à angle fermé
C. Insuffisance pulmonaire
Dysthyroïdie
A 15-year-old female presents to her primary care physician
complaining of runny nose and itchy eyes. She said that she first
had these symptoms during the spring a few years ago, but each
year, they have been bothering her more. Which of the following
drugs would prevent the release of the main chemical mediator in
her case?
a.
b.
c.
d.
Cromolyn sodium
Diphenhydramine
Ranitidine
Theophylline
A 33-year-old man with a history of asthma comes into the
emergency department after getting stung by a bee. The treating
physician wants to give epinephrine subcutaneously. Which of the
following contra-indications to this medication is noted?
a.
b.
c.
d.
Diabetes mellitus
Narrow-angle glaucoma
Pulmonary failure
Thyroid disease
293. JG est un patient ayant une réaction allergique immédiate au JG is a patient with an immediate allergic reaction to
ticarcilline/clavulanate. Quel autre médicament pourrait-il ticarcillin/clavulanate. Select the antimicrobial that JG may take
prendre sans risque de réaction allergique ?
without risk of allergic reaction.
A.
B.
C.
D.
Piperacillin/tazobactam
Amoxicillin/clavulanate
Ceftriaxone
Aztreonam
a.
b.
c.
d.
90
Piperacillin/tazobactam
Amoxicillin/clavulanate
Ceftriaxone
Aztreonam
294. Lequel des carbapenems suivants n’agit pas contre le Which of the following carbapenem antibiotics does not have
Pseudomonas aeruginosa ?
coverage against P. aeruginosa?
A.
B.
C.
D.
Ertapenem
Doripenem
Imipenem-cilastatin
Meropenem
a.
b.
c.
d.
Ertapenem
Doripenem
Imipenem-cilastatin
Meropenem
295. Quel agent est recommandé pour un homme de 25 ans, suspecté Which agent is recommended for a 25-year old man with suspected
d’avoir une pneumonie acquise en communauté, et ayant Community acquired pneumonia (CAP) who completed a 5 day
compléter lors d’une ancienne pneumonie il y a 6 semaines un course of Clarithromycin for suspected CAP 6 weeks ago?
traitement par clarithromycine pendant 5 jours ?
a. Clarithromycin 1,000 mg/day po
b. Levofloxacin 750 mg/day po
c. Piperacillin-tazobactam 4.5 g IV q6h
A. Clarithromycin 1,000 mg/jour po
d. Doxycycline 100 mg po bid
B. Levofloxacin 750 mg/jour po
C. Piperacillin-tazobactam 4.5 g IV q6h
D. Doxycycline 100 mg po bid
296. Lequel parmi ces agents n’est PAS recommandé pour les Which of the following agents is not recommended for respiratory
infections respiratoires où une infections à SAMR est suspectée ? tract infections with possible MRSA infections?
A.
B.
C.
D.
Ceftaroline
Daptomycin
Linezolid
Vancomycin
a.
b.
c.
d.
91
Ceftaroline
Daptomycin
Linezolid
Vancomycin
297. Un médicament est administre sous forme de prodrogue. Ce
dernier augmente l’expression des cytochromes p-450
convertissant la prodrogue en forme active. Quel effet sera
observé lors d’une administration chronique de long court ?
A.
B.
C.
D.
La puissance/dose va diminuer
La puissance/dose va augmenter
L’efficacité va diminuer
L’efficacité va augmenter
A drug is administered in the form of an inactive pro-drug. The prodrug increases the expression of a cytochrome P-450 that converts
the pro-drug to its active form. With chronic, long-term
administration of the pro-drug, which of the following will be
observed?
A. The potency will decrease
B. The potency will increase
C. The efficacy will decrease
D. The efficacy will increase
What is the mechanism of action of Warfarin?
298. Quel est le mécanisme d’action de la warfarine ?
a- Augmente le niveau plasmatique du facteur IX
b- Inhibe la thrombine et les étapes préliminaires de la
coagulation
c- Inhibe la synthèse de prothrombine et des facteurs VII, IX
et X de la coagulation
d- Active la plasminogène
299. Pour un médicament, la pharmacocinétique réfère à la relation :
A. Dose-concentration plasmatique
B. Concentrations-effet du médicament
C. Dose-clairance du médicament au niveau des organes
d’élimination
D. Dose-effet du médicament
a. Increase in the plasma level of factor IX
b. Inhibition of thrombin and early coagulation steps
c. Inhibition of synthesis of prothrombin and coagulation
factors VII, IX, and X
d. Activation of plasminogen
Pharmacokinetics refers to the relationship of drug
a.
b.
c.
d.
92
Dose to drug concentration in plasma
Concentrations to drug effect
Dose to drug clearance at the eliminating organs
Dose to drug effect
300. Lequel parmi ces facteurs n’affecte PAS l’absorption d’un Which one from the following factors will not affect drug
médicament ?
absorption?
A.
B.
C.
D.
La motilité gastrique
Le flux sanguin
L’ingestion d’aliments
La demi-vie d’élimination du médicament
a. Gastric motility
b. Blood flow
c. Food intake
d. Drug elimination half-life
301. Un élève de 6 ans à l'école élémentaire, vu en clinique, se trouve A 6-year-old elementary school student seen in the clinic is found
avec une infestation de poux du cuir chevelu. Les mesures visant to have lice infestation of the scalp. Measures to eradicate the
à éradiquer l'infection chez ce patient et le reste de la famille infection in the patient and the rest of the family include:
comprennent:
a. Calamine lotion with 1% menthol and sterilization of all
bedding and clothes
A. Lotion calamine avec 1% de menthol et stérilisation de
b.
Sulfur ointment and sterilization of all bedding and clothes
toute la literie et les vêtements
c. Permethrin 1% and sterilization of all bedding and clothes
B. Pommade de Soufre et stérilisation de toute la literie et
d. Crotamiton and sterilization of all bedding and cloths
les vêtements
C. Perméthrine 1% et stérilisation de toute la literie et les
vêtements
D. Crotamiton et stérilisation de toute la literie et les
vêtements
302. Un patient prend une dose potentiellement mortelle de A patient took a potentially lethal overdose of aspirin. What drug
l'aspirine. Quel médicament adjoint serait-il utile, si risque vital, would be a helpful, if not lifesaving, adjunct to manage this severe
pour gérer cette intoxication grave à l'aspirine?
aspirin poisoning?
A.
B.
C.
D.
Acétaminophène
N-acétylcystéine
Phénobarbital
Bicarbonate de sodium
A.
B.
C.
D.
93
Acetaminophen
N-acetylcysteine
Phenobarbital
Sodium bicarbonate
303. Lequel parmi les suivants est un agent approprie et une dose de
départ pour un patient avec une hypertension non compliquée ?
A. Atenolol 25 mg une fois/jour
B. Atenolol 100 mg une fois/jour
C. Propranolol 80 mg 2 fois/jour
Propranolol 120 mg 2 fois/jour
A. Atenolol 25 mg once daily
B. Atenolol 100 mg once daily
C. Propranolol 80 mg twice daily
D. Propranolol 120 mg twice daily
304. L’information la plus importante à un patient mis sous prazocine
est :
A. Faire attention au développement d’une éruption
cutanée
B. Faire attention à une toux sèche
C. Prendre la première dose au coucher
Ne pas prendre le médicaments en même temps que les
antiacides
The most important information to tell a patient started on
prazosin is to
A. watch out for the development of a rash.
B. watch out for a dry cough.
C. take the first dose at bedtime.
D. Do not take the drug at the same time as antacids.
305. Une femme de 22 ans souffrant d’asthme est traitée par
salbutamol par inhalation. Ce dernier est un bronchodilatateur
ayant un poids moléculaire de 239 Daltons. Lequel des processus
de passage suivants, explique au mieux le transfert du
médicament à travers la muqueuse bronchique?
A.
B.
C.
D.
Which of the following is an appropriate agent and starting dose
for a patient with uncomplicated hypertension?
Diffusion aqueuse
Diffusion lipidique
Diffusion facilitée
Transport actif
A 22-year-old woman suffering from asthma was prescribed
albuterol by inhalation. Albuterol is a bronchodilator with a
molecular weight of 239 Daltons. Which of the following transport
mechanisms most likely accounts for the transfer of the drug
through the bronchial mucosa?
A. Aqueous diffusion
B. Lipid diffusion
C. Facilitated diffusion
D. Active transport
94
306. Un homme de 55 ans connu ayant un diabète, se présente à son
médecin pour un suivi. Il a une histoire de tension élevée, et
aujourd’hui sa tension moyenne mesurée est de 156/95 mmHg.
Lequel parmi les suivants devrait être recommandé comme
premier choix selon JNC8 ?
A.
B.
C.
D.
Ramipril
Amlodipine
Hydrochlorothiazide
Un des précédents
ABCD-
307. Quel type de traitement serait le plus approprié pour une femme
de 58 ans avec le diabète de type 2 et un taux de cholestérol
total de 210 mg/dL, taux de cholestérol HDL de 52 mg/dL, de
triglycérides 958 mg/dL, et de cholestérol LDL de 131 mg /dL?
A.
B.
C.
D.
A 55 year old man known to have diabetes presents to the
physician for follow up. He has a history of elevated BP and today
his average BP measured is 156/95mmHg. Which of the following
drugs would be recommended as a first choice for this patient
according to JNC8?
Le Fénofibrate 160 mg par jour
L’Atorvastatine 40 mg par jour
La Pravastatine 20 mg par jour
La cholestyramine deux fois par jour
95
Ramipril
Amlodipine
Hydrochlorothiazide
Any of the above
Which type of therapy would be most appropriate for a 58-year-old
female with type 2 diabetes and total cholesterol 210 mg/dL, HDL
cholesterol 52 mg/dL, triglycerides 958 mg/dL, and LDL cholesterol
131 mg/dL?
Fenofibrate 160 mg daily
Atorvastatin 40 mg daily
Pravastatin 20 mg daily
Cholestyramine twice daily
308. Ce médicament n’active pas les récepteurs aux opioïdes ; il a été
proposé comme traitement de maintenance dans l’abus des
opioïdes. Une dose unique bloque l’effet de l’héroïne injectée
pour
48h :
a.Amphétamine
b.Buprénorphine
c. Naloxone
d. Naltrexone
309. Lequel parmi les effets suivants n’apparaît pas en cas
d’intoxication aux salicylés ?
a. Hyperventilation
b. Diarrhée
c. Acidose métabolique
d. Alcalose respiratoire
This drug, which does not activate opioid receptors, has been
proposed as a maintenance drug in treatment programs for opioid
addicts; a singly oral dose will block the effects of injected heroin
for up to 48 hours:
a. Amphetamine
b. Buprenorphine
c. Naloxone
d. Naltrexone
310. Lequel parmi les suivants est un inhibiteur irréversible de la
cyclooxygénase ?
a. Aspirine
b.Phenylbutazone
c. Indométhacine
d. Piroxicam
Which of the following is an irreversible inhibitor of
cyclooxygenase?
a. Aspirin
b. Phenylbutazone
c. Indomethacin
d. Piroxicam
311. Lequel parmi les glucocorticoïdes suivants a aussi une activité
minéralocorticoïde significative?
a. Hydrocortisone
b. Triamcinolone
c. Dexamethasone
d. Betamethasone
Which of the following glucocorticoids has significant
mineralocorticoid activity also?
a. Hydrocortisone
b. Triamcinolone
c. Dexamethasone
d. Betamethasone
96
Which one of the following effects does not occur in salicylate
intoxication ?
a. Hyperventilation
b. Diarrhea
c. Metabolic acidosis
d. Respiratory alkalosis
312. La N-acetylcystéine est utile en cas d’intoxication aigue au
paracetamol parce que :
a- Elle réagit avec le paracétamol pour former un complexe non
toxique
b- Elle inhibe la génération de métabolites toxiques du
paracétamol
c- C’est un consommateur de radicaux libres
d- Elle remplit les stocks de glutathion hépatique qui à son tour
se lie au métabolite toxique du paracétamol
313. La milrinone est un exemple de :
a. Inhibiteur de la Phosphodiesterase I
b. Inhibiteur de la Phosphodiesterase II
c. Inhibiteur de la Phosphodiesterase III
d. Inhibiteur de la Phosphodiesterase IV
N-acetyl cysteine is beneficial in acute paracetamol poisoning
because
a. It reacts with paracetamol to form a nontoxic complex b. It
inhibits the generation of the toxic metabolite of
paracetamol
c. It is a free radical scavenger
d. It replenishes hepatic glutathione which in turn binds the toxic
metabolite of paracetamol
314. L’héparine non fractionnée se lie à l’antithrombine III et inactive
le(s) facteur(s) de coagulation suivant(s) :
a. Xa
b. IXa
c. IIa
d. Toutes les réponses sont correctes
Unfractionated heparin binds to antithrombin III and inactivates
clotting factor(s)
a. Xa
b. IXa
c. IIa
d. All of the above
315. Les dihydropyridines bloquent les types suivants de canaux
calciques :
a. Canaux L-type sensibles au voltage
b. Canaux T-type sensibles au voltage
c. Canaux N-type sensibles au voltage
d. Canaux calciques opérés par des récepteurs
The dihydropyridines block the following type of calcium channels.
a. L-type voltage sensitive channels
b. T-type voltage sensitive channels
c. N-type voltage sensitive channels
d. Receptor operated calcium channels
97
Milrinone is an example of
a. Phosphodiesterase I inhibitor
b. Phosphodiesterase II inhibitor
c. Phosphodiesterase III inhibitor
d. Phosphodiesterase IV inhibitor
316. Lequel parmi les suivants a une activité β2-sélective agoniste,
utilisée pour la prophylaxie de l’asthme ?
a. Albuterol
b. Ipratropium
c. Metaproterenol
d. Salmeterol
Which of the following is a very long acting β2 – selective agonist
that is used for asthma prophylaxis?
a. Albuterol
b. Ipratropium
c. Metaproterenol
d. Salmeterol
317. La terbutaline stimule de préférence les récepteurs suivants :
a. Alpha
b. Beta 1
c. Beta 2
d. Dopaminergiques
Terbutaline has a preference for stimulation of which of the
following receptors?
a. Alpha
b. Beta 1
c. Beta 2
d. Dopaminergic
318. Laquelle des propositions concernant la vancomycine est
correcte :
a- Elle est bactériostatique
b- Elle se lie aux penicillin binding proteins
c- Elle n’est pas sensible aux pénicillinases
d- Elle a une excellente biodisponibilité
Which of the following statements about vancomycin is accurate?
a. It is bacteriostatic
b. It binds to penicillin binding protein
c. It is not susceptible to penicillinase
d. It has the advantage of oral bioavailability
319. Le mécanisme d’action des tétracyclines implique :
a- Une liaison à une composante de la sous-unité 50S du
ribosome
b- Un blocage de la liason de l’aminoacyl-ARNt aux ribosomes
bactériens
c- L’inihibition sélective des peptidyl transférases ribosomiales
d- L’inihibition de la polymérase de l’ARN qui est ADNdépendante
The mechanism of antibacterial action of tetracycline involves
a. Binding to a component of the 50S ribosomal subunit
b. Blockade of binding of aminoacyl – tRNA to bacterial
ribosomes
c. Selective inhibition of ribosomal peptidyl transferases
d. Inhibition of DNA–dependent RNA polymerase
98
320. Les pénicillines interfèrent avec la synthèse de la paroi
bactérienne :
a- En inhibant la synthèse du N-acetyl muramic acid
pentapeptide
b- En inhibant la conjugaison entre l’acide N-acetyl muramique
et la N-acetyl glucosamine
c- En inhibant les transpeptidases et carboxypeptidases qui
ont des liaisons croisées avec les résidus de peptidoglycans
d- Par contrefaçon de la D-alanine dans la paroi bactérienne
321. L’acide clavulanique est combiné à lamoxicilline parce que :
a- Il tue les bactéries non tuées par l’amoxicilline
b- Il réduit la clairance rénale de l’amoxicilline
c- Il antagonise les effets indésirables de l’amoxicilline
d- Il inhibe les betalactamase qui détruisent l’amoxicilline
Penicillins interfere with bacterial cell wall synthesis by
a. Inhibiting synthesis of N-acetyl muramic acid
pentapeptide
b. Inhibiting conjugation between N-acetyl muramic acid and Nacetyl glucosamine
c. Inhibiting transpeptidases and carboxy peptidases which cross
link the Peptidoglycan residues
d. Counterfeiting for D-alanine in the bacterial cell wall
322. Quelle phrase décrit le mieux le mécanisme par lesquels
l’amiloride cause ses effets épargneurs de potassium :
a-Bloque les effets agonistes de l’aldostérone au niveau des
récepteurs des tubules rénaux
b- Bloque les canaux sodiques des tubules distaux, et puis les
échanges Na+-K+
c-Augmente l’inactivation métabolique de l’aldostérone
d-Stimule la Na, K-ATPase au niveau des tubules proximaux
Which phrase best describes the mechanism by which Amiloride
causes its potassium-sparing effects?
a. Blocks the agonist effects of aldosterone with its renal tubular
receptors
b. Blocks distal tubular sodium channels and, ultimately, Na+-K+
exchange
c. Hastens metabolic inactivation of aldosterone
d. Stimulates a proximal tubular Na, K-ATPase
c. Augmentation de la formation de l’oxyde nitrique
d- Augmentation de la synthèse de thromboxane
Which one of the following is the other most likely mechanism by
which nebivolol helps lower blood pressure?
a. Competitively blocks postsynaptic (α1) adrenergic
receptors
b. Competitively blocks presynaptic (α2) adrenergic
receptors
c. Increases nitric oxide formation
d. Increases synthesis of thromboxane A2
323. Lequel parmi les suivants est le mécanisme secondaire le plus
probable par lequel le nebivolol réduit la pression artérielle ?
a- Blocage compétitif des récepteurs adrénergiques
postsynaptiques (α1)
b- Blocage compétitif des récepteurs adrénergiques
présynaptiques (α2)
c. Increases nitric oxide formation
d. Increases synthesis of thromboxane A2
99
Clavulanic acid is combined with amoxicillin because:
a. It kills bacteria that are not killed by amoxicillin
b. It reduces renal clearance of amoxicillin
c. It counteracts the adverse effects of amoxicillin
d. It inhibits beta lactamases that destroy amoxicillin
324. Un homme de 42 ans avec IDM est traité par alteplase ; son
statut électrocardiographique et hémodynamique s’améliore
rapidement. Par quel mécanisme l’alteplase a-t-elle agi ?
a- Blocage des récepteurs ADP des plaquettes
b- Inhibition de la production de la thromboxane plaquettaire
c- Prévention de l’agrégation des plaquettes adjacentes en
bloquant les récepteurs aux glycoprotéines IIb/IIIa
d- Promotion de la conversion de la plaminogène en plasmine
325. Laquelle parmi les propositions suivantes décrit le mieux le
mécanisme d’action de l’abciximab ?
Bloque les récepteurs à la thrombine de façon sélective
Bloque les récepteurs à l’ADP
c- Bloque les récepteurs aux glycoprotéines IIb/IIIa
d- Inhibe la cyclooxygénase
A 42-year-old man with an acute MI is treated with alteplase, and
electrocardiographic and hemodynamic status improves quickly. By
what mechanism did the alteplase cause its beneficial effects?
a. Blocked platelet ADP receptors
b. Inhibited platelet thromboxane production
c. Prevented aggregation of adjacent platelets by blocking
glycoprotein IIb/IIIa receptors
d. Promoted conversion of plasminogen to plasmin
Which of the following statements best describes the mechanism
of action of Abciximab?
a. Blocks thrombin receptors selectively b. Blocks ADP receptors
c. Blocks glycoprotein IIb/IIIa receptors
d. Inhibits cyclooxygenase
326. L’oxybutinine agit par :
Inhibition de l’acetylcholinestérase au niveau des récepteurs
muscariniques et nicotiniques
Cause un blocage neuromusculaire
Antagonise les récepteurs a1
Se lie aux récepteurs muscariniques
Oxybutynin works by
a. Inhibiting acetylcholinesterase at muscarinic and nicotinic
receptors
b. Causing a neuromuscular blockade
c. Antagonizing a1-adrenoceptors
d. Binding to muscarinic receptors
327. Un homme de 27 ans a été récemment diagnostiqué de diabète
type 2 et traité par un médicament. En buvant de l’alcool, il
ressentit des effets très violents. Quel médicament prend-il
probablement ?
a.
Glyburide
b. Metformine
c. Pioglitazone
d. Tolbutamide
A 27-year-old man was recently diagnosed with Type-2 diabetes
mellitus and placed on a medication. As he was drinking alcohol
with his friends, he became violently ill. What medication is he
most likely taking?.
a. Glyburide
b. Metformin
c. Pioglitazone
d. Tolbutamide
100
328. Quelle substance parmi les suivantes est un analogue du peptide
endogène qui inhibe la sécrétion de glucagon ?
a. Exenatide
b. Glipizide
c. Pramlintide
d. Rosiglitazone
Which of the following drugs is an analog of an endogenous
peptide that inhibits glucagon secretion?
a. Exenatide
b. Glipizide
c. Pramlintide
d. Rosiglitazone
329. Quelle substance parmi les suivantes diminue la synthèse des
hormones thyroidiennes et la conversion périphérique de T4 en
T3 ?
a. Lanreotide
b. Levothyroxine
c. Octreotide
d. Propylthiouracil
330. Laquelle parmi les propositions suivantes décrit l’interaction
entre les AINS et le furosémide ?
Vasoconstriction de l’artériole afférente
Vasoconstriction de l’artériole efférente
Augmentation de la réabsorption tubulaire de sodium
Stimulation du récepteur à l’ADH
Which of the following drugs will decrease synthesis of thyroid
hormones and decrease the peripheral conversion of T4 to T3?
a. Lanreotide
b. Levothyroxine
c. Octreotide
d. Propylthiouracil
331. L’ Hydrochlorothiazide peut être utile dans la prévention de
calculs rénaux par le mécanisme suivant :
a-Liaison au calcium urinaire
b-Liaison au sodium urinaire
c. Causing resorption of calcium in the renal tubules
d. Increasing calcium excretion in the renal tubules
Hydrochlorothiazide may be helpful in prevention of renal stones
through which of the following mechanisms?
a. Binding to urine calcium
b. Binding to urine sodium
c-Cause la résorption du calcium au niveau des tubules rénaux
d-Augmente l’excrétion de calcium au niveau des tubules rénaux
332. En cas de glaucome à angle ouvert, l’acetazolamide est donné
pour produire :
a- Une diminution de la production de l’humeur aqueuse
b- Une augmentation de la pression intro-oculaire
c- Une inhibition du transport sodium-calcium
d- Une stimulation de l’anhydrase carbonique
In case of open-angle glaucoma, Acetazolamide is given to produce
which of the following responses?
a. Decrease production of aqueous humor
b. Increasing intraocular pressure
c. Inhibition of sodium-calcium transport
d. Stimulating carbonic anhydrase
101
Which of the following statements describes how NSAIDs may
interfere with the effects of Furosemide?
a. Afferent arteriole vasoconstriction
b. Efferent arteriole vasoconstriction
c. Increase tubular sodium reabsorption
d. Stimulation of the ADH receptor
333. Le teriparatide est un segment recombinant de l’hormone
suivante :
Hormone de stimulation folliculaire
Hormone de croissance
Hormone parathyroidienne
Hormone thyroidienne
Teriparatide is a recombinant segment of which of the following
hormones?
a. Follicle-stimulating hormone
b. Growth hormone
c. Parathyroid hormone
d. Thyroid hormone
334. Quel est le mécanisme de l’orlistat ?
Se lie aux sels biliaires
Inhibe l’alphaglucosidase
Inhibe la formation de chylomicrons
Inhibe la lipase
Which of the following is the mechanism of orlistat?
a. Binding bile salts
b. Inhibition of α –glucosidase
c. Inhibition of chylomicrons formation
d. Inhibition of lipase
335. Un homme de 76 ans avec insuffisance rénale stade 5,
hémodialysé depuis 2 ans, se présente avec des crampes
musculaires. Sa phosphatémie est élevée, et le médecin lui
prescrit du sévélamer. Comment ce dernier agit-il ?
Diminution de la résorption du phosphate des os
Augmentation de l’excrétion dans les selles
Augmentation de l’excrétion dans la salive
Augmentation de l’excrétion dans les urines
A 76-year-old man with stage 5 chronic kidney disease who has
been on hemodialysis for 2 years presents with muscle cramping.
His serum phosphate is markedly elevated. His physician prescribes
sevelamer. How will sevelamer act?
a. Decrease phosphate resorption from bone
b.
Increased excretion in the feces
c. Increased excretion in the sweat
d. Increased excretion in the urine
336. Quel est le mécanisme le plus probable de la caféine ?
a- Augmentation de l’adénosine diphosphate
b- Augmentation de la guanosine monophosphate cyclique
c- Inhibition des récepteurs à la guanosine
d- Inhibition des récepteurs à la thymine
What is the most likely mechanism of action of caffeine?
a. Increase in adenosine diphosphate
b. Increase in cyclic guanosine monophosphate
c. Inhibition of guanosine receptors
d.
Inhibition of thymine receptors
102
337. Lequel parmi les médicaments suivants est hépatotoxique ?
a. Barbituriques
b. Benzodiazepines
c. Salicylés
d. Paracétamol
Which of the following medications is the most commonly
associated with hepatotoxicity?
a. Barbiturates
b. Benzodiazepines
c. Salicylates
d. Acetaminophen
338. La mesure des taux sanguins est indiquée en cas d’intoxication
aux suivants sauf :
Digoxine
Hypoglycémiants oraux
Monoxyde de carbone
Fer
31. Drug level as part of poisoning management is indicated in all of
the following, except:
a. Digoxin
b. Oral hypoglycemics
c. Carbon monoxide
d. Iron
339. L’antidote qui agit par antagonisme pharmacologique direct
est :
a.
Déferoxamine
b. Naloxone
c. Pralidoxime
d. Diurèse alcaline
32. The antidote that acts by direct pharmacological antagonism is:
a. Deferoxamine
b. Naloxone
c. Pralidoxime
d. Alkaline dieresis
340. La déferoxamine agit en cas d’intoxication aiguë au fer comme :
Chélateur
Antagoniste
Inactivateur
Compétiteur
Deferoxamine acts in case of acute iron poisoning as:
a. Chelator
b. Antagonist
c. Inactivator
d. Competitor
341. On traite l’arrhythmie cardiaque due aux antidépresseurs
tricycliques par :
a. Dopamine
b. Bicarbonate de sodium
c. Epinephrine
d. Phenylephrine
Cardiac dysrhythmias in case of tricyclic antidepressants toxicity is
best treated with:
a. Dopamine
b. Sodium bicarbonate
c. Epinephrine
d. Phenylephrine
103
342. Le traitement de première ligne de l’intoxication aux betabloquants :
a.
Glucagon
b. Calcium
c. Inhibiteurs de la Phosphodiesterase
d. Catécholamines
The first line treatment for beta blockers poisoning:
a. Glucagon
b. Calcium
c. Phosphodiesterase inhibitors
d. Catecholamines
343. Laquelle de ces pairs de substance:antidote est fausse :
a. Methanol: Ethanol
b. Cyanure: Thiocyanate
c. Methemoglobinemie: Bleu de Methylene
d. Monoxyde de Carbone: Oxygène
Which of the following pairs is False regarding drugs and their
appropriate antidotes:
a. Methanol: Ethanol
b. Cyanide: Thiocyanate
c. Methemoglobinemia: Methylene blue
d. Carbon monoxide: Oxygen
344. Laquelle des propositions est fausse concernant le charbon actif :
a- Adsorbe tous les toxiques ingérés et préveint leur absorption
gastrointestinale
b- Le mieux est de l’utiliser dans la première heure après
ingestion
c- Contre-indiqué en cas d’obstruction gastrointestinale
d- Des doses multiples de charbon actif interrompent un cycle de
circulation enterohépatique
345. Après un surdosage en aspirine, le problème acido-basique de
base est d’habitude :
Une acidose respiratoire
Une acidose métabolique
Une alcalose respiratoire
Une alcalose métabolique
Regarding Activated charcoal which is incorrect:
a. Adsorbs all toxic ingestants preventing further GI
absorption
b. Best if used within first hour of ingestion
c. Contraindicated in case of gastrointestinal obstruction
d. Multiple dose activated charcoal interrupts
enterohepatic recirculation
346. Lequel parmi les suivants est l’antidote des benzodiazépines ? A.
Flumazenil
B. Bleu de Methylene
C. Deferoxamine
D. Alkalinisation des urines
Which of the following is the antidote for the toxin
Benzodiazepines?
A. Flumazenil
B. Methylene blue
C. Deferoxamine
D. Alkalinize urine
104
Following aspirin overdose the initial acid-base derangement is
usually:
a. Respiratory acidosis
b. Metabolic acidosis
c. Respiratory alkalosis
d. Metabolic alkalosis
347. Lequel parmi les suivants est l’antidote du plomb ? A. Naloxone
B. Nitrite
C. CaEDTA
D. Dialyse
Which of the following is the antidote for the toxin Lead?
A. Naloxone
B. Nitrite
C. CaEDTA
D. Dialysis
348. Lequel parmi les suivants est le site d’action primaire de la
warfarine ?
A-Rein
Foie
C-Sang
D-Cœur
Which of the following is the primary site of activity for the drug
Warfarin?
a. Kidney
b. Liver
c. Blood
d. Heart
349. Lequel parmi ces médicaments est le plus associé à une réaction
inflammatoire hépatique ?
Which of the following drugs is mainly associated with the reaction
of hepatitis?
A.
Acide valproique
B.
Quinidine
C.
Isoniazide
D.
Ethosuximide
350. Lequel parmi les suivants est associé à une réaction de
dysfonctionnement des tendons ?
A- Digoxine
B- Niacine
C- Tetracycline
D- Fluoroquinolones
A. Valproic acid
B. Quinidine
C. Isoniazid
D. Ethosuximide
Which of the following drugs is associated with the reaction of
Tendon dyfunction?
A. Digitalis
B. Niacin
C. Tetracycline
D. Fluoroquinolones
351. Lequel parmi les suivants est associé avec une extrême
photosensibilité ?
A- Digoxine
B- Niacine
C- Tetracycline
D- Fluoroquinolones
Which of the following drugs is associated with the reaction of
extreme photosensitivity?
A. Digitalis
B. Niacin
C. Tetracycline
D. Fluoroquinolones
105
352. Lequel parmi les suivants n’est pas lié à la toxicité de la
prednisone?
A- Cataracte
B- Hypotension
C- Psychoses
D- Acné
Which of the following is not related to a drug toxicity of
Prednisone? A. Cataracts
B. Hypotension
C. Psychosis
D. Acne
353. lequel parmi les suivants n’est pas lié à la toxicité de l’atenolol ?
A- Insuffisance cardiaque congestive
B- Tachycardie
C- Bloc AV
D- Apparence sédatée
Which of the following is not related to a drug toxicity of
Atenolol? A. CHF
B. Tachycardia
C. AV block
D. Sedative appearance
354. Les diurétiques épargneurs de potassium ont un effet primaire
sur les…….des reins.
A- Tubules proximaux
B- Anse de Henlé
C- Tubes collecteurs
D- Tubules distaux
Potassium sparing diuretics have the primary effect upon the
found in the kidney.
A. Proximal convoluted tubule
B. Loop of Henle
C. Collecting duct
D. Distal convoluted tubule
355. Lequel parmi les suivants n’est pas lié à la toxicité de la
nitroglycérine ?
A- Céphalées
B- Tachycardie
C- Vertige
D- Vomissements
Which of the following is not directly related to a drug toxicity of
Nitroglycerin?
A. Headaches
B. Tachycardia
C. Dizziness
D. Vomiting
356. Lequel parmi les suivants n’est pas directement lié à la toxicité de
l’ibuprofen ?
A- Nausée
B- Dysfonctionnement rénal
C- Anémie
D- Fonte musculaire
Which of the following is not directly related to a drug toxicity of
Ibuprofen?
A. Nausea
B. Renal dysfunction
C. Anemia
D. Muscle wasting
106
357. Lequel parmi les suivants ne fait pas partie de l’étymologie du
mot « Pharmacologie » ?
a) Médicament
b) Plante
c) Poison
d) Etude
Which of the following is NOT part of the etymology of the word
pharmacology?
a) Drug
b) Herb
c) Poison
d) Study
358. Lequel parmi les suivant décrit un agoniste ?
a- Toute substance entraînant un changement de fonction
biologique par une réaction chimique
b- Une molécule de régulation spécifique d’un système
biologique où le médicament agit
c- Un médicament qui se lie à et stimule une activité cellulaire
d- Un médicament qui se lie à un récepteur et inhibe ou oppose
une activité cellulaire
Which of the following describes an agonist?
a) Any substance that brings about a change in biologic function
through its chemical action
b) A specific regulatory molecule in the biologic system where a
drug interacts
c) A drug that binds to a receptor and stimulates cellular activity
d) A drug that binds to a receptor and inhibits or opposes cellular
activity
359. Un xénobiotique est considéré :
a- Endogène
b- Exogène
c- Un poison inorganique
d- Une toxine
Xenobiotics are considered:
a) Endogenous
b) Exogenous
c) Inorganic poisons
d) Toxins
360. Lequel parmi les suivants serait une toxine d’origine biologique ?
a) Pb
b) As
c) Hg
d) Atropine
Which of the following would be a toxin of biological origin?
a) Pb
b) As
c) Hg
d) Atropine
361. La vaste majorité des medicaments ont un poids moléculaire
entre 100 et 1000. Les grosses molécules, tel que l’alteplase,
devraient être administrées :
a- Dans le compartiment où elle va agir
b- Par voir orale pour qu’elle ne soit pas absorbée trop vite
c- Par voie rectale pour prévenir l’irritation de la paroi
The vast majority of drugs have molecular weights (MW) between
100 and 1,000. Large drugs, such as alteplase (t-PA), must be
administered:
a) Into the compartment where they have their effect
b) Orally so they do not absorb too quickly
c) Rectally to prevent irritation to the stomach lining and vessels
107
stomacale et des vaisseaux
d- Par voie intraosseuse
d) Via the intraosseous (IO) route
362. Lequel parmi les suivants a lieu avec les petites molécules tel que
le lithium ?
a- Endocytose médiée par un récepteur
b- Vasodilatation si injecté par voie intraveineuse
c- Liaison spécifique à un récepteur
d- Liaison non spécifique
Which of the following occurs with drugs that are extremely small,
such as Lithium?
a) Receptor mediated endocytosis
b) Vasodilation when injected intravenously (IV)
c) Specific receptor binding
d) Nonspecific binding
363. Les médicaments s’adéquatent au récepteur selon le modèle de
la clef et de la serrure. Les liaisons covalentes sont les plus …. et
les…. spécifiques :
a- Fortes ; plus
b- Fortes ; moins
c- Faibles ; plus
d- Faibles ; moins
364. Qu’est-ce qui détermine le degré de mouvement d’un
médicament entre les compartiments de l’organisme?
a- Constante de partition
b- Degré d’ionisation
c- Taille
d- Toutes les réponses sont correctes
Drugs fit receptors using the lock and key model. Covalent bonds
are the
and the
specific.
a) Strongest; Most
b) Strongest; Least
c) Weakest; Most
d) d) Weakest; Least
365. Lequel parmi les suivants n’est PAS une cible protéique pour la
liaison d’un médicament ?
A.
Site d’action
B.
Enzyme
C.
Molécule de transport
D.
Récepteur
Which of the following is NOT a protein target for drug binding?
a) Site of action
b) Enzyme
c) Carrier molecule
d) Receptor
366. Lequel parmi les suivants est un exemple de médicament agissant
directement sur un récepteur ?
a- Protamine qui se lie stoechiométriquement aux
Which of the following is an example of a drug acting directly
through receptors?
a) Protamine binds stoechiometrically to heparin anticoagulants
108
What determines the degree of movement of a drug between body
compartments?
a) Partition constant
b) Degree of ionization
c) Size
d) All of the above
anticoagulants hépariniques
b- Les beta-bloquants adrénergiques pour la tachycardie
induite par l’hormone thyroidienne
c- L’epinephrine augmentant la fréquence cardiaque et la
tension artérielle
d- Les agents utilisés en chimiothérapie du cancer
367. Qu’ajoute-t-on en sous-classifiant une substance, par exemple
antituberculeux versus antibactérien ?
A.
Le coût
B.
La taille
C.
L’ionisation
D.
La précision
b) Adrenergic beta blockers for thyroid hormone-induced
tachycardia
c) Epinephrine for increasing heart rate and blood pressure
d) Cancer chemotherapeutic agents
368. Quel effet indésirable n’est PAS observé avec les antagonistes
cholinergiques ?
a- Rétention urinaire
b- Confusion
c- Miosis
d- Constipation
Which of the following adverse effects (side-effects) is
NOT commonly seen with cholinergic antagonists?
a) Urinary retention
b) Confusion
c) Miosis
d) Constipation
369. Le chloramphenicol a un risque maximal pour :
a- Nouveau-nés
b- Patients en gériatrie
c- Hommes adultes
d- Patients atteints d’insuffisance cardiaque congestive
The drug chloramphenicol is most risky for:
a) Neonates
b) Geriatric patients
c) Adult males
d) Congestive heart failure patients
370. Comment varie le taux de filtration glomérulaire après 40 ans ?
A- Augmentation de 1% chaque année
B- Augmentation de 2% chaque année
C- Diminution de 1% chaque année
D- Diminution de 2% chaque année
How does the glomerular filtration rate (GFR) change after the age
of 40?
a) Increase 1% each year
b) Increases 2% each year
c) Decreases 1% each year
d) Decreases 2% each year
109
What is added with drug subclassification, such as an antitubercular
drug versus an antibacterial drug?
a) Cost
b) Size
c) Ionization
d) Precision
371. Une diminution des fonctions rénales et hépatiques, chez les âgés
par exemple, prolonge la demi-vie des médicaments, … la
liaison aux protéines plasmatiques, et …. le volume de
distribution.
a- Augmente ; augmente
b- Diminue ; diminue
c- Augmente ; diminue
d- Diminue ; augmente
A decrease in renal and liver function, as seen in the elderly, would
prolong drug half-life, plasma protein binding, and
volume of
distribution.
a) Increase; Increase
b) Decrease; Decrease
c) Increase; Decrease
d) Decrease; Increase
372. La pharmacocinétique est l’effet de ….. et la pharmacodynamique est l’effet de…..
a- Un médicament sur un médicament ; l’organisme sur un
médicament
b- L’organisme sur un médicament ; un médicament sur un
médicament
c- Médicament sur l’organisme ; l’organisme sur le
médicament
d- L’organisme sur un médicament ; un médicament sur
l’organisme
Pharmacokinetics is the effect of the
the effect of the
.
a) Drug on a drug; Body on the drug
b) Body on the drug; Drug on a drug
c) Drug on the body; Body on the drug
d) Body on the drug; Drug on the body
373. Lequel parmi les suivants n’est PAS une action de l’organisme
sur un médicament ?
a- Absorption
b- Distribution
c- Métabolisme
d- Effets indésirables
374. Si un médicament est lié à 80% aux éléments figurés du sang ou
aux protéines plasmatiques, quelle fraction est considérée
comme forme libre ?
a-20%
b-50%
c-80%
d-100%
Which of the following is NOT an action of the body on a drug?
a) Absorption
b) Distribution
c) Metabolism
d) Side effects
110
and pharmacodynamics is
If a drug is 80% bound to blood elements or plasma proteins, what
part is considered the free form?
a) 20%
b) 50%
c) 80%
d) 100%
375. Quelle proposition décrit le mieux la Concentration Minimale
Effective (CME) ?
a- La concentration plasmatique minimale de médicament qui
peut être
détectée
b- La concentration plasmatique minimale de médicament qui
rentre aux tissus
c- La concentration plasmatique minimale de médicament qui
interagit avec les récepteurs
d- La concentration plasmatique minimale de médicament qui
produit un effet
376. Si un patient rate trois doses de son médicament journalier,
quelle est en général la meilleure solution ?
a- Prendre 4 fois la dose la fois prochaine
b- Attendre 3 jours (semaine complète) puis retourner au régime
normal
c- Ne rien faire et continuer un régime normal
d- Prescrire une dose supérieure pour que les doses ratées
aient moins d’effet
Which of the following describes minimal effective concentration
(MEC)?
a) The minimal drug plasma concentration that can be detected
b) The minimal drug plasma concentration to enter tissues
c) The minimal drug plasma concentration to interact with
receptors
d) The minimal drug plasma concentration to produce effect
377. Quel mécanisme de perméation médicamenteuse se passe à
travers des jonctions serrées et il est dû un gradient de
concentration ?
A.
Diffusion aqueuse
B.
Diffusion lipidique
C.
Molécules de transport
D.
Endocytose et exocytose
Which of the following drug permeation mechanisms occurs across
epithelial tight junctions and is driven by a concentration gradient?
a) Aqueous diffusion
b) Lipid diffusion
c) Carrier molecules
d) Endocytosis and exocytosis
378. Quel mécanisme de perméation médicamenteuse est basé sur
l’équation de Henderson- Hasselbalch pour le rapport de
solubilité d’un acide faible ou une base faible ?
a- Diffusion aqueuse
b- Diffusion lipidique
Which of the following drug permeation mechanisms uses the
Henderson- Hasselbalch equation for the ratio of solubility for the
weak acid or weak base?
a) Aqueous diffusion
b) Lipid diffusion
111
If a patient misses three doses of their daily drug, which of the
following (in general) is the best solution?
a) Take a 4x dose at the next dose time
b) Wait 3 more days (week total) then return to normal regimen
c) Do nothing and continue normal regimen
d) Prescribe a higher dosage pill so missed doses will have less
effect
c- Molécules de transport
d- Endocytose et exocytose
c) Carrier molecules
d) Endocytosis and exocytosis
379. Quel mécanisme de perméation médicamenteuse est utilisé par
les peptides, aminoacides, et autres molécules larges ou
insolubles?
a- Diffusion aqueuse
b- Diffusion lipidique
c- Molécules de transport
d- Endocytose et exocytose
Which of the following drug permeation mechanisms is used for
peptides, amino acids, and other large or insoluble molecules?
a) Aqueous diffusion
b) Lipid diffusion
c) Carrier molecules
d) Endocytosis and exocytosis
380. 35-Lequel parmi les suivants n’est PAS nécessaire pour la
bioéquivalence médicamenteuse ?
a- Même dose ou concentration
b- Même forme
c- Même voie d’administration
d- Même effets indésirables
Which of the following is NOT needed for drug bioequivalence?
a) Same strength or concentration
b) Same dosage form
c) Same route of administration
d) Same side effects
381. Pour un médicament administré par voie intraveineuse, quelle est
la bioéquivalence ?
a) 0%
b) 25%
c) 75%
d) 100%
For intravenous (IV) dosages, what is the bioavailability assumed to
be?
a) 0%
b) 25%
c) 75%
d) 100%
382. Pour qu’un générique soit bioéquivalent à un princeps, …% des
sujets devraient avoir une concentration du générique qui diffère
de moins de …% de la moyenne du princeps dans la population
testée.
a) 50; 50 %
b) 80; 20 %
c) 20; 80 %
d) 95; 5 %
For a measured bioavailability, generic drug to be bioequivalent to
an innovator drug (per FDA), it must be in
of subjects to fall within
of the mean of the test
population
a) 50; 50 %
b) 80; 20 %
c) 20; 80 %
d) 95; 5 %
112
383. Lequel n’est PAS un processus pharmacocinétique ?
a- Altération du médicament par les enzymes hépatiques
b- Elimination par les urines des métabolites médicamenteux
c- Passage du médicament des intestins à la circulation
d- Dilatation des artères coronaires par le médicament
Which of the following is NOT a pharmacokinetic process?
a) Alteration of the drug by liver enzymes
b) Drug metabolites are removed in the urine
c) Movement of drug from the gut into general circulation
d) The drug causes dilation of coronary vessels
384. Un médicament peut produire une réponse thérapeutique s’il
est :
A.
Lié aux protéines plasmatiques
B.
Concentré dans la bile
C.
Concentré dans les urines
D.
Non lié aux protéines plasmatiques
Which of the following can produce a therapeutic response? A drug
that is:
a) Bound to plasma albumin
b) Concentrated in the bile
c) Concentrated in the urine
d) Unbound to plasma proteins
385. Quel mécanisme explique le mieux l’accés des hormones
stéroidiennes à leurs sites de fixation intracellulaires ? Elles :
a- Traversent la membrane cellulaire par des pores aqueux
b- Ont un haut coefficient de perméabilité
c- Sont passivement transportées par transport
membranaire
d- Requièrent un transport vésiculaire
Which of the following most correctly describes steroid hormones
with respect to their ability to gain access to intracellular binding
sites?
a) They cross the cell membrane via aqueous pores
b) They have a high permeability coefficient
c) They are passively transported via membrane carriers
d) They require vesicular transport
113
386. Lequel parmi les tests suivants détermine le contrôle à long terme
du diabète ?
a. Glycémie à jeun
b. Test de tolérance orale du glucose
c. Test de tolérance intraveineuse du glucose
d. Niveau de l’HbA1C
Which of the following tests can determine the long term
control of diabetes mellitus?
A. Fasting blood sugar level
B. Oral glucose tolerance test
C. IV glucose tolerance test
D. HbA1C level
387. Le niveau de l’HbA1C reflète le contrôle du diabète sur :
A.
Le dernier mois
B.
Les derniers 2-3 mois
C.
Les derniers 3-4 mois
D.
Les derniers 6 mois
388. La présence de protéines dans les urines indique :
a- Une cirrhose hépatique
b- Une glomérulonéphrite
c- Une cétoacidose diabétique
d- Une famine sévère
HbA1C level reflects the control of diabetes mellitus over the
last:
A. 1 month
B. 2-3 months
C. 3-4 months
D. 6 months
The presence of protein in urine indicates:
A. liver cirrhosis
B. glomerulonephritis
C. diabetic ketoacidosis
D. severe starvation
389. La présence de nitrites dans les urines indique:
a- Une infection du tractus urinaire
b- Une insuffisance hépatique
c- Une maladie rénale
d- Un problème gastrointestinal
The presence of nitrites in urine indicates:
A. urinary tract infection
B. hepatic failure
C. kidney disease
D. GIT disorder
390. Laquelle n’est PAS une cause d’hypocalcémie ?
a- L’insuffisance rénale chronique
b- L’Hyperparathyroidisme c- L’Hypoparathyroidisme
d- Une déficience en vitamine D
Which of the following is not a cause of hypocalcemia?
A. chronic renal failure
B. hyperparathyroidism
C. hypoparathyroidism
D. vitamin D deficiency
391. L’hyperbilirubinémie peut être due à :
a- Une insuffisance rénale aiguë
Hyperbilirubinemia could be due to:
A. acute renal failure
114
b- Une hémolyse
c- Des calculs rénaux
d- Une déficience en fer
B. hemolysis
C. renal stones
D. iron deficiency
392. La concentration de l’acide urique devrait être normalement de :
A. 3 - 7 mg/dl
B. 3 – 7 g/dl
C. 0.3 – 0.7 mg/dl
D. 0.3 – 0.7 g/dl
Uric acid in the serum should be normally:
A. 3 - 7 mg/dl
B. 3 – 7 g/dl
C. 0.3 – 0.7 mg/dl
D. 0.3 – 0.7 g/dl
393. Un enfant de 4 ans, né prématuré, a développé un enduit blanc et
douloureux sur sa muqueuse buccale qui s’étend jusqu’aux lèvres.
Quel est le microorganisme en cause le plus probable ?
a) Actinomyces
b) Aspergillus
c) Candida
d) Microsporum
A premature baby, now 4 years old, has developed a white
coating on her buccal mucosa extending onto her lips. It
appears to be painful. What is the most likely causative agent?
a) Actinomyces
b) Aspergillus
c) Candida
d) Microsporum
394. Ce pathogène urinaire pousse sur une surface d’agar et peut
causer des calculs rénaux
a) Citrobacter freundii
b) Enterobacter aerogenes
c) Klebsiella oxytoca
d) Proteus mirabilis
This urinary pathogen "swarms" across agar surfaces and may
cause bladder and renal calculi (stones)
a) Citrobacter freundii
b) Enterobacter aerogenes
c) Klebsiella oxytoca
d) Proteus mirabilis
395. Ce pathogène cause une diarrhée riziforme :
a) Vibrio cholera
b) Camylobacter jejuni
c) Helicobacter pylori
d) Shigella dysenteriae
“Rice water stools” are associated with disease caused
by which organism
a) Vibrio cholera
b) Camylobacter jejuni
c) Helicobacter pylori
d) Shigella dysenteriae
The primary viral cause of diarrhea in young children, globally
is
a) Cryptosporidium
396. La cause virale primaire d’une diarrhée chez les jeunes enfants est
globalement :
a) Cryptosporidium
115
b) Rotavirus
c) Adenovirus
d) Norovirus
397. …. n’est PAS un virus qui cause les infections respiratoires basses
chez les bébés
a) Adenovirus
b) Rhinovirus
c) RSV
d) Klebsiella Pneumoniae
b) Rotavirus
c) Adenovirus
d) Norovirus
-------- is NOT a virus that commonly cause of severe lower
respiratory tract disease in infants
a) Adenovirus
b) Rhinovirus
c) RSV
d) Klebsiella Pneumoniae
398. Lequel des microorganismes suivants est associé avec une ulcération
gastrique :
a) Camylobacter jejuni
b) Helicobacter pylori
c) Shigella dysenteriae
d) Enteroinvasive E. coli
Which of the following organisms would most likely be
associated with gastric ulceration
a) Camylobacter jejuni
b) Helicobacter pylori
c) Shigella dysenteriae
d) Enteroinvasive E. coli
399. La caractéristique la plus importante de la diarrhée causée par
Shigella dysenteriae est :
A.
Diarrhée sanglante
B.
Insuffisance rénale
C.
Diarrhée riziforme
D.
Pas de sécrétion de mucus ou de pus
The most important characteristic of diarrhea caused by
Shigella dysenteriae is:
a) Bloody diarrhea
b) Renal insufficiency
c) Rice-watery diarrhea
d) No pus or mucous secretion
400. Une caractéristique importante des mycobactéries est qu’elles
sont :
A.
Des Bacilles gram négatif
B.
A croissance rapide (temps double ; 15 minutes)
C.
Teinture rapide par acide
D.
Alpha hemolytiques
An important characteristic of mycobacteria is that they are:
a) Gram negative rods
b) Rapid growing (doubling time 15 minutes)
c) Acid fast staining
d) Alpha hemolytic
401. Selon l’OMS, la vaccination a entraîné l’éradication mondiale de :
a- La poliomyélite
According to WHO, vaccination has led to the eradication of
such disease
116
b- La variole
c- La fièvre jaune
d- La rougeole
402. les préparations de sené peuvent être utilisées dans le
traitement de:
A.
La constipation
B.
Le diabète
C.
La fièvre
D.
La toux
a) Poliomyelitis
b) Smallpox
c) Yellow fever virus
d) Measles virus
Senna preparations can be used for treatment of :
a. Constipation
b. Diabetes
c. Fever
d. Cough
403. La valériane peut être utilisée pour :
A.
Les douleurs rhumatismales
B.
L’anxiété et troubles du sommeil
C.
La migraine
D.
La diarrhée
Valeriana can be used for the treatment of :
a. Rheumatic pain
b. Anxiety and sleep disorders
c. Migraine
d. Diarrhea
404. La morphine est un alcaloïde obtenu de :
a- La capsule du pavot
b- Les fruits du capsicum
c- Les herbes d’Hyoscyamus
d- Le fruit de l’anis
405. L’atropine est un :
A.
Ester d’alcaloïde
Alcaloide aliphatique
B.
C.
Glycoside aromatique
D.
Dérivé d’anthracène
406. Les préparations de Ginkgo sont efficaces comme :
a. Immunostimulant
Morphine is an alkaloid obtained from :
a. Poppy capsule
b. Capsicum fruits
c. Hyoscyamus herb
d. Anise fruit
Atropine is an :
a. Ester alkaloid
b.Aliphatic alkaloid
c. Aromatic glycoside
d.anthracene derivative
Ginkgo preparations are effective as :
a. Immunostimulant
b. Antidiarrheal
c. Expectorant
d. Antihelminitic
b- Antidiarrhéique
c- Expectorant
d- Antihelmintique
117
407. Les préparations de licorice sont contrindiquées en cas de :
a- Constipation
b- Fièvre
c- Angine de poitrine
d- Hypertension
Liquorice preparations are contraindicated in case of a.
Constipation
b. Fever
c. Angina pectoris
d. Hypertension
408. 16-La diosmine est recommandée en cas de :
a- Problèmes gastrointestinaux
b- Hypertension
c- Insuffisance cardiaque
d- Veines variqueuses
Diosmin is recommended in case of
a. GIT disorders
b. Hypertension
c. Heart failure
d. Varicose veins
409. La vincristine est utilisée pour le traitement de :
a- L’insuffisance cardiaque
b- La bronchite
c- Les problèmes vasculaires
d- Le cancer
410. Les bioflavonoides sont utilisés comme :
aLaxatifs de lest
b- Laxatifs stimulants
c- Antioxydants
d- Supplément alimentaire
411. Les préparations d’ivy sont utilisées comme :
a- Préparations expectorantes
b- Préparations digestives
c- Préparations amincissantes
d- Dépresseur du système nerveux central
Vincristine is used for the treatment of a. Congestive heart
failure
b. Bronchitis
c. Vascular disorders
d. Cancer
Natural bioflavonoids are used as :
a. Bulk laxative
b. Stimulant laxative
c. Antioxidant
d. Dietary supplement
Ivy preparations are used in :
a. Expectorant preparations
b. Digestive preparations
c. Slimming preparations
d. CNS depressant preparations
412. Le menthol est utilisé comme :
a- Antiseptique
b- Antihelmintique
c- Antirhumatismal
d- Astringent
Menthol is used as :
a. Antiseptic
b. Anthelmintic
c-.Antirheumatic
d- Astringent
118
413. Le thymol est classifié comme :
a- Alcaloïde
b- Glucoside
c- Huile volatile
d- Acide aromatique
Thymol is classified under :
a.alkaloids
b.glycosides
c.volatile oils
d. aromatic acids
414. Les saponines sont caractérisées par :
a- Une mousse stable dans l’eau
b- Une mousse temporaire dans l’eau
c- Pas de mousse dans l’eau
d- Une mousse avec l’alcool uniquement
Saponins are characterized by :
a. Giving a stable foam in water
b. Giving temporary foam with water
c. Giving no foam with water
d. Giving foam with alcohol only
415. Une boisson utilisée comme carminatif pour les bébés contient :
A.
fenouil
B.
Capsicum
C.
Coriandre
D.
Senné
Baby drink use as carminative should contain:
A.
Fennel
B.
Capsicum
C.
Coriander
D.
Senna pods
416. La quinine est utilisée comme :
a- Anticoagulant
b- Antidépresseur
c- Antidiabétique
d- Antipaludique
Quinine is used as an
a. Anticoagulant
b. Antidepressant
c. Antidiabetic
d. Antimalarial
417. Clou de girofle est utilisé pour :
A.
Les douleurs dentaires
B.
Les ulcères des pieds
C.
Les douleurs de la poitrine
D.
Les douleurs rhumatismales
418. Les préparations de Ginseng sont utilisées :
A.
Dans les troubles hépatiques
B.
Dans les troubles cardiaques
C.
Dans les troubles rénaux
Clove is used in:
a. Tooth pain
b. Foot ulcers
c. Chest pain
d. Rheumatic pain
Ginseng preparations can be used :
A.
In liver disorders
B.
In cardiac disorders
C.
In kidney problems
D.
As immune-stimulant
119
D.
Comme immunostimulant
419. Les mélanges à base de plusieurs plantes sont :
A.
Plus sécurisés qu’une préparation à base d’une seule
B.
Non recommandés à tous les patients
C.
Recommandés pour les enfants
D.
Recommandés pour les femmes enceintes
420. Les amincissants naturels ne devraient pas inclure de:
a-vitamines
b-Produits non identifiés
c-supplément nutritionnel
d-Antioxydant naturel
421. Une personne souffrant d’insomnie peut prendre
A.
Du thé à la Camomille allemande
B.
Du sirop d’ipecac
C.
Une infusion de grains noirs
D.
Du thé au Ginseng
Multiherbal mixture preparations are:
A.
Safer than single herbal preparations
B.
Not recommended to all patients
C.
Recommended for children
D.
Recommended for pregnant women
Natural Slimming preparations should not contain:
A.
Vitamins
B.
Unidentified matters
C.
Nutritional supplements
D.
Natural anti-oxidants
A person suffers from insomnia can drink:
A.
German chamomile Tea bags
B.
Ipeca syrup
C.
Black seed infusion
D.
Ginseng Tea bags
422. Les préparations d’hyoscine sont disponibles sous forme de :
a-Sirops
b-Comprimés
c-Mélanges d’herbes
D Capsules
423. Les composantes de l’aromathérapie devraient inclure :
A. Huiles volatiles seulement
B. Huiles volatiles et fixes
C. Huiles fixes seulement
D. Flavonoides naturels
424. Le gingembre est recommandé comme :
A.
Antimicrobien
B.
Anticoagulant
C.
Antihypertenseur
D.
Antidépresseur
425. Le catharanthus est la source essentielle de :
Hyoscine preparations are available as:
A.
Syrups
B.
Tablets
C.
Multiherbal mixtures
D.
Capsules
Aromatherapy components should include:
a. Volatile oils only
b. Fixed oils and volatile oils
c. Fixed oils only
d. Natural Flavonoids
Ginger is recommended as :
A. As antimicrobial
B. Anticoagulant
C. Antihypertensive
D. Antidepressant
Catharanthus herb is the main source of:
120
a- Vincristine
b- Ergotamine
c- Hyoscyamine
d- Nicotine
426. Comment les médicaments de prescription sont différents des
médicaments du comptoir OTCs?
A. Ils contiennent des quantités élevées d'ingrédients actifs
a- Vincristine
b- Ergotamine
c- Hyoscyamine
d- Nicotine
How are prescription drugs different from OTC ones?
A. They contain higher amounts of active ingredients
B. They don't contain dyes or preservatives
B. Ils ne contiennent pas de colorants ou conservateurs
C. They're unsafe for use without medical supervision
C. Ils sont dangereux à utiliser sans surveillance médicale
D. They can be toxic
D. Ils peuvent être toxiques
427. Vous devez suivre certaines précautions lorsque vous traitez avec You should follow certain precautions when you self-treat with
des médicaments du comptoir OTCs. Que devrez-vous faire?
OTC drugs. What should you do?
A. diagnostiquer le problème de santé attentivement
A. Diagnose your health problem carefully
B. Lire attentivement l'étiquette et suivre les instructions de dosage
B. Read the label carefully and follow dosage instructions
C. Suivre tous les avertissements de l'étiquette
C. Follow any label warnings
D. Tout ce qui précède
D. All of the above
428. L'aspirine peut provoquer des problèmes avec laquelle de ces Aspirin can cause problems with which of these conditions?
conditions?
A. Asthma
A. asthme
B. Bronchitis
B. Bronchite
C. Sinusitis
121
C. Sinusite
D. Low blood pressure
D. Une pression artérielle basse
429. Les personnes allergiques à l'aspirine peuvent également être People allergic to aspirin may also be allergic to which other
allergiques à d'autres médicaments du comptoir?
OTC drugs?
A. Naproxen
A. Naproxen
B. acétaminophène
B. Acetaminophen
C. épinéphrine
C. Epinephrine
D. stéroïdes
D. Steroid
430. Vous pouvez prendre des médicaments anti-inflammatoires non You can take nonsteroidal anti-inflammatory drugs safely for a
stéroïdiens en toute sécurité avant de voir un médecin pour un maximum duration before seeing a doctor for a diagnosis?
diagnostic, pour une durée maximale
A. 1 to 3 days
A. 1 à 3 jours
B. 7 to 10 days
B. 7 à 10 jours
C. 3 to 5 days
C. 3 à 5 jours
D. 30 days
D. 30 jours
431. L'acétaminophène est moins probable que l'aspirine à causer des Acetaminophen is less likely than aspirin to cause gastric
douleurs gastriques, mais une surdose d'aussi peu que 4 grammes distress, but an overdose of as little as 4 grams can lead to
peut conduire à une maladie du foie irréversible. Qui est le plus à irreversible liver disease. Who is most at risk?
risque?
A. People who drink alcohol
A. Les gens qui boivent de l'alcool
B. Anyone on a diet
122
B. Toute personne sur un régime alimentaire
C. Smokers
C. Les fumeurs
D. A and C
D. A et C
432. Les personnes âgées sont particulièrement vulnérables aux effets Older adults are especially vulnerable to the side effects of
secondaires duquel de ces médicaments du comptoir?
which of these OTC drugs?
A. Les laxatifs
A. Laxatives
B. Les analgesiques
B. Pain relievers
C. Les antihistaminiques
C. Antihistamines
D. Les AINS topiques
D. Topical NSAIDs
433. De nombreux médicaments du comptoir pour les enfants viennent Many OTC medications for children come in liquid form. What
sous forme liquide. Quelle est la meilleure façon de mesurer la is the best way to measure the dosage?
dose?
A. Use a kitchen tablespoon
A. Utilisez une cuillère à soupe de cuisine
B. Use a kitchen teaspoon
B. Utilisez une cuillère à café de cuisine
C. Use a special dosing spoon
C. Utilisez une cuillère de dosage spéciale
D. Any spoon would work
D. Toute cuillère fonctionnerait
434. Que devez-vous rechercher
médicament du comptoir?
lorsque
vous
sélectionnez
un What should you look for when you select an OTC drug?
A. One that helps a wide range of symptoms
123
A. Un qui permet un large éventail de symptômes
B. One that helps your primary symptom
B. Un qui aide votre symptôme principal
C. One that's a time-release formula
C. Un qui est une formule à libération prolongée
D. One that doesn't have red dye
D. Un qui n'a pas le colorant rouge
435. En cas d'intoxication par le fer, les organes les plus touchés sont les
suivants:
A. Le cœur et le foie parce que la ferritine est largement abondante
dans ces organes et l’excès de fer provoque la destruction des
tissus.
B. Le rein parce que la ferritine est largement abondante dans ces
organes et l’excès de fer provoque la destruction des tissus.
C. Le cerveau parce que la ferritine est largement abondante dans
ces organes et l’excès de fer provoque la destruction des tissus.
In iron intoxication, the most affected organs include:
A. The heart and liver because ferritin is greatly abundant in
those organs for excess iron to build up and causes tissue
destruction.
B. The kidney because ferritin is greatly abundant in those
organs for excess iron to build up and causes tissue
destruction.
C. The brain because ferritin is greatly abundant in those
organs for excess iron to build up and causes tissue
destruction.
D. Red blood cells
D.Les globules rouges
436. Lequel des énoncés suivants à propos de l’intoxication par les
antidépresseurs tricycliques (ATC) est vrai ?
A. La concentration de l’ATC dans le sérum sanguin est un outil
diagnostique principal
B. L’ECG peut être utilisé pour prédire la probabilité de suite
neurologique grave et toxique
C. Un CT scan du cerveau doit être utilisé conjointement avec des
Which of the following is true in Tricyclic Antidepressants (TCA)
intoxication?
A. TCA blood serum concentration is a main diagnostic tool
B. ECG can be used to predict the likelihood of subsequent
serious neurologic (eg, seizure) toxicity
C. Brain CT scan must be used in conjunction with physical
findings to guide therapy.
D. Ipecac should be used as a first line treatment within 4
124
résultats physiques pour guider le traitement.
hours post ingestion
D. L’Ipeca devrait être utilisé comme un traitement de première
ligne dans les 4 heures après l'ingestion
437. Lequel des éléments suivants à propos de la phase deux de la
toxicité du Fer est vrai ?
Which of the following is true of Iron toxicity phase two?
A. It occurs 14-24 hours post ingestion
A. Elle se produit 14-24 heures après l'ingestion
B. It is usually asymptomatic
B. Elle est généralement asymptomatique
C. It consists of marked systemic toxicity caused by this
mitochondrial damage and hepatocellular injury
C. Elle se compose d’une toxicité systémique marquée et causée par
un dommage mitochondrial et lésion hépatocellulaire
D. Iron serum levels are increased
D. Les taux sériques du fer sont augmentés
438. Dans le traitement de l’intoxication avec les antidépresseurs
tricycliques (ATC), lequel des énoncés suivants est vrai?
In treating Tricyclic Antidepressants (TCA) intoxication which of
the following is true?
A. L’alcalinisation du sérum est en première ligne du traitement et la
thérapie la plus efficace pour les arythmies.
A. Serum alkalinization is the first-line and most effective
therapy for arrhythmias.
B. La lidocaïne est en première ligne du traitement et la thérapie la
plus efficace pour les arythmies.
B. Lidocaine is the first-line and most effective therapy for
arrhythmias.
C. Les Agents de classe1-A et 1-C (par exemple, procaïnamide,
disopyramide, quinidine, flécaïnide, encaïnide) sont des agents de
troisième ligne du traitement.
C. Class 1-A and 1-C agents (eg, procainamide, disopyramide,
quinidine, flecainide, encainide) are third-line agents
D. L’amiodarone est en première ligne du traitement et la thérapie
125
D. Amiodarone is the first-line and most effective therapy for
arrhythmias.
la plus efficace pour les arythmies.
439. Lequel des énoncés suivants à propos du traitement de la toxicité de
fer est vrai ?
Which of the following is true of Iron toxicity
management?
A. La décontamination avec l'irrigation du côlon entier et la
chélation utilisant l'administration intraveineuse de la déféroxamine
sont deux options utilisées dans le traitement de l'intoxication de
fer.
A.
Bowel decontamination with whole bowel
irrigation
and
chelation
using
intravenous
administration of deferoxamine are two options used in
iron intoxication treatment.
B. Gastric lavage is highly recommended in iron
intoxication treatment.
C. Hemoperfusion is useful in iron intoxication
treatment.
D. Serum alkalinization is the first-line and most
effective therapy in iron intoxication
B. Le lavage gastrique est fortement recommandé dans le
traitement de l'intoxication de fer.
C. L’Hémoperfusion est utile dans le traitement de l'intoxication par
le fer.
D. L’alcalinisation du sérum est la thérapie la plus efficace dans
l'intoxication du fer
440. Le toxidrome des stupéfiants comprend:
Narcotic toxidrome includes:
A. salivation
A. Salivation
B. Lacrymation
B. Lacrimation
C. Hypertension
C. Hypertension
D. Dépression respiratoire
D. Respiratory depression
441. Les patients pédiatriques moins de cinq ans semblent mieux
récupérer que les adultes après l'empoisonnement de
l'acétaminophène; le mécanisme est du au fait que:
A. Les patients pédiatriques ont un volume de la distribution plus
126
Pediatric patients younger than five years appear to recover
better than adults after Acetaminophen poisoning; the
mechanism is be due to the fact that:
élevé
A. Pediatrics have a higher volume of distribution
B. Les patients pédiatriques ont une désintoxication améliorée de
NAPQI
B. Pediatrics have an enhanced detoxification of NAPQI
C. Les patients pédiatriques ont une diminution dans les stocks de
glutathion
C. Pediatrics have a decrease in glutathione stores
D. Pediatrics have a higher metabolic CYP3A4 rate
D. Les patients pédiatriques ont un taux métabolique plus élevé du
CYP3A4
442. Une femme de 53 ans ingère la moitié d'une bouteille de comprimés
d'aspirine et se présente à l'urgence avec des vomissements et des
douleurs gastro-intestinaux. Le traitement de ce patient pourrait
inclure l'administration de:
A 53-year-old woman ingests half of a bottle of aspirin tablets
and later presents to the emergency department with
gastrointestinal pain and vomiting. The treatment of this
patient would include administration of:
A. bicarbonate de sodium, car il augmente l'ionisation du salicylate
dans l'urine
A. Sodium bicarbonate because it increases ionization of
salicylate in the urine
B. Bicarbonate de sodium, car il diminue l’ionisation du salicylate
dans l'urine
B. Sodium bicarbonate because it decreases ionization of
salicylate in the urine
C. L'acide ascorbique, car il augmente l'ionisation du salicylate dans
l'urine
C. Ascorbic acid because it increases ionization of salicylate in
the urine
D. L'acide ascorbique, car il diminue l’ionisation du salicylate dans
l'urine
D. Ascorbic acid because it decreases ionization of salicylate in
the urine
443. Une femme de 33 ans est amenée à l'urgence après avoir ingéré une A 33-year-old woman is brought to the emergency department
grande quantité mais inconnue de l'acétaminophène. L’Utilisation
after ingesting a large but unknown quantity of
de la N-acétylcystéine dans le traitement de ce patient est basée sur acetaminophen. Use of N-acetylcysteine in the treatment of
le but de:
this patient is based on the goal of:
127
A. inhiber l'absorption du médicament
A. Inhibiting drug absorption
B. Accroître l'élimination du médicament
B. Increasing drug elimination
C. inactiver un métabolite toxique
C. Inactivating a toxic metabolite
D. Diminution de récepteur cible de liaison
D. Decreasing target receptor binding
444. Un garçon de deux ans mangea 12 comprimés à croquer
d’acétaminophène possédant une saveur raisin qu'il les a trouvés
dans la salle de bain depuis deux heures. Il a déjà eu deux épisodes
de vomissements. Sa mère appelle le centre de poison et demande
des conseils. Elle affirme que son fils est maintenant ludique et "a
l'air bien." Quelles recommandations donneriez-vous à sa mère
(Supposons que vous pouvez recommander un traitement)?
A two- year old boy reportedly ate 12 grape flavored chewable
acetaminophen tablets that he found in the bathroom two
hours ago. He has already had two episodes of vomiting. His
mother calls the poison center and asks for advice. She states
that her son is now playful and "looks fine." What
recommendations would you give to his mother (Assume you
can recommend a treatment)?
A. amener le patient à l'urgence
A. Bring the patient to the ER
B. Offrir le charbon activé au patient
B. Give the patient activated charcoal
C. Offrir l'ipecac au patient
C. Give the patient ipecac
D. Rien ne doit être fait
D. Nothing needs to be done
445. Lequel des suivants est une cause d'origine médicamenteuse
secondaire potentiel de l'hypertension?
A.
B.
C.
D.
L'acétaminophène
Orlistat
Phénylpropanolamine
Le dextrométhorphane
Which of the following is a potential secondary drug-induced
cause of hypertension?
A. Acetaminophen
B.Orlistat
C. Phenylpropanolamine
D. Dextromethorphan
446. Lequel des patients suivants est à très haut risque de maladie
cardiovasculaire et devrait immédiatement recevoir un médicament
128
Which of the following patients is at very high risk for
cardiovascular disease and should immediately receive a
pour faire baisser la pression artérielle?
medication to lower blood pressure?
A) Une femme de 58 ans atteinte de diabète et une pression
artérielle de base de 128/75 mm Hg
A. A 58-year-old woman with diabetes and a baseline blood
pressure of 128/75 mm Hg
B. A 45-year-old man with uncomplicated hypertension and a
baseline blood pressure of 150/96 mm Hg
C. A 68-year-old man with heart failure and a baseline blood
pressure of 150/88 mm Hg
D. A 35-year-old man with dyslipidemia and a baseline blood
pressure of 150/984 mm Hg
B) Un homme de 45 ans avec hypertension non compliquée et une
pression artérielle de base de 150/96 mm Hg
C) Un homme de 68 ans avec une insuffisance cardiaque et une
pression artérielle de base de 150/88 mm Hg
D) Un homme de 35 ans a la dyslipidémie et une pression artérielle
de base de 150/984 mm Hg
447. Lequel des éléments suivants serait le plus approprié pour un
patient souffrant d'hypertension qui a eu un infarctus du myocarde?
A) La nifédipine
Which of the following would be most appropriate for a
patient with hypertension who has had a myocardial
infarction?
A. Nifedipine
B. Doxazosin
C. Metoprolol
D. Acebutolol
B) La doxazosine
C) Le Metoprolol
D) L’Acebutolol
448. Lequel des éléments suivants est l’objectif le plus aproprié de la
pression artérielle pour un homme de 65 ans afro-américain a
l'hypertension et pas d'autres problèmes médicaux?
Which of the following is the most appropriate blood pressure
goal for a 65-year-old African-American with hypertension and
no other medical problems?
A. <140/90 mm Hg
B. <130/85 mm Hg
C. <130/80 mm Hg
D. <125/75 mm Hg
A) <140/90 mm Hg
B) <130/85 mm Hg
C) <130/80 mm Hg
129
D) <125/75 mm Hg
449. Afin de minimiser le risque d'hypokaliémie de diurétiques, la
stratégie la plus appropriée inclurait
A) limitant la dose de l’hydrochlorothiazide de 12,5 à 25 mg.
B) en utilisant diurétiques de l'anse.
C) en utilisant des diurétiques épargneurs de potassium que la
thérapie primaire.
In order to minimize the risk of hypokalemia from diuretics,
the most appropriate strategy would include
A. limiting the dose of hydrochlorothiazide to 12.5 to 25 mg.
B. using loop diuretics.
C. using potassium-sparing diuretics as the primary therapy.
D. having all patients switch their dietary salt to potassium
chloride salt substitutes.
D) faire passer tous les patients du sel alimentaire aux substituts de
chlorure de potassium
450. Lequel des éléments suivants est un agent approprié et la dose de
départ pour un patient présentant une hypertension non
compliquée?
A) L'aténolol 25 mg une fois par jour
B) L'aténolol 100 mg une fois par jour
Which of the following is an appropriate agent and starting
dose for a patient with uncomplicated hypertension?
A. Atenolol 25 mg once daily
B. Atenolol 100 mg once daily
C. Propranolol 80 mg twice daily
D. Propranolol 120 mg twice daily
C) Le Propranolol 80 mg deux fois par jour
D) Le Propranolol 120 mg deux fois par jour
451. Quel médicament est le plus susceptible de causer des effets
secondaires de type vasodilatation (maux de tête, bouffées de
chaleur), car il bloque le mouvement du calcium dans les cellules
musculaires lisses?
A) Le Propranolol
130
Which drug is most likely to cause vasodilation-type side
effects (headache, flushing) because it blocks the movement of
calcium across smooth muscle cells?
A. Propranolol
B. Captopril
C. Clonidine
B) Le Captopril
D. Verapamil
C) L'énalapril
D) La Clonidine
E) Le vérapamil
452. Lorsque les patients présentant une insuffisance rénale développent
une toux sévère parmi les inhibiteurs de l'ECA, quelle serait la
meilleure solution pour fournir une protection rénale?
When patients with renal insufficiency develop severe cough
from ACE inhibitors, what would be the best alternative to
provide renal protection?
A. Propranolol
B. Hydrochlorothiazide
C. Losartan
D. Doxazosin
A) Le Propranolol
B) L’ Hydrochlorothiazide
C) Le Losartan
D) La doxazosine
453. Lequel des éléments suivants est vrai concernant la clonidine?
A) Elle a été utilisée pour le sevrage tabagique et le retrait
narcotique.
Which of the following is true concerning clonidine?
A. It has been used for smoking cessation and narcotic
withdrawal.
B. It can cause systemic lupus.
C. It can cause hemolytic anemia.
D. It has been used to treat benign prostatic hypertrophy.
B) Elle peut causer le lupus systémique.
C) Elle peut provoquer une anémie hémolytique.
D) Elle a été utilisée pour traiter l'hypertrophie bénigne de la
prostate
454. Quel agent provoque la rétention d'eau et la tachycardie de telle
sorte qu'il devrait être utilisé conjointement avec un diurétique et
131
Which agent causes so much fluid retention and tachycardia
that it should be used along with a diuretic and a beta blocker?
A. Diltiazem
B. Minoxidil
C. Captopril
D. Losartan
un bêta-bloquant?
A) Diltiazem
B) Minoxidil
C) Captopril
D) Losartan
455. Lequel des éléments suivants est une contre-indication à l'énalapril?
Which of the following is a contraindication to enalapril?
A. Bilateral renal artery stenosis
B. A pheochromocytoma
C. Undetected diabetes
D. A family history of end-stage renal failure
A) une sténose de l'artère rénale bilatérale
B) Un phéochromocytome
C) le diabète non détecté
D) Une histoire de la famille de l'insuffisance rénale au stade
terminal
456. Quels effets secondaires potentiels devez-vous mentionner à un
patient ayant récemment commencé sur la nifédipine?
What potential side effects would you mention to a patient
recently started on nifedipine?
A. Increased hair growth (hirsutism)
B. Painful or swollen breasts (gynecomastia)
C. Systemic lupus
D. Leg edema
A) L’augmentation de la pilosité (hirsutisme)
B) de poitrines gonflées ou douloureuses (gynécomastie)
C) Le lupus érythémateux
D) œdème des membres inferieurs
457. Quel médicament est plus susceptible de provoquer une
vasodilatation car il bloque les récepteurs alpha post-synaptiques?
132
Which drug is most likely to cause vasodilation because it
blocks postsynaptic alpha receptors?
A. Propranolol
B. Prazosin
C. Verapamil
D. Clonidine
A) Propranolol
B) Prazosin
C) vérapamil
D) Clonidine
458. Une femme de 35 ans hypertendue, prenant HCTZ, l'énalapril, et le
diltiazem . Elle désire devenir enceinte. L'étape la plus importante
serait de
A 39-year-old woman with hypertension is taking HCTZ,
enalapril and diltiazem. She desires to become pregnant. The
most important step would be to
A. switch HCTZ to furosemide .
B. switch diltiazem to amlodipine
C. discontinue diltiazem.
D. discontinue enalapril.
A) changer l’HCTZ au furosémide
B) changer le diltiazem à l'amlodipine
C) discontinuer le diltiazem.
D) discontinuer l'énalapril.
459. Laquelle des mesures non médicamenteuses suivantes est
susceptible d'être plus efficace pour abaisser la tension artérielle
chronique?
A) Réduire la caféine
B) Lancer le biofeedback
Which one of the following nondrug measures is likely to be
most effective to lower blood pressure chronically?
A. Reduce caffeine
B. Start biofeedback
C. Lose 7 Kg
D. Begin garlic capsules
C) diminuerle poids de 7 kilos
D) Commencer capsules d'ail
460. L'agent de choix pour un patient souffrant de diabète est
The agent of choice for a patient with diabetes is
A. propranolol.
133
B. doxazosin.
C. clonidine.
D. enalapril.
A) propranolol.
B) doxazosine.
D) la clonidine.
E) l'énalapril
461. Lors d'une intoxication aux chlorates, une methémoglobinemie de In chlorates poisoning, a 20% methemoglobinemia was
20% a été observée. Le traitement préconisé est :
observed. The recommended treatment is:
A- oxygenothérapie hyperbare
A- hyperbaric oxygen
B- administration d'une solution de bleu de méthylène a 1% par B- the administration of methylene blue 1% solution IV
voie IV
C- the administration of a sodium thiosulfate solution
C- administration d'une solution de thiosulfate de sodium
D- the administration of Glutathion
D- administration de Glutathione
462. La chlorpromazine :
Chlorpromazine:
A- provoque une anémie hémolytique par action directe sur A- causes hemolytic anemia by direct action on the red blood
l'hématie
cell
B- entraîne une agranulocytose
B- causes agranulocytosis
C- provoque une pancytopénie
C- causes pancytopenia
D- provoque une anémie
immunoallergique
hémolytique
par
mécanisme D- causes hemolytic anemia via an immunoallergic mechanism
463. Parmi les toxiques responsables de la thrombocytopénie, on cite :
134
Which of the following toxic substances causes
thrombocytopenia:
A- acétaminophène
A- acetaminophen
B- acénocoumarine
B- acenocoumarin
C- As203
C- As203
D- Vancomycine
D-Vancomycin
464. Une toxicité hépatique imprévisible est caractérisée par :
Unpredictable hepatic toxicity is characterized by:
A- une période de latence courte
A- a short latency period
B- une récidive lente après ré administration
B- slow recurrence after re administration
C- une incidence d'apparition faible
C- a low incidence of appearance
D- une relation dose- effet
D- a dose effect relationship
465. L’atteinte de la bordure en brosse du tubule proximal est évaluée The involvement of the proximal tubular brush border is
par le dosage de :
assessed by the quantitative determination of:
A- la LDH.
A- LDH.
B- la NAG.
B- NAG.
C- la Gamma GT.
C- Gamma GT.
D- la Glutamate déshydrogénase
D- glutamate dehydrogenase.
466. La toxicité rénale de la gentamycine est évaluée essentiellement par Renal toxicity of gentamicin is essentially evaluated by
dosage de :
assaying:
A- β D glucosidase.
A- D- β-glucosidase.
B- phosphatase alcaIine
B- alKaIine phosphatase
135
C- NAG.
C- NAG
D- leucine aminopeptidase
D- leucine aminopeptidase
467. L 'efficacité optimale du charbon activé est obtenue :
The optimum efficiency of active charcoal is obtained:
A- après administration de sirop d'ipéca
A- after the administration of syrup of ipecac
B- 2 heures après intoxication par voie orale
B- 2 hours after oral intoxication
C- 30 minutes après intoxication par voie orale
C- 30 minutes after oral intoxication
D- en association avec la cholestyramine
D- in combination with cholestyramine
468. Pour induire rapidement des vomissements ( 1 à 3 minutes ), il faut To quickly induce vomiting (1-3 minutes), use:
utiliser :
A- apomorphine
A- I'apomorphine
B- syrup of ipecac
B- le sirop d'ipéca
C- BAL
C- le BAL
D- sodium chloride
D- le chlorure de sodium
469. Les méthodes d'épuration extra rénales peuvent :
Renal extra purification methods can:
A- S'appliquer aux poisons cytotoxiques
A- Apply to cytotoxic poisons
B- Eliminer une substance de masse moléculaire élevée > 1500
B- Eliminate high molecular weight substances> 1500
C- Etre appliquées aux substances ayant un volume de distribution C- Be applied to substances having a low volume of
faible
distribution
D- Etre appliquées aux substances ayant un volume de distribution D- Be applied to substances with a high volume of distribution
136
élevée
470. La nécrose péri-portale est provoquée expérimentalement par:
The periportal necrosis is experimentally caused by:
A- La galactosamine
A- Galactosamine
B- L’acétyl hydrazine
B- Acetyl hydrazine
C- L’acétaminophène
C- Acetaminophen
D- L’alcool allylique
D- Allyl alcohol
471. Le lavage d’estomac est contre indiqué dans les cas suivants sauf Gastric lavage is contra-indicated in the following cases except
un :
one:
A- Ingestion de Flash ( produit ménager contenant de la soude )
A-Ingestion of Flash
B- Ingestion de détergents
(household product containing sodium hydroxide)
C- Ingestion de paracetamol
B- Ingestion of detergents
D- Coma
C- Ingestion of paracetamol
D- Coma
472. Le Bleu de méthylène :
Methylene blue:
I. Est utilisé dans le traitement de méthémoglobinémies I. Is used in the treatment of methemoglobinemia higher than
supérieures à 10 %
10%
II. Est un agent méthémoglobinisant
II. Induces methemoglobinemia
III. Est disponible en solution de 5%
III. Is available as a solution of 5%
IV. Agit par l’intermédiaire de son Leucodérivé
IV. Acts through its leuco derivatives
137
V. Est à éviter chez les sujets cirrhotiques
V. Is avoided in cirrhotic subjects
La réponse correcte est la suivante :
The correct answer is:
A- I , III
A- I and III
B- II , IV
B- II and IV
C-I , IV
C-I and IV
D- III , V
D- III and V
473. La phase de saturnisme est caractérisée chez l’enfant par :
Lead poisoning in children is characterized by:
A- L’apparition d’un liseré de Burton
A- The appearance of a Burton border
B- Anémie hypochrome et macrocytaire
B- macrocytic hypochromic and anemia
C- Troubles gastro-intestinaux : diarrhée
C- Gastrointestinal disorders: diarrhea
D- Troubles nerveux : Encéphalopathie
D- nervous disorders: encephalopathy
474. La Buprénorphine:
Buprenorphine:
A- Est un morphinomimétique à structure héxacyclique
A- is an morphinomimetic to hexacyclic structure
B- Est un antagoniste morphinique
B- is an opioid antagonist
C- Est un morphinomimétique à structure diphénylpropylamine
C Is a morphinomimetic to diphenylpropylamine structure
D- Est une phénylpipéridine à structure tétracyclique
D- Is a phenylpiperidine to tetracyclic structure
475. La dépression respiratoire rencontrée lors d'une intoxication aux The respiratory depression encountered during a salicylate
138
salicylés est observée pour des concentrations :
poisoning is observed at a concentration of:
A- < 450 mg/L
A- <450 mg / L
B- < 650 mg/L
B- <650 mg / L
C- > 650 et < 900 mg/L
C-> 650 and <900 mg / L
D- > 900 mg/L
D-> 900 mg / L (+)
476. Les troubles cardiovasculaires observés lors d'une intoxication à Cardiovascular disorders observed in poisoning with ethylene
l'éthylène glycole:
glycol:
A- Correspondent à des lésions provoquées par des Inclusions de A- correspond to lesions caused by inclusions of calcium
cristaux d’oxalate de calcium
oxalate crystals
B- Sont toujours bénins : tachycardie et hypotension modérée
B- are always benign tachycardia and moderate hypotension
C- Débutent dans les 4 heures qui suivent l'exposition
C- begin within 4 hours of exposure
D- Suivent toujours l'apparition d'un œdème pulmonaire
D- always follow the onset of pulmonary edema
477. Un sujet sombrant dans un état comateux a été admis d'urgence à
l'hôpital. Il présente une acidose métabolique et une méthanolémie
de 0.65 g/L. On procède à la correction de l'acidose suivie de
l'épuration du toxique dont la méthode de choix s'avère être dans
ce cas:
A. L'hémoperfusion
A subject in coma was admitted to the hospital emergency
department. He presented with metabolic acidosis and
methanolemia 0.65 g / L. The correction of acidosis followed
by detoxification should be done by:
A. hemoperfusion
B. Peritoneal dialysis
B. La dialyse péritonéale
C. The administration of diuretics
C. L'administration de diurétiques
D. Hemodialysis
139
D. L'hémodialyse
478. Un surdosage aux benzodiazépines est caractérisé par les signes An overdose of benzodiazepines is characterized by the
cliniques suivants :
following clinical signs:
A. Contractions musculaires et fasciculation
A. Muscle twitching and fasciculation
B. Hallucinations et confusion mentale
B. Hallucinations and confusion
C. Hyperventilation et tachycardie
C. Hyperventilation and tachycardia
D. Somnolence et obnubilation
D. Drowsiness and clouding
479. Le délirium tremens correspondant aux symptômes aigus d'arrêt Delirium tremens corresponding to acute symptoms of alcohol
d'absorption alcoolique est caractérisé par les signes cliniques malabsorption is characterized by the following clinical signs:
suivants :
A. Slow Pulse
A. Ralentissement du pouls
B. High hyperthermia
B. Excès d’hyperthermie
C. Transient hallucinations that are often nocturnal
C. Hallucinations transitoires souvent nocturnes
D. Ocular paralysis
D. Une paralysie oculaire
480. Un des signes cliniques suivants est constamment observé lors One of the following clinical signs is consistently observed at a
d’une salicylémie de 700 mg/L :
salicylic plasma level of 700 mg / L:
A- Hyperthermie
A- hyperthermia
B- Œdème pulmonaire
B- Pulmonary edema
C- Convulsions
C- Convulsions
140
D- Collapsus cardio-vasculaire
D- Circulatory collapse
481. Alcoolisme : La pathologie de surcharge :
Alcoholism: The overload condition:
A. Est observée lorsque l'ingestion d'alcool atteint 50% de la ration A. Is observed when alcohol intake reaches 50% of caloric
calorique
intake
B. Correspond
à
une
dénutrition
physiopathologique complexe
de
mécanisme B. Represents a pathophysiological complex of malnutrition
C. Is related to an increase of triglyceride synthesis
C. Est liée à une augmentation de la synthèse des triglycérides
D. Is never associated with hepatic steatosis, first step in the
D. N’est jamais associée à une stéatose hépatique première étape progression to cirrhosis
de l'évolution vers la cirrhose
482. Un sujet âgé a été admis en urgence dans un centre hospitalier. Il se
trouve dans un état comateux avec dépression respiratoire et
cardio-vasculaire et chute de la pression artérielle. L’examen des
urines par CPG-MS a révélé la présence de 2- amino – 5nitrobenzophénone. On conclut à l’absorption de doses massives
de :
A- Méthylparathion
An elderly was urgently admitted to a hospital. He is in a coma
with respiratory and cardiovascular depression and drop in
blood pressure. Urinalysis by GC-MS revealed the presence of
2-amino - 5-nitrobenzophenone. It is concluded as the
absorption of massive doses of:
A- methyl parathion
B- Flunitrazepam
B- Flunitrazepam
C-Nitrazepam
C- Nitrazepam
D- Flunitrazepam or Nitrazepam
D- Flunitrazepam ou Nitrazepam
483. Le traitement d'une intoxication par un antidépresseur tricyclique se The treatment of poisoning with tricyclic antidepressants is:
fait :
A. Gastric lavage and administration of activated charcoal
141
A. Lavage gastrique et administration de charbon activé
B. Administration of acetylcysteine
B. Administration d’acétylcystéine
C. extrarenal Treatment
C. Epuration extra-rénale
D. massive Alkalinization by THAM
D. Alcalinisation massive par le THAM
484. Quel trouble acido-basique est habituellement observé au début What acid-base disorder is usually observed at the start of
d'une intoxication aigue par les salicylés ?
acute poisoning with salicylates?
A- Acidose respiratoire
A- respiratory acidosis
B- Acidose mixte
B- Joint Acidosis
C- Alcalose respiratoire
C- respiratory alkalosis
D- Acidose métabolique
D- Metabolic Acidosis
485. Laquelle des propositions suivantes concernant la biodisponibilité
du médicament est correcte ?
ABCD-
Elle est strictement dépendante de la dose
Elle dépend de la voie d'administration
Elle est augmentée par l'effet de premier passage hépatique
Elle reste la même après un changement d'excipient
486. Toutes ces assertions concernant la ciclosporine sont correctes à
l’exception de :
ABCD-
C’est un immunosuppresseur très lipophile
Elle a une forte affinité pour les érythrocytes
Elle a une faible biodisponibilité
Son élimination est essentiellement hépatique
Which of the following statements concerning the
bioavailability of a drug is correct?
ABCD-
It is strictly dose-dependent
It depends on the route of administration
It is increased by the first pass hepatic effect
It remains the same after a change of excipient
All of these statements concerning cyclosporine are corrects
EXCEPT :
A- It is a highly lipophilic immunosuppressive drug
B- It has a strong affinity for the erythrocyte
142
C- It has low bioavailability
D- It has essentially a hepatic elimination
487. Le suivi thérapeutique plasmatique est recommandé pour tous ces
médicaments à l’exception des :
ABCD-
Antiépileptiques
Aminosides
Digitaliques
Anti vitamines K
The Therapeutic Drug Monitoring is recommended for all the
below drugs EXCEPT:
A- Antiepileptics
B- Aminoglycosides
C- Digitalis
D- Anti-vitamin K
488. La zone thérapeutique de la digoxine chez l’adulte varie de :
ABCD-
The therapeutic range for digoxin in adults is:
0,8 à 2 ng/mL
0,8 à 2 mg/mL
1 à 2,5 ng/mL
1,5 à 4 mg/mL
A- 0.8 to 2 ng / mL
B- 0.8 to 2 mg / mL
C-1 to 2.5 ng / mL
D 1.5 to 4 mg / mL
489. Laquelle des propositions suivantes concernant la vancomycine est
CORRECTE?
ABCD-
C'est un antibiotique à large spectre
Elle possède une demi-vie supérieure à 50 heures
Elle est active sur le staphylocoque doré méthicilline-résistant
Elle est hématotoxique
Which of the following statements of Vancomycin is CORRECT?
A- It is a Broad-Spectrum Antibiotic
B- It has a half-life greater than 50 hours
C- It is active against MRSA
D- It is hematotoxic
143
Phenytoin is characterized by all of the followings EXECPT:
A- Phenytoin is not bound to albumin
B- It is mainly eliminated via CYP2C9
C- Its clearance is concentration-dependant
D- It induces a concentration dependant side effects
490. La Phenytoine se caracterise par tout le suivants SAUF :
A- La phénytoïne est non liée à l'albumine
B- Est principalement éliminée par le CYP2C9
C- Sa clairance est concentration-dépendante
Elle produit des effets indésirables concentration-dépendantes
491. Un médicament M décrit par un modèle monocompartimental, est
administré à deux patients A et B sous forme de perfusion
intraveineuse à vitesse constante. Une dose de charge est
seulement donnée au patient A (Css, Q, Vd). A quel moment l’état
d’équilibre des concentrations plasmatiques du médicament est
obtenu pour A ?
ABCD-
Immédiatement
Après 2 demi-vies
Après 5 à 7 demi-vies
Ceci dépend de la demi-vie et de la vitesse de perfusion de M
Two patients are receiving a constant I.V. infusion of a drug M.
Patient A received a loading dose (Css Q, Vd), while Patient B
did not. Assume that the drug disposition can be described by
a one compartment model. At what time would steady-state
plasma concentrations of the drug be obtained for Subject A?
A- Immediately
B- After 2 half-lives
C- After 7 half-lives
D- It would depend on the half-life and infusion rate of the
drug M
492. Tous ces changements physiologiques peuvent survenir chez les
patients âgés, SAUF:
All the following physiologic changes may occur in elderly
patients EXCEPT:
A- La réduction du taux de filtration glomérulaire
A- A decrease in GFR
B- La réduction du pH gastrique
B- A decrease in gastric pH
C- La réduction de l'albumine sérique
C- A decrease in serum albumin
144
D- La réduction de la masse hépatique
D- A decrease in hepatic mass
493. Quelle serait la dose d’un médicament à administrer pour obtenir What would be the dose of a drug to be administered to
une concentration cible de 10g/L si son volume de distribution (Vd) achieve a target concentration of 10 g/L if the volume of
distribution (Vd) is 350 liters?
est de 350 litres?
A- 350 g
A- 350 g
B- 350 mg
B- 350 mg
C- 3.5 mg
C- 3.5 mg
D- 35 g
D- 35 g
494. La demi-vie d’élimination de sulfate de quinidine (F=0.8, réaction
d’ordre 1), est presque 6h. Quel serait le pourcentage du
médicament restant dans le corps 12 heures après l’administration
de 100 mg IV bolus:
The elimination half-life of quinidine sulfate (F = 0.8, 1st order
reaction) is almost 6h. What would be the percentage of the
drug remaining in the body 12 hours after the administration
of 100 mg IV bolus:
ABCD-
12.5%
A- 25%
B- 50%
C- 75%
12.5%
25%
50%
75%
495. Un médicament est administré par IV bolus à la dose de 40 mg. Sa A 40mg of a drug is administered by IV bolus. Its initial plasma
concentration plasmatique initiale (Co) est de 5mg/L. Sa demi-vie concentration (Co) is 5 mg/L. Its plasmatic half-life is 25 hours.
d'élimination plasmatique est de 25 heures. Le volume de The volume of distribution of this drug is:
A- 125 liters
distribution de ce médicament est :
B- 25 liters
A- 125 litres
C- 8 liters
D- 5 liters
145
B- 25 litres
C- 8 litres
D- 5 litres
496. Toutes ces propositions concernant l’isoniazide sont correctes
SAUF :
A- Donne essentiellement un métabolite inactif
All the following concerning isoniazid are correct EXCEPT:
A- It mainly produces an inactive metabolite
B- It presents a polymorphism in acetylation
B- Présente un polymorphisme d’acétylation
C- The dosage adjustment is primarily based on its t½
C- Son adaptation posologique est essentiellement fondée sur la
t½
D- It induces a risk of hepatotoxicity in poor metabolizers
D- induit un risque d’hépatotoxique chez les métaboliseurs lents
497. Laquelle des modifications suivantes, est observée chez la femme
enceinte:
Which of the following changes is observed in pregnant
women:
A- Diminution du débit sanguine pulmonaire
A- The pulmonary blood flow is decreased
B- Augmentation significative des réactions métaboliques de phase
II
B- A significant increase in phase II metabolic reactions
C- Augmentation significative de volume apparent de distribution
C- A significant increase in the apparent volume of
distribution
D- Filtration glomérulaire souvent non altérée
D- GFR is usually not altered
498. Tout est vrai en ce qui concerne le passage des médicaments de la
mère vers l’enfant via le lait maternel, SAUF :
A- Seule la forme non ionisée des médicaments diffuse dans le
lait à travers les membranes biologiques.
146
All the following concerning the passage of a drug from the
mother to the child via the mother's milk is true, EXCEPT:
B- Les médicaments acides faibles passent plus facilement que
les bases faibles.
C- Les bases faibles se trouvent piégées dans le compartiment
lacté
D- Le débit sanguin augmente au cours de la tété
A- Only the unionized form of the drug diffuses into the milk
through biological membranes.
B- Weak acid drugs pass more easily than weak bases.
C- Weak bases are trapped in the milk compartment
D- Blood flow increases during breastfeeding
499. La tératogénicité de tous ces médicaments a été démontrée SAUF The teratogenicity of all the following drugs has been shown
pour?
EXCEPT for?
ABCD-
Lithium
Acyclovir
Isotrétinoïne
Acide valproïque
A- Lithium
B- Acyclovir
C- Isotretinoïn
D- Valproic Acid
500. Le traitement antipsychotique de 1er choix chez la femme enceinte
est :
A- L’halopéridol
B- L’olanzapine
C- La risperidone
D- La clozapine
The first- choice antipsychotic treatment in pregnant women
is:
A.
B.
C.
D.
Haloperidol
Olanzapine
Risperidone
Clozapine
501. Lequel parmi ces médicament est préconisé antiulcéreux de 1ere Which of the following drugs is recommended as first antiintention chez la femme enceinte :
ulcer line treatment in pregnant women:
A- Hydroxyde d’Al +Hydroxyde de Mg
B- Alginate
C- Omeprazole
A- Mg hydroxide + Al hydroxide
147
D- Misoprostol
B- Alginate
C- Omeprazole
D- Misoprostol
502. Lequel parmi ces antiulcéreux peut être utilisé pour traiter l’ulcère Which of the following anti-ulcer drugs can be used to treat
gastrique chez la femme enceinte :
gastric ulcers in pregnant women:
ABCD-
Ranitidine
Omeprazole
Rabeprazole
Esomeprazole
A- Ranitidine
B- Omeprazole
C- Rabeprazole
Esomeprazole
503. Si une femme suit déjà un traitement par la tyroxine, lorsqu’elle If a woman has been treated with thyroxine, when she
tombe enceinte elle doit :
becomes pregnant, she must:
A- Arrêter immédiatement le traitement
A- Stop immediately the treatment
B- Arrêter progressivement le traitement pour éviter un état de
B- Stop processing to avoid kickback condition
rebond
C- Continuer la Tyroxine durant toute la grossesse
C- Continue thyroxine throughout pregnancy
D- Effectuer une interruption de la grossesse
D- Do abortion
504. Lequel parmi ces produits est un traitement de choix de la Which of the following products is a preferred treatment for
tyrotoxicose chez la femme enceinte :
tyrotoxicosis in pregnant women:
A- Métamizole
B- Propranolol
A- Metamizol
148
C- Iode non radioactif
D- Propyl- thio-uracile (PTU)
B- Propranolol
C- Non-radioactive iodine
D- Propyl ThioUracil (PTU)
505. Un garçon de 17 ans a pris un comprimé de naproxène pour son mal A 17-year-old boy took a tablet of naproxen for headache.
de tête. Le naproxène est un acide faible ayant un pKa de 5,2. Quel Naproxen is a weak acid with a pKa of 5.2. What percentage of
est probablement le pourcentage du médicament solubilise dans le the drug is most likely water soluble in the patient’s plasma?
plasma du patient?
A- 1%
A- 1%
B- 50%
B- 50%
C- 70%
C- 70%
D- > 99%
D- > 99%
506. Un garçon de 12 ans récemment diagnostiqué de diabète de type 1,
a commencé un traitement avec deux administrations souscutanées quotidiennes d'insuline. Lequel des processus de transport
suivants, explique au mieux l'absorption de l'insuline à partir du site
d'injection?
ABCD-
Diffusion passive
Diffusion facilitée
Endocytose
Transport actif
A 12-year-old boy recently diagnosed with type 1 diabetes
started a therapy with two daily subcutaneous administrations
of insulin. Which of the following transport mechanisms best
explains the absorption of insulin from the site of injection?
A- Passive diffusion
B- Facilitated diffusion
C- Endocytosis
D- Active transport
149
507. Une femme de 22 ans souffrant d’asthme est traitée par salbutamol
par inhalation. Ce dernier est un bronchodilatateur ayant un poids
moléculaire de 239 Daltons. Lequel des processus de passage
suivants, explique au mieux le transfert du médicament à travers la
muqueuse bronchique?
A 22-year-old woman suffering from asthma was prescribed
albuterol by inhalation. Albuterol is a bronchodilator with a
molecular weight of 239 Daltons. Which of the following
transport mechanisms most likely accounts for the transfer of
the drug through the bronchial mucosa?
A- Diffusion aqueuse
A- Aqueous diffusion
B- Diffusion lipidique
B- Lipid diffusion
C- Diffusion facilitée
C- Facilitated diffusion
D- Transport actif
D- Active transport
508. Une femme âgée de 36 traitée par métronidazole pour son infection
à trichomonas. Le médicament a une clairance hépatique de 4,86 L
/h et une clairance rénale de 0,54 l /h. Dans l'hypothèse que seuls le
foie et les reins sont impliqués dans l'élimination de ce médicament,
quel est le pourcentage du médicament administré qui sera éliminé
par le foie?
ABCD-
20%
50%
70%
90%
A 36-year-old woman recently treated with metronidazole for
her trichomoniasis. The drug has a hepatic clearance of 4.86
L/h and a renal clearance of 0.54 L/h. On the assumption that
only the liver and kidney are involved in the elimination of this
drug, what percentage of the administered drug will be
eliminated by the liver?
A- 20%
B- 50%
C- 70%
D- 90%
509. Un homme japonais de 59 ans ayant une fibrillation auriculaire s’est
présenté à son médecin se plaignant d’une coloration rouge des
urines. L'homme avait reçu une dose standard de warfarine, un
anticoagulant métabolisé par l’isoenzyme CYP2C9. Laquelle des
150
A 59-year-old Japanese man with atrial fibrillation presented to
his physician complaining of red urine. The man had been
receiving a standard dose of warfarin that is an anticoagulant
drug biotransformed by CYP2C9 isozyme. Which of the
propositions suivantes est la cause la plus probable du trouble de following is the most likely cause of the patient’s problem?
patient?
A- Increased protein binding of warfarin
A- Augmentation de la fixation protéique de la warfarine
B- Decreased renal excretion of warfarin
B- Diminution ion de l’excrétion rénale de la warfarine
C- Genetic polymorphism of CYP2C9
C- Polymorphisme génétique du CYP2C9
D- Increased CYP2C9 synthesis in a person of Asian origin
D- Augmentation de la synthèse du CYP2C9 chez une personne
d'origine asiatique
510. La pharmacocinétique d'un nouveau médicament décrit par une
réaction d’ordre 1, un modèle monocompartimental est étudiée
chez un volontaire sain. Une dose de 20 mg a été administrée par
voie intraveineuse. La concentration plasmatique du médicament
variait de 2 mg /L initialement à 1 mg /L 2 heures plus tard. Quel est
probablement le volume de distribution du médicament (en litres)?
511.
The pharmacokinetics of a new drug that follows first- order,
one-compartment model kinetics was studied in a healthy
volunteer. A 20-mg dose was given intravenously. The plasma
concentration of the drug turned out to be 2 mg/L initially and
1 mg/L 2 hours later. What is the most likely volume of
distribution of the drug (in liters)?
A- 10
A- 10
B- 5
B- 5
C- 40
C- 40
D- 20
D- 20
Un homme volontaire sain, de 23 ans a reçu une dose
intraveineuse de 2 mg d'un nouveau médicament au cours d'un
essai clinique. Le médicament décrit par une réaction de 1er ordre
et un modèle cinétique à monocompartimental, avait un volume
de distribution de 10 L. Après 6 heures, la concentration
plasmatique du médicament était de 50 g /L. Laquelle parmi les
151
A 23-year-old healthy volunteer male received an intravenous
dose of 2 mg of a new drug during a clinical trial. The drug
follows first-order, one-compartment model kinetics and has
a volume of distribution of 10 L. After 6 hours the plasma
concentration of the drug was 50 μg/ L. Which of the
suivantes, est probablement la demi-vie du médicament ?
following is most likely the half-life of the drug
A- 1 heure
A- 1hour
B- 3 heures
B- 3 hours
C- 4 heures
C- 4 hours
D- 6 heures
D- 6 hours
512. La pharmacocinétique d'un nouveau médicament a été étudiée
chez des volontaires sains. Il a été constaté que le médicament suit
une réaction de 1er ordre et un modèle cinétique
monocompartimental Son volume de distribution était de 100 L.
Après administration orale de 200 mg, la concentration plasmatique
théoriquement au temps 0 s’est avérée 1 mg / L. Laquelle parmi les
suivantes, est probablement la biodisponibilité du médicament ?
ABCD-
0,1
0,5
0,8
1.0
The pharmacokinetics of a new drug was studied in healthy
volunteers. It was found that the drug follows first-order, onecompartment model kinetics and has a volume of distribution
of 100 L. After the oral administration of 200 mg, the
theoretical plasma concentration at time 0 turned out to be 1
mg/L. Which of the following is most likely the oral
bioavailability of the drug?
A- 0.1
B- 0.5
C- 0.8
D- 1.0
513. Un homme de 22 ans, souffrant d'autisme adulte et d’un
comportement violent, a commencé un traitement qui comprenait
buspirone, un médicament ayant grand effet de premier passage
hépatique. Laquelle des propriétés pharmacocinétiques suivantes
152
A 22-year-old man suffering from adult autism and violent
behavior started a treatment that included buspirone, a drug
with a large first-pass effect. Which of the following
pharmacokinetic properties of the drug is most likely affected
du médicament est probablement touchée par cet effet de premier by this large first-pass effect?
passage élevé?
A- Volume of distribution
A- Le volume de distribution
B- Oral bioavailability
B- La biodisponibilité orale
C- Renal clearance
C- Clairance rénale
D- Sublingual bioavailability
D- Biodisponibilité sublinguale
514. Une femme âgée de 52 ans souffrant de polyarthrite rhumatoïde a
débuté un traitement qui comprenait Infliximab, un anticorps
monoclonal dirigé contre le facteur de necrose tumorale-α (TNF-α).
Le médicament avait un volume de distribution d'environ 3 L. Lequel
parmi les suivants, représente probablement le principal site de
distribution de ce médicament?
A- Tissus gras
A 52-year-old woman suffering from rheumatoid arthritis
started a treatment that included Infliximab, a monoclonal
antibody against tumor necrosis factor-α (TNF-α). The drug has
a volume of distribution of about 3 L. Which of the following is
most likely the main site of distribution of this drug?
A- Fat tissue
B- Plasma
B- Plasma
C- Extracellular fluids
C- Fluides extracellulaires
D- Cell cytosol
D- Cytosol cellulaire
515. Un homme de 63 ans, récemment diagnostiqué ayant une
hypertension, a commencé une thérapie avec HCTZ, un
comprimé/jour. L’HCTZ est une substance médicamenteuse acide
ayant un pKa d'environ 9. Laquelle des parties de tube digestif
suivantes représente probablement le site principal d'absorption de
ce médicament?
153
A 63-year old man was recently diagnosed with hypertension;
he started a treatment of HCTZ, one tablet daily. HCTZ is an
acidic drug with a pKa of about 9. Which of the following
parts of the digestive tract most likely represents the main
site of absorption of that drug?
A- Estomac
A- Stomach
B- Colon
B- Colon
C- Rectum
C- Rectum
D- Intestin grêle
D- Small intestine
516. Une femme de 69 ans est amenée aux urgences de l'hôpital local par
son fils, qui a déclaré que sa mère était léthargique, désorientée, et
combative quelques heures plus tôt. De plus, elle avait ingéré un
grand nombre de comprimés d'aspirine comme tentative de
suicide. Un traitement approprié a été instauré, comprenant
l'administration de bicarbonate de sodium pour augmenter
l'élimination du salicylate. Laquelle des propositions suivantes,
explique au mieux le mécanisme de cette élimination accrue?
ABCD-
Diminution de transport actif tubulaire du salicylate
Diminution de la biodisponibilité du salicylate
Piégeage des ions salicylés dans les urines
Augmentation de la filtration glomérulaire des salicylates
A 69-year-old woman was brought to a local hospital
emergency department by her son, who reported that his
mother was found lethargic, disoriented, and combative a
few hours earlier. In addition, she had ingested a large
number of aspirin tablets in a suicide attempt. An appropriate
therapy was instituted, which included the administration of
sodium bicarbonate to increase the elimination of salicylate.
Which of the following best explains the mechanism of the
increased elimination?
A- Decreased tubular active transport of salicylate
B- Decreased bioavailability of salicylate
C- Urinary ion trapping of salicylate
D- Increased glomerular filtration of salicylate
517. Un homme âgé de 44 ans, a pris une forte dose d'acétaminophène
en solution aqueuse pour traiter une forte céphalée. Deux heures
plus tard, la douleur n’est pas diminuée. Puisque l'acétaminophène
doit être efficace dans environ 30 minutes après son administration
orale, laquelle des conditions suivantes, retarde probablement son
absorption orale chez cette patient?
154
A 44-year-old man took a large dose of acetaminophen in
aqueous solution to treat a strong headache. Two hours later,
the pain was not still the same. Because acetaminophen
should be effective in about 30 minutes after its oral
administration, which of the following conditions most likely
delayed the oral absorption of the drug in this patient?
ABCD-
Une augmentation modérée de péristaltisme intestinal
La présence d'une forte douleur
Un grand volume de distribution du médicament
Une très faible clairance du médicament
A- A moderate increase in intestinal peristalsis
B- The presence of a strong pain
C- A large volume of distribution of the drug
D- A very low clearance of the drug
518. Un garçon de 4 ans souffrant de leucémie lymphoblastique aiguë
était sur le point de recevoir une injection intrathécale de
méthotrexate, un médicament qui ne peut pas traverser la barrière
hémato-encéphalique. Lequel des énoncés suivants, explique au
mieux la mauvaise pénétration du méthotrexate dans le cerveau?
A- La pression du fluide céphalo-rachidien est inférieure à la
pression de perfusion cérébrale.
B- Le pH du liquide céphalo-rachidien est inférieur à celui du
plasma.
C- Les cellules endothéliales des capillaires cérébraux et le plexus
choroïde ont des jonctions serrées.
D- Le méthotrexate est complètement ionisés dans le sang.
A 4-year-old boy suffering from acute lymphoblastic leukemia
was about to receive an intrathecal injection of methotrexate,
a drug that cannot cross the blood−brain barrier. Which of the
following statements best explains why methotrexate, cannot
easily enter the brain?
A- Cerebrospinal fluid pressure is lower than cerebral
perfusion pressure.
B- The pH of cerebrospinal fluid is lower than plasma pH.
C- Endothelial cells of brain capillaries and choroid plexus
have tight junctions.
D- Methotrexate is completely ionized in blood.
519. L’absorption des médicaments est influencée par l’alimentation. Drug absorption is affected by food intake. Which of these
Laquelle parmi ces interactions Médicament-Aliment est incorrecte : Food-Drug interactions is incorrect:
A- Le thé retarde l’absorption du Fer
B- Le lait ralentie l’absorption du Fluor
C- L’absorption de la Ciprofloxacine est améliorée en présence
A- Tea delays the absorption of iron
B- Milk slowers the absorption of Fluor
C- The absorption of ciprofloxacin is enhanced in the
du lait
D- L’absorption de l’Itraconazole est améliorée en présence des
boissons gazeuses
presence of milk
D- The absorption of Itraconazole is enhanced in the
presence of soft drinks/ carbonated beverages
155
520. La cinétique de dissolution d’une substance dépend de tous ces The dissolution rate of a substance depends on all the
facteurs SAUF :
following factors EXCEPT:
La forme galénique de la substance
La nature de l’appareillage de dissolution
Les caractères physicochimiques de la substance
Les caractéristiques pharmacodynamiques de la substance
521. Dans les cinétiques de réaction d’ordre zéro, la vitesse de transfert
est :
ABCD-
ABCD-
Variable et indépendante de la concentration
Constante et dépendante de la concentration
Variable et dépendante de la concentration
Constante et indépendante de la concentration
The dosage form of the substance
The type of the dissolution apparatus
The physicochemical characteristics of the substance
The pharmacodynamic properties of the substance
In zero order reaction, the rate of transfer is:
A- Variable and independent of the concentration
B- Constant and concentration-dependent
C- Variable and concentration-dependent
D- Constant and independent of the concentration
ABCD-
522. L’énalapril n’est absorbé oralement que sous forme d’ester Enalapril is orally absorbed as enalaprilat that is an ethyl ester
éthylique d’énalaprilate. Ceci pourrait être expliqué par le fait que la salt. This could be explained by the fact that the ester form:
forme ester:
A- Is absorbed by an active transport system
A- Est absorbée par un système de transport actif
B- Undergoes hydrolysis in the stomach, thus it is dissolved
B- Subit une hydrolyse dans l’estomac, donc dissoute plus
faster
rapidement
C- It is more permeable to the intestinal membrane
C- Est plus perméable à la membrane intestinale
D- It has a lower molecular weight
D- A un poids moléculaire plus faible
523. La classification BCS prend en compte les propriétés suivantes d'un BCS classification takes into account the following properties
médicament SAUF :
of a drug EXCEPT
ABCD-
Solubilité
Dissolution
Perméabilité
Indice thérapeutique
A- Solubility
B- Dissolution
C- Permeability
156
D- Therapeutic index
524. Laquelle parmi ces conditions n’est pas nécessairement prise en Which of the following conditions is not necessary to take in
considération avant l’établissement d’une corrélation in vivo/ in consideration prior the establishment of an in vivo/in vitro
vitro de niveau A ?
level A correlation?
La préexistence de la relation PK/PD
La libération du PA est le facteur limitant
La linéarité de la vitesse d’entrée du P.A.
Peut être envisagée après plusieurs administrations
525. Laquelle parmi les propositions suivantes est correcte ?
The preexistence of the PK/PD relationship
The release of the active ingredient is the limiting factor
The linearity of the rate input of the active ingredient
May be considered after several administrations
Which of the following statements is correct?
ABCD-
ABCD-
A- L’équation Cp = Cp0-kt correspond à une élimination d’ordre 1
B- L’équation lnCp=lnCp0-kt correspond à une équation d’ordre
zéro
C- L’équation Cp=Ae-t+B-t correspond à un modèle
monocompartimental
D- La t½ d’un modèle à 1 compartiment est égale à 0.693/k
526. Laquelle parmi ces assertions concernant le flip-flop est incorrecte :
A- The equation Cp = Cp0-kt is for first-order elimination
B- The equation lnCp=lnCp0-kt is for zero-order elimination
C- The equation Cp=Ae-t+B-t is for one-compartment
model
D- The half-life of a one-compartment model is determine by t1/2
=0.693/k
Which of these statements concerning flip-flop is incorrect :
A- Souvent apparaît avec une demi-vie d’absorption supérieure à
A- Usually used when the absorption half-life is greater than
celle de l’élimination
B- Concerne particulièrement les préparations à libération
prolongée
C- Il aboutit souvent à une augmentation de Tmax
Il aboutit souvent à une diminution du Cmax
the elimination half-life
B- Concerns particularly the sustained-release preparations
C- It often results with an increase of Tmax
D- It often results with a decrease of Cmax
527. Un médicament D a une clairance de 4 L/h et un volume de Drug D has a Clearance of 4 L/hr and a volume of distribution
distribution de 250 L. Combien est sa demi-vie ?
of 250 L. What is the drug half-life?
157
A- Environ 6 minutes
A- About 6 minutes
B- Environ 24 hours
B- About 24 hours
C- Environ 2.5 hours
C- About 2.5 hours
D- Environ 40 hours
D- About 40 hours
528. Les gels peuvent être utilisés pour administrer des médicaments par Gels can be used to administer medications in all of the
toutes les voies suivantes, sauf:
following, except:
ABCD-
A.
B.
C.
D.
Orale
Vaginale
Topique
Sous-cutanée
Orally
Vaginally
Topically
Subcutaneously
529. Quelle serait la date maximale d'utilisation d'une solution What would be the maximum beyond use date of an
extemporanée aqueuse?
extemporaneous aqueous solution?
A.
B.
C.
D.
ABCD-
3 mois
1 mois
14 jours
6 mois
530. Lequel des suivants à propos des émulsions n’est pas correct ?
A. Peuvent être utilisées par voie topique, par voie orale, et par
voie intraveineuse
B. Peuvent être aromatisées afin de surmonter les problèmes de
goût
C. Sont thermodynamiquement stables avec un système à deux
phases
158
3 months
1 month
14 days
6 months
Which of the following is not correct about emulsions?
A. Can be used topically, orally, and intravenously
B. Can be flavored as to overcome taste problems
C. Are thermodynamically stable with a two-phase
system
D. Some emulsions can be self-emulsifying
D. Certaines émulsions peuvent être auto- émulsifiantes
531. Laquelle des préparations suivantes ne nécessite pas généralement Which of the following options does not usually necessitate a
un conservateur?
preservative?
A.
B.
C.
D.
A.
B.
C.
D.
Préparations auriculaires
Préparations nasales
Préparations ophtalmiques en uni-doses
Préparations ophtalmiques multi-doses
Otic preparations
Nasal preparations
Single-dose ophthalmic preparations
Multi-dose ophthalmic preparations
532. Quelle forme galénique des suivantes est appropriée pour une Which of the following dosage forms would be suitable for the
administration d’un médicament afin de traiter une condition administration of medications to treat a systemic condition?
systémique?
A. Ophthalmic preparations
A. Préparations ophtalmiques
B. Otic preparations
B. Préparations auriculaires
C. Vaginal ovules
C. Ovules vaginaux
D. Rectal suppositories
D. Suppositoires
533. Parmi les agents suivants, lequel peut servir comme un humectant, Which of the following agents can serve as a humectant, a
un agent mouillant, un lubrifiant, et un conservateur?
wetting agent, a lubricant, and a preservative?
A.
B.
C.
D.
A.
B.
C.
D.
Gélatine
Alcool
Glycérine
L'huile minérale
Gelatin
Alcohol
Glycerin
Mineral oil
534. Les agents qui accélèrent la pousse des cheveux comprennent tous Hair growth agents include all of the following except
les suivants, sauf :
A. Vasodilators
159
A.
B.
C.
D.
B. Hair follicle stimulants
C. Nourishing agents
D. Exfoliating agents
Les vasodilatateurs
Les stimulants du follicule pileux
Les agents nourrissants
Les agents exfoliants
535. Lequel des énoncés suivants n’est pas considéré comme un critère Which one of the following statements is not considered a
approprié d'un solvant?
criterion of an appropriate solvent?
ABCD-
Il doit être inerte
Il faut qu’il puisse dissoudre le soluté complètement
Quand il est chauffé, il devrait rester sûr et non toxique
Il devrait avoir un effet synergique avec le principe actif.
A. It should be inert
B. It should dissolve the solute completely
C. When it is heated, it should remain safe and nontoxic
D. It should have a synergistic effect with the active
ingredient.
536. Donner un exemple d'un solvant qui est destiné à un usage interne Give an example of a solvent that is intended for internal use
dans la préparation des solutions non aqueuses :
in the preparation of non-aqueous solutions.
ABCD-
L'alcool méthylique
l'alcool amylique
Les ethers volatils
La Glycérine
ABCD-
537. Quelle solution a la plus grande osmolarité?
ABCD-
Methyl alcohol
Amyl alcohol
Volatile ethers
Glycerin
Which solution has the largest osmolarity?
0,20 M KBr dans l'eau
0,20 M de MgCl2 dans l'eau
0,20 M CH3OH dans l'eau
0,20 M Na3PO4 dans l'eau
A- 0.20 M KBr in water
B- 0.20 M MgCl2 in water
160
C- 0.20 M CH3OH in water
D- 0.20 M Na3PO4 in water
538. Une solution est saturée:
A solution is saturated:
B- Quand nous avons besoin de chauffer pour avoir une
dissolution
C- Lorsque le soluté est soluble jusqu'à 10 g / l
D- Lorsque nous ne pouvons pas dissoudre de soluté
supplémentaire
E- Quand elle est préparée avec un tampon
539. En ce qui concerne la solubilité, lequel des énoncés suivants est
vrai?
A- Solide / liquide: augmentation de la température → diminution
de la solubilité
B- Liquide / liquide: augmentation de la température → diminution
de la solubilité
C- Gaz / liquide: augmentation de la température → augmentation
de la solubilité
D- Gaz / liquide: augmentation de la température → diminution de
la solubilité
540. Vous avez un mélange qui est une combinaison de deux ou plusieurs
substances, et il semble homogène partout, même sous un
microscope électronique. Comment on appelle ce mélange?
A- un système dispersé
B- une émulsion
ABCD-
When we need to heat for the dissolution to happen
When the solute is soluble up to 10 g/l
When we cannot dissolve additional solute
When it is prepared with a buffer
Concerning solubility, which of the following is true?
A- Solid/liquid: increase of temperature  decrease of
solubility
B- Liquid/liquid: increase of temperature  decrease of
solubility
C- Gaz/liquid: increase of temperature  increase of
solubility
D- Gaz/liquid: increase of temperature  decrease of
solubility
You have a mixture that is a combination of two or more
substances, and it looks the same throughout, even under an
electronic microscope. What is the term for that mixture?
A- a dispersed system
161
C- une suspension
D- une solution
B- an emulsion
C- a suspension
D- a solution
541. Un patch transdermique:
A transdermal patch:
A- Est utilisé pour avoir un effet local
B- A besoin d'une formation compliquée du malade pour qu’il
sache l'utiliser
C- Est associé à une mauvaise observance du patient
D- Peut provoquer des réactions d'hypersensibilité
542. L'administration par voie intradermique :
ABCD-
ABCD-
Is used to have a local effect
Needs a complex patient training for use
Is associated with bad patient compliance
May induce hypersensitivity reactions
Intradermal route of administration:
Est utilisée uniquement pour les grands volumes
Les préparations doivent être stériles
Est une voie entérale
A le meilleur profil d’absorption du médicament
ABCD-
543. En ce qui concerne les systèmes de délivrance de médicaments par
voie nasale:
A- La biodisponibilité de petites molécules de médicament doit
être améliorée au moyen de substances améliorant l'absorption
B- La concentration de la substance améliorant l'absorption doit
être suffisante pour modifier les propriétés de la cavité nasale
pendant au moins 3 à 4 mois après l'utilisation
C- Les substances améliorant l'absorption utilisées ne doivent pas
être irritantes
D- Le piégeage du médicament par le mucus nasal va augmenter
162
Used only for large volumes
Preparations must be sterile
Is an enteral route of administration
Has the best drug absorption profile
Concerning the nasal drug delivery systems:
A- The bioavailability of smaller drug molecules should be
improved by means of absorption enhancers
B- The concentration of the absorption enhancer used should
be high enough to modify the nasal cavity properties for at
least 3 to 4 months after use
C- The absorption enhancer used should not be irritating
D- Trapping of the drug by the nasal mucus will increase its
son effet systémique
systemic effect
544. Tous les énoncés suivants concernant les vaporisateurs nasaux sont
vrais, sauf:
ABCD-
Ils ont une meilleure absorption que celle des gouttes nasales
Le médicament est placé dans une solution ou suspension
Ils sont constitués de jet atomisé de liquide
Ils subissent une perte rapide après l’administration
All the following statements concerning nasal sprays are true
except:
A- They have a better absorption than nasal drops
B- The drug is put in a solution or suspension
C- They consist of atomized jet of liquid
D- They undergo quick loss after administration
545. Les caractéristiques des gels nasaux comparées aux solutions
incluent:
ABCD-
Une absorption inferieure
Plus d’écoulement retronasal
Plus de fuite antérieure de la formulation
Temps de séjour superieur
The characteristics of nasal gels over solutions include:
A- Lower absorption
B- More post-nasal drip
C- More anterior leakage of the formulation
D- Higher residence time
546. Lequel des énoncés suivants est vrai en ce qui concerne l'absorption
systémique nasale?
Which one of the following statements is true regarding nasal
systemic absorption?
A- L'hydrosolubilité supérieure donne une meilleure absorption
B- Poids moléculaire bas donne une perte par fuite élevée
C- Une grande sensibilité à la dégradation enzymatique réduit
A- Higher the hydrosolubility, better the absorption
B- Lower the molecular weight, higher the leakage loss
C- Higher the susceptibility to enzyme degradation, lower the
l'absorption
D- Poids moléculaire élevé donne une meilleure absorption
absorption
D- Higher the molecular weight, better the absorption
163
547. Lequel des énoncés suivants est vrai en ce qui concerne l'absorption
de médicaments après administration pulmonaire?
Which one of the following statements is true regarding
absorption of drugs after pulmonary administration?
A- L'absorption est plus élevée au niveau de la trachée en raison de A- Absorption is higher at the trachea level due to large
la grande surface de contact
B- L'étroitesse des jonctions cellulaires est maximale dans la
trachée et minimale dans les voies aériennes distales
C- Les macrophages présents au niveau alvéoles augmentent la
biodisponibilité des médicaments
D- Il n'y a aucune absorption réelle après administration
pulmonaire
548. Les facteurs propres au patient qui affectent l’absorption
pulmonaire des médicaments comprennent tous les éléments
suivants, sauf:
ABCD-
Compliance au traitement
Poids
Coordination de la production d'aérosols avec l'inspiration
Le volume inhalé
549. Lequel des énoncés suivants sur la clairance mucociliaire est vrai?
A- La clairance mucociliaire est pertinente dans les voies auriculaire
et oculaire de l'administration des médicaments
B- La clairance mucociliaire inclut le rôle des macrophages dans les
alvéoles pulmonaires
C- La clairance mucociliaire augmente généralement la
biodisponibilité des médicaments
D- La clairance mucociliaire est un facteur limitant pour le passage
164
surface of contact
B- The tightness of cell junctions goes down from a maximum
in the trachea to a minimum in the distal airways
C- Macrophages present at the alveoli level increase drugs
bioavailability
D- There is no real absorption after pulmonary administration
The patient dependent factors that affect pulmonary drugs
absorption include all of the following except:
ABCD-
Compliance
Weight
Coordination of aerosol generation with inspiration
Inhaled volume
Which one of the following statements about mucociliary
clearance is true?
A- Mucociliary clearance is relevant in otic and ocular routes
of drugs administration
B- Mucociliary clearance includes the role of macrophages in
the lungs alveoli
C- Mucociliary clearance usually increases drugs
bioavailability
des médicaments dans le sang
D- Mucociliary clearance is a limiting factor for absorption of
drugs to the blood
550. Concernant les nébuliseurs:
Concerning nebulizers:
A- Ils livrent la quantité totale d'un médicament en secondes
B- Ils sont spécifiquement utilisés pour les petites doses de
médicaments
C- Ils ne nécessitent pas de propulseur.
D- Ils nécessitent une bonne coordination du patient
A- They deliver the total quantity of a drug in seconds
B- They are specifically used for small drug doses
C- They do not require propellant
D- They require patient coordination
551. Concernant les inhalateurs de poudre sèche pré-dosés:
Concerning Pre-metered DPIs (Dry powder inhalers):
A- Le médicament est le plus souvent dans une solution
B- Ils comprennent des doses mesurées à l’avance ou des fractions
de doses
C- Ils ont besoin d'un gaz propulseur
D- Ils ont un réservoir interne contenant une quantité suffisante de
formulation pour plusieurs doses
A- The medication is most commonly stored in solution
B- They include previously measured doses or dose fractions
C- They need a propellant gas
D- They have an internal reservoir containing sufficient
formulation for multiple doses
Which of the following statements is true regarding niacin?
A- Absolutely contraindicated in patients with hyperuricemia
B- Associated with risk of hypoglycemia
C- SR niacin is the most hepatotoxic dosage form of niacin
D- Highly effective for LDL-C reduction
552. Lequel des énoncés suivants concernant la niacine est vrai ?
a- Absolument contrindiquée chez les patients
hyperuricémiques
b- Associée à un risque d’hypoglycémie
c- NIACIN SR est la forme la plus toxique de niacin
d- Très efficace pour réduire le LDL cholestérol
553. Un homme de 66 ans avec CAD, DMII and HTN commence un
traitement par bisoprolol et ramipril.quelques jours après, il
165
A 66 years old male with CAD, DMII and HTN was started on
bisoprolol and ramipril. Few days after starting the drug
développe un œdème sévère de la langue et du cou et peut à peine
respirer. Lequel des suivants recommandez-vous ?
a- Stop ramipril imédiatement et
commencer aliskiren.
b- Stop ramipril imédiatement et
commencer lisinopril.
c- Stop ramipril imédiatement et
commencer valsartan
d- Continuer ramipril et ajouter valsartan
554. SF est un homme de 75 ans ayant une tension artérielle de base de
170 mm ded Hg et traitépar valsartan rt hydrochlorothiazide 25mg
en dose quotidienne unique. Il se présente chez le médecin avec
une tension de 147/88 mm de Hg. Lequel des suivants
recommandez-vous ?
abcd-
Ajouter amlodipine
Ajouter lisinopril
Ajouter furosémide
Ne rien changer au traitement
555. Une femme de 39 ans enceinte au 7ème mois se présente chez son
gynécologue pour sa visite mensuelle. Le medecin trouve à l’examen
clinique une tension artérielle de 158/95 mm de Hg et l’analyse des
urines montre une protéinurie. lequel des suivants considérez-vous :
a- Pas de traitement anti hypertenseur puisque la TA est <
160mm Hg
b- Commencer lisinopril
c- Commencer bisoprolol
166
therapy he developed a severe swelling in his tongue and neck
and could barely breathe. Which of the following would you
recommend?
ABCD-
Stop ramipril abruptly and start aliskiren
Stop ramipril abruptly and start lisinopril
Stop ramipril abruptly and start valsartan
Continue ramipril and add valsartan
SF is a 75 year old male with a baseline BP of 170mmHg, has
been treated with valsartan 160mg and hydrochlorothiazide
25mg as a single pill taken once daily. He presents to the
physician and his BP is 147/88mmHg. Which of the following
would you recommend?
A- Add amlodipine
B- Add lisinopril
C- Add furosemide
D- No change in the regimen
A 39 years old pregnant female in her 7th month presents to
her gynecologist for her monthly work up. Upon physical exam
her BP is 158/95mmHg and urinanalysis reveals proteinuria.
Which of the following would you consider?
A- No antihypertensive treatment since the SBP
is<160mmHg
B- Start lisinopril
C- Start bisoprolol
D- Start labetalol
d- Commencer labetolol
556. Un homme de 80 ans ave histoire d’hypertension (pour au moins 10
ans) et de migraine. Aujourd’hui sa TA est 158/72 mm Hg’ le pouls
est 58/mn’ créatinine 1.2mg/dl, potassium 4.3mEq/l. il prend
actuellement lisinopril 40mg/jour et vérapamil SR 240 mg/jour.il
pèse 73 Kg, sa taille est ?? il fume un paquet de cigarettes par
jour,et consomme 2-3 cannettes de boissons alcooliques par
semaine.lequel est le meilleur à ajouter à son traitement :
abcd-
Spironolactone 25mg/j
Amlodipine 5 mg/j
Losartan 50 mg/j
Hydrochlorothiazide 12.5 mg/j
557. Un homme agé de 65 ans hypertendu et dyslipidémique’ prenant
amlodipine 5mg/j , et atorvastatin 20 mg/j. il rapporte etre en trè
bonne condition. Il se présente chez le medecin aujourdhui pour un
examen de contrôle. Sa TA est 133/80 mm Hg, lequel des suivants
considérez-vous :
a- Garder le meme traitement et encourager
une bonne observance
b- Augmenter la dose de amlodipine à
10mg/j pour atteindre TA <130mm Hg
c- Diminuer la dose d’amlodipine à 2.5 mg/j
pour augmenter la TA
d- Ajouter hydrochlorothiazide 25 mg/j
167
An 80-year-old man has a past medical history of hypertension
(for the past 10 years), and migraine headaches. His BP today
is 158/72 mm Hg (156/70 mm Hg when repeated), heart rate is
58 beats/minute, serum creatinine is 1.2 mg/dL, and
potassium is 4.3 mEq/L. He is currently on lisinopril 40 mg daily
and verapamil SR 240 mg daily, weighs 73 kg, is 70 in tall,
smokes cigarettes 1 pack/daily, and consumes two to three
ethanol-containing drinks weekly. Which is the most
appropriate recommendation to add to his antihypertensive
regimen?
A- Spironolactone 25 mg daily
B- Amlodipine 5 mg daily
C- Losartan 50 mg daily
D- Hydrochlorothiazide 12.5mg daily
A 65 years old male known to have hypertension and
dyslipidemia, on amlodipine 5mg po once daily and
atorvastatin 20mg po once daily respectively. He reports no
symptoms and explains, “he has never felt any better”. He
presents today to the physician for regular follow up. His BP is
133/80mmHg which of the following would you consider?
A- Keep on the same regimen and encourage on continued
compliance to drug and non pharmacological treatment
B- Increase the dose of amlodipine to 10mg orally once
daily to reach a SBP<130mmHg
C- Decrease the dose of amlodipine to 2.5mg once daily to
increase the SB
D- Add hydrocholothiazide 25mg po once daily
558. Une femme de 65 ans récemment diagnostiquée de diabète et
d’hypertension.elle est connue etre hyperuricémique et a eu 2
attaques de goutte durant l’année précédente.les données de
laboratoire sont normales et elle n’a pas d ‘allergiesconnues à des
médicaments ou aliments. Le medecin demande vos
recommandations d’après les JNC8 guideline?
abcd-
Hydrochlorothiazide
Nitrendipine
Ramipril
Any of the above
559. Un homme de 65 ans hypertendu et diabétique dévelope
récemment un dysfonctionnement rénal avec albuminurie. Aujourd
hui sa ClCr est 40ml/mn et sa TA 148/95 mm hg. Quelle TA doit-on
viser pour lui :
abcd-
<150/90 mm Hg
<140/90 mm Hg
140/85 mm Hg
130/90 mm hg
560. Laquelle des statines suivantes est recommandée chez une femme
enceinte ayant une hyperlipémie :
abcd-
Atorvastatine
Rosuvastatine
Lovastatine
aucune
A 65 years old black female recently diagnosed with
hypertension and diabetes. She is known to have
hyperuricemia with 2 gout attacks in the previous year. Her lab
tests are all normal and when asked about allergies, she has no
known food or drug allergies. The physician is asking for your
recommendation. Which of the following would you
recommend according to JNC8 guideline?
A- Hydrochlorothiazide (CI in gout)
B- Nitrendipine
C- Ramipril
D- Any of the above
A 65 years old male with hypertension and diabetes developed
recently renal dysfunction with albuminuria. Today his CrCl is
40ml/min and his BP is 148/95mmHg. What is the goal BP
according to JNC8?
A- <150/90mmHg
B- <140/90mmHg
C- <140/85mmHg
D- <130/90mmHg
Which of the following statins is recommended for a pregnant
woman with dyslipidemia?
ABCD-
168
Atorvastatin
Rosuvastatin
Lovastatin
None of the above
561. Quelle est la thérapie la plus appropriée pour une femme de 58 ans
diabétique type 2, son chol total est 210 mg/dl, HDL chol 52mg/dl,
trigl 958mg/dl et LDL chol 131 mg/dl ?
abcd-
Fenofibrate 160 mg/j
Atorvastatin 40 mg/j
Pravastatin 20 mg/j
Cholestyramine 2 fois/j
Which type of therapy would be most appropriate for a 58year-old female with type 2 diabetes and total cholesterol 210
mg/dL, HDL cholesterol 52 mg/dL, triglycerides 958 mg/dL, and
LDL cholesterol 131 mg/dL?
A- Fenofibrate 160 mg daily
B- Atorvastatin 40 mg daily
C- Pravastatin 20 mg daily
D- Cholestyramine twice daily
562. Un homme de 45 ans obèse, sinon en bonne santé, se présente pour
un examen de routine les résultats sont les suivants :
TA 135/85mm Hg, gluc à jeune 115mg/dl, TG 180 mg/dl, LDL
195mg/dl, HDL 38mg/dl.
Lequel des suivants recommandez-vous ?
a- Commencer un traitement anti hypertenseur avec
hydrochlorothiazide
b- Commencer un traitement hypolipémiant avec simvastatin
40 mg/j
c- Commencer un traitement hypolipémiant avec rosuvastatin
20 mg
d- Le traitement médical n’est pas encore recommandé donc
juste des conseils non pharmacologiques.
563. Une femme de 56 ans fume 2 paquets de cigarette s par jour est
admise aux urgences pour une douleurthoracique sévère du côté
169
A 45 year old male overweight but otherwise healthy
performed a routine yearly checkup. The results were the
following: BP: 135/85mmHg, Fasting blood glucose 115mg/dl,
TG180mg/dl, LDL195mg/dl, HDL38mg/dl. which of the
following would you recommend at this point?
ABCD-
Start antihypertensive drug with Hydrochlorothiazide
Start antihyperlipidemic drug with simvastatin 40mg
Start antihyperlipidemic drug with rosuvastatin 20mg
Drug therapy is not yet indicated so just counsel on nonpharmacologic treatment
A 56-year-old female who smokes two packs of cigarettes per
day is admitted to the emergency room with severe left-sided
gauche et engourdissement du bras gauche. Elle fut diagnostiquée
pour syndrome coronaire aigu .son profile lipidique montre :
Chol total 229mg/dl
HDL 48mg/dl
LDL 152mg/dl.
Quel agent hypolipémiant sera le plus approprié pour sa thérapie :
a- Une thérapie médicale ne sera pas
efficace et n’est pas indiquée
b- Ezetimibe 10 mg/j
c- Omega 3 1g 2 fois/j
d- Atorvastatin 80mg/j
564. Une femme de 42 ans diabétique et hypertendue traitée par
sulfonylurée et ramipril . son taux de TG 340mg/dl, HDL42 mg/dl et
LDL 168mg/dl.
BCDE-
Lovastatin 20mg
Simvastatin 10mg
Atorvastatin 20mg
Rosuvastatin 20mg
chest pain and numbness down her left arm. She is diagnosed
with acute coronary syndrome and her lipid profile was
obtained in the emergency room and reveals the following:
total cholesterol 229 mg/dL, HDL cholesterol 48 mg/dL,
triglycerides 147 mg/dL, and LDL 152 mg/dl. What would be
the most appropriate lipid-lowering therapy?
A- Drug therapy is not beneficial and is not indicated
B- Ezetimibe 10 mg daily
C- Omega-3 fatty acids 1 g twice daily
D- Atorvastatin 80 mg daily
A 42-year-old woman with type 2 diabetes mellitus and HTN
treated with a sulfonylurea, and ramipril respectively. Her
TG340 mg/dL, and HDL42 mg/dL and LDL168mg/dl. Based on
pooled cohort equations, it was found that DR’s 10-yr risk for
developing a ASCVD event is 6.7%. What is the most suitable
drug to be initiated in DR at this stage?
A.
B.
C.
D.
565. Une femme de 38 ans hyperlipémique, se présente à la clinique
pour aggravation de ses douleurs musculaires. Son taux de chol était
167mg/dl, LDL 42mg/dl, HDL 42mg/dl, et les TG 254mg/dl lors de sa
dernière visite.il a été décidé d’ajouter un autre agent
hypolipémiant à sa statine. Elle n’avait pas senti auparavant des
douleurs musculaires et se demandait si le médicament ajouté ne
serait responsable de ses douleurs. Quel médicament avait-on
170
Lovastatin 20mg
Simvastatin 10mg
Atorvastatin 20mg
Rosuvastatin 20mg
A 38-year-old woman with hyperlipidemia presents to the
primary care clinic with worsening muscle aches. Her total
cholesterol was 268 mg/dL, her LDL was 167 mg/dL, her HDL
was 42 mg/dL, and her triglyceride was 254 mg/dL during her
last visit. The decision was made to add another lipid-lowering
medication to her simvastatin. She never had muscle aches
before and is wondering if the new medication is causing her
probablement ajouté à la simvastatine :
abcd-
Cholestyramine
Colestipol
Ezetimibe
Gemfibrozil
566. Une femme de 35 ans semble avoir une hyperlipémie familiale
combinée.son chol total et son taux de TG sont élevés. Son taux de
HDL est diminué. Lequel des médicaments suivants,utilisé en
monothérapie est le plus adapté pour diminuer son taux de chol et
VLDL :
abcd-
Atorvastatin
Cholestyramine
Ezetimibe
gemfibrosil
567. Lequel des suivants est une toxicité majeure associée avec la
gemfibrosil :
abcd-
Flatulence et constipation
Cholethiasis
Hyperuricémie
Atteinte hépatique
new pain. What medication was most likely added to her
simvastatin?
A- Cholestyramine
B- Colestipol
C- Ezetimibe
D- Gemfibrozil
A 35 year old woman appears to have familial combined
hyperlipidemia. Her serum concentrations of total cholesterol,
LDL cholesterol, and triglycerides are elevated. Her serum
concentration of HDL cholesterol is somewhat reduced. Which
of the following drugs is most likely to increase this patient’s
triglyceride and VLDL cholesterol concentrations when used as
monotherapy?
A- Atorvastatin
B- Cholestyramine
C- Ezetimibe
D- Gemfibrozil
Which of the following is a major toxicity associated with
gemfibrozil therapy?
A- Bloating and constipation
B- Cholelithiasis
C- Hyperuricemia
D- Liver damage
The pleiotropic effects of statins include:
A- Enhanced fibrinogen expression
B- Decreased nitric oxide production
C- Decreased inflammation
D- Enhanced smooth muscle remodeling
568. Les effets pléiotropiques des statinescomportent :
a- Expression du fibrinogène augmentée
b- Diminution de la production de l’oxyde nitrique
c- Diminution de l’inflammation
171
d- Amélioration du remodelage des muscles lisses
569. Selon les nouvelles recommandations de la prise en charge de
ACC/AHA, les patients suivants doivent être traités par des statines
SAUF :
a- Un homme sain de 30 ans ayant LDL de 80mg/dl et sans
aucun facteur de risque CV
b- Une femme de 46 ans diabétique ayant LDL de 80mg/dl
c- Un homme de 60 ans ayant LDL de 110 mg/dl et un risque
CV estimé à 10 % depuis 10 ans
d- Une femme de 45 ans avec une histoire de choc ischémique
et une valeur LDL inconnue.
570. Dragéification des comprimés:
According to the new recommendations for cholesterol
management from the ACC/AHA, all of the following patients
should be considered for statin therapy except:
A- A 30-year-old healthy man with an LDL of 165 mg/dL
and no CV risk factors
B- A 46-year-old healthy woman with a history of diabetes
and an LDL of 80 mg/dL
C- A 60-year-old man with an LDL of 110 mg/dL and a 10year estimated risk for CV disease of 10 %
D- A 45-year-old man with a history of ischemic stroke and
unknown LDL level
Sugar-coating of tablets:
A. Conserve le contour de la tablette originale
A. Retains the contour of the original tablet
B. Augmente le poids substantiellement de 30 à 50%
B. Increases weight substantially by 30 to 50%
C. Permet le revêtement multi particules
C. Allows multi-particulate coating
D. Nécessite une seule étape à appliquer
D. Requires a single step only to apply
172

Documents pareils

No. Français English 1 Comment sont des médicaments de

No. Français English 1 Comment sont des médicaments de D. Les patients pédiatriques ont un taux métabolique plus élevé du CYP3A4 Une femme de 53 ans ingère la moitié d'une bouteille de comprimés d'aspirine et se présente à l'urgence avec des vomissemen...

Plus en détail